ELEMENTE DER ZAHLENTHEORIE UND AUFBAU DES

Werbung
ELEMENTE
DER ZAHLENTHEORIE
UND
AUFBAU DES ZAHLENSYSTEMS
von
Rolf Waldi
Inhaltsverzeichnis
Kapitel I. Elementare Zahlentheorie
§1
§2
§3
§4
§5
§6
§7
§8
§9
Grundlegende Regeln und Prinzipien
Teilbarkeit in Z
Primzahlen
Vollkommene Zahlen
Kongruenzrechnung
Lineare Kongruenzen
Der kleine Satz von Fermat
Summen von Quadraten
Pythagoras–Tripel
3
12
19
25
31
36
43
50
55
-
11
18
24
30
35
42
49
54
59
60
66
74
80
-
65
73
79
83
Kapitel II.
Algebraische Grundbegriffe
§1
§2
§3
§4
Ringe und Körper
Restklassenringe und Polynomringe
Gruppen
Die prime Restklassengruppe modulo p
Kapitel III.
Aufbau des Zahlensystems
§1
§2
§3
§4
§5
§6
§7
§8
Addition und Multiplikation natürlicher Zahlen
Der Ring der ganzen Zahlen
Die g–adische Darstellung natürlicher Zahlen
Die rationalen Zahlen
Die reellen Zahlen
Konvergente Folgen
Dezimalbrüche
Die komplexen Zahlen
1
84 - 92
93 - 96
97 - 101
102 - 106
107 - 113
114 - 120
121 - 127
128 - 135
Anhang A: Die Gleichung X 3 + Y 3 = Z 3
§1
§2
§3
§4
§5
Teilbarkeit in Integritätsbereichen
Euklidische Ringe
Imaginär – quadratische
Zahlbereiche
√
Der Ring Z[− 21 + i 23 ]
Die Gleichung X3 + Y3 = Z3
136
143
148
153
159
-
142
147
152
158
162
-
168
175
181
186
191
Anhang B: Quadratische Irrationalzahlen
§1
§2
§3
§4
§5
Reel-quadratische Zahlkörper
Die Pell’sche Gleichung
Anwendungen der Pell’schen Gleichung
Kettenbrüche
Periodische Kettenbrüche u. quadr. Irrationalzahlen
2
163
169
176
182
187
Kapitel I. Elementare Zahlentheorie
§1 Grundlegende Regeln und Prinzipien
Es wird vorausgesetzt, daß der Leser mit ganzen Zahlen rechnen kann und
mit der Mengenschreibweise vertraut ist. Bis auf weiteres bedeuten kleine
Buchstaben
a, b, c, . . . , x, y, z
ganze Zahlen, das heißt sie stehen für
1, 2, 3, . . .
0
−1, −2, −3, . . .
(positive ganze Zahlen)
(Null)
(negative ganze Zahlen)
Die natürlichen Zahlen sind bei uns die Zahlen
0, 1, 2, 3, . . .
Ihre Gesamtheit wird mit N bezeichnet. Z steht für die Gesamtheit aller
ganzen Zahlen
. . . , −3, −2, −1, 0, 1, 2, 3, . . .
Wir werden von den folgenden Regeln für das Rechnen mit ganzen Zahlen
Gebrauch machen:
Verknüpfungsregeln der Addition
(1) (a + b) + c = a + (b + c)
(Assoziativgesetz)
(2) a + b = b + c
(Kommutativgesetz)
(3) a + (−a) = 0
(−a ist inverses Element zu a bzgl. +)
(4) 0 + a = a
(0 ist neutrales Element bzgl. +)
Daraus ergibt sich
(1.1) Satz. Jede Gleichung X + a = b mit a, b ∈ Z hat in Z eine Lösung,
und nur eine.
Das soll heißen: Zu jedem Paar a, b ganzer Zahlen gibt es genau eine Zahl
x ∈ Z, so daß x + a = b.
Beweis.
Existenz: Setze x := b + (−a). Dann ist (nach den Regeln (1) bis (4))
x + a = (b + (−a)) + a = b + ((−a) + a) = b + (a + (−a)) = b + 0 = 0 + b = b
3
Eindeutigkeit: Sei y ∈ Z beliebig mit y + a = b. Dann ist
b + (−a) = (y + a) + (−a) = y + (a + (−a)) = y + 0 = y
Verknüpfungsregeln der Multiplikation
(5) (a · b) · c = a · (b · c)
(Assoziativgestz)
(6) a · b = b · a
(Kommutativgesetz)
(7) 1 · a = a
(1 ist neutrales Element bzgl. ·)
(8) a · b = 0
dann u. nur dann, wenn a = 0 oder b = 0
Distributivgesetz
(9) a · (b + c) = (a · b) + (a · c)
Für a + (−b) schreibt man auch a − b. Dann gilt auch
a · (b − c) = (a · b) − (a · c)
Wir verwenden gelegentlich die Abkürzungen
=⇒“ für daraus folgt“,
”
”
⇐⇒“ für genau dann wenn“, und
”
”
:= “ für ist definitionsgemäß gleich“.
”
”
(1.2) Kürzungsregel. Aus a · b = a · c und a 6= 0 folgt b = c.
(9)
Beweis. a · b = a · c =⇒ (a · b) − (a · c) = (a · c) − (a · c) = 0 =⇒ a · (b − c) =
(8)
(a·b)−(a·c) = 0 =⇒ b−c = 0 =⇒ c = (b−c)+c = b+((−c)+c) = b+0 = b.
Da für Addition und Multiplikation das Assoziativgesetz gilt, kann man sich
viele Klammern sparen: Schreibe:
a+b+c
für
(a + b) + c
a·b·c
für
(a · b) · c
Damit sind auch endliche Summen und Produkte ohne Klammern erklärbar:
a1 + a2 + a3 + a4 := (a1 + a2 + a3 ) + a4
..
.
a1 + a2 + . . . + an−1 + an := (a1 + . . . + an−1 ) + an
a1 · a2 · a3 · a4 := (a1 · a2 · a3 ) · a4
..
.
a1 · a2 · . . . · an−1 · an := (a1 · . . . · an−1 ) · an
4
Durch die Regelung Punktrechnung geht vor Strichrechnung“ können wei”
tere Klammern eingespart werden:
a · b + c · d steht für (a · b) + (c · d)
Ferner wird der Malpunkt oft weggelassen und man verwendet die abkürzende
Schreibweisen
k
X
an := a1 + . . . + ak ;
n=1
k
Y
an := a1 a2 . . . an ; an = a
. . . a}
| · {z
n=1
n-mal
Wichtig ist die
Formel für die endliche geometrische Reihe
(1 + x + x2 + . . . + xn−1 )(1 − x) = 1 − xn
Beweis.
(1 + x + . . . + xn−1 )(1 − x) =
1 + x + x2 + . . . + xn−2 + xn−1
−(x + x2 + . . . + xn−2 + xn−1 + xn )
= 1
− xn
Anordnung der ganzen Zahlen. Z ist in natürlicher Weise angeordnet:
. . . − 5, −4, −3, −2, −1, 0, 1, 2, 3, 4, 5, . . .
Definition. a heißt kleiner als b( a < b“) wenn a in der obigen Reihung
”
links von b steht. Schreibe auch a > b (a größer als b) falls b < a. Damit gilt
offensichtlich: a > 0 genau dann, wenn −a < 0.
Anordnungsregeln
(1) Es gilt entweder a > b oder a = b oder a < b
(2) Aus a > b und b > c folgt a > c
(3) Aus a > b folgt a + c > b + c
(4) Aus a > b und c > 0 folgt ac > bc
Schreibweisen
a ≥ b steht für a > b oder a = b“.
”
a ≤ b steht für a < b oder a = b“.
”
5
Definition. Für a ∈ Z definiert man den Betrag |a| von a als
(
a, falls a ≥ 0
|a| :=
−a, falls a < 0
Betragsregeln
(1) |a| ≥ 0; |a| = 0 ⇐⇒ a = 0
(2) |ab| = |a| |b|
(3) |a + b| ≤ |a| + |b|
Minimum und Maximum. Seien a1 , . . . , an ganze Zahlen. Die kleinste der
Zahlen a1 , . . . , an heißt das Minimum und die größte das Maximum von
a1 , . . . , an . Schreibe dafür Min(a1 , . . . , an ) bzw. Max(a1 , . . . , an )
Grundprinzipien. Wir wollen das folgende Prinzip anerkennen:
(P) Prinzip vom kleinsten Element. Jede nicht leere Menge A von
natürlichen Zahlen besitzt ein kleinstes Element, d.h.:
Es gibt ein Element a0 ∈ A, so daß x ≥ a0 für alle x ∈ A. Schreibe dann
a0 = MinA. Aus diesem Prinzip lassen sich weitere Grundsätze ableiten:
A(x) bezeichne eine Aussage, die für eine natürliche Zahl x zutreffen kann,
d.h.:
Für x ∈ N gilt entweder A(x) oder A(x) gilt nicht.
Beispiele
a) A(x) sei die Aussage
x ist eine gerade Zahl“.
”
Die Aussagen A(1), A(3), A(5) sind falsch.
Die Aussagen A(2), A(4), A(6) sind wahr (richtig).
b) A(x) sei die Aussage
1 + 2 + 3 + ... + x =
x(x + 1)
für x ≥ 1
2
A(x) ist für alle x ≥ 1 richtig, also allgemein gültig.
(G) Prinzip vom kleinsten Gegenbeispiel. Sei A(x) eine Aussage über
natürliche Zahlen x ≥ a0 . Ist A(x) nicht allgemein gültig, so gibt es dafür ein
kleinstes Gegenbeispiel. Das soll heißen:
Ist A(x) falsch für wenigstens ein x ≥ a0 , so gibt es eine Zahl a ≥ a0 , so daß
6
gilt:
(1) A(a) ist falsch.
(2) A(x) ist richtig für alle x ∈ N mit a0 ≤ x < a.
(G) ergibt sich leicht aus dem Prinzip vom kleinsten Element:
Sei N = {x|x ∈ N, x ≥ a0 und A(x) ist falsch}. Nach Voraussetzung ist N
nicht die leere Menge ∅. Nach (P) existiert daher das Minimum a = Min N .
Zeige nun, daß a die Bedingungen (1) und (2) erfüllt.
Zu (1): Wegen a ∈ N ist A(a) falsch.
Wegen a = MinN ist
(∗)
x ≥ a für alle x ∈ N .
Zu (2): Sei x ∈ N mit a0 ≤ x < a. Wegen (∗) ist dann x 6∈ N . Nach Definition
von N bedeutet dies, daß A(x) richtig ist.
Aus dem Prinzip vom kleinsten Gegenbeispiel ergibt sich leicht das wichtigere
Prinzip der vollständigen Induktion.
(V) Prinzip der vollständigen Induktion. Sei A(x) eine Aussage über
natürliche Zahlen x mit folgenden beiden Eigenschaften:
(i) A(0) ist richtig.
(ii) Für alle n ∈ N folgt aus der Richtigkeit von A(n) schon die Richtigkeit
von A(n + 1).
Dann ist A eine allgemein gültige Aussage, d.h.
A(x) ist richtig für alle x ∈ N.
Das Induktionsprinzip (V) läßt sich aus (G) herleiten:
Sei A(x) eine Aussage mit den Eigenschaften (i) und (ii). Angenommen A
wäre nicht allgemein gültig. Dann gibt es gemäß (G) zu A ein kleinstes Gegenbeispiel a, d.h.: A(a) ist falsch und A(b) ist richtig für alle b < a. Wegen
(i) ist dann a > 0, also a − 1 ≥ 0 ist eine natürliche Zahl. Nach Wahl von a
ist ferner A(a − 1) richtig. Wende nun (ii) auf n = a − 1 an: Aus der Richtigkeit von A(a − 1) folgt die Richtigkeit der Aussage A((a − 1) + 1) = A(a),
Widerspruch!
Also ist die Ausnahme A ist nicht allgemein gültig“ falsch, d.h. A ist allge”
mein gültig. Die Gültigkeit des folgenden Beweisprinzips ergibt sich ebenfalls
aus dem Prinzip vom kleinsten Gegenbeispiel:
Schema eines Beweises durch vollständige Induktion. Sei A(x) eine
Aussage über natürliche Zahlen x mit x ≥ a0 . Es soll gezeigt werden, daß
A(x) für alle x ≥ a0 richtig ist. Dazu geht man folgermaßen vor:
7
(a) Man zeigt, daß A(a0 ) richtig ist (Induktionsbeginn).
(b) Man nimmt an, daß A(x) gilt für alle x ∈ N mit a0 ≤ x ≤ n (Induktionsannahme (Induktionsbehauptung)).
(c) Man schließt aus (a) und (b), daß auch A(n + 1) gilt (Induktionsschluß).
Beispiele von Beweisen durch vollständige Induktion
(a) A(x) sei die Aussage: 1 + 2 + . . . + x =
Induktionsbeginn: A(1) : 1 =
1+1
2
x(x+1)
2
(a0 = 1)
ist richtig.
Induktionsannahme: Für alle x ∈ N mit 1 ≤ x ≤ n gilt
A(x) : 1 + 2 + . . . + x = x(x+1)
.
2
Induktionsschluß: Nach Induktionsannahme ist A(n) richtig, d.h.
1 + 2 + . . . + n = n(n+1)
2
Es folgt
1 + 2 + . . . + n + (n + 1) = n(n+1)
+ (n + 1) = n(n+1)+2(n+1)
= (n+2)(n+1)
=
2
2
2
(n+1)((n+1)+1)
, d.h. A(n + 1) ist richtig.
2
(b) Der binomische Lehrsatz. Seien n und ν ∈ N.
Fakultäten: 0! := 1; n! := 1 · 2 · 3 · . . . · (n − 1)n für n ≥ 1
(1! = 1; 2! = 1 · 2 = 2; 3! =1 ·2 · 3 = 6; 4! = 1· 2 · 3 · 4 = 24, . . .)
n
n
Binomialkoeffizienten:
:= 1 und
:= n(n−1)·...·(n−ν+1)
für
ν!
0
ν
ν≥1
1.3 Regel.
n
n
n
(a)
= 1,
= 0 für ν > n und
=
n
ν
ν
n
n
(b)
=
für 0 ≤ ν ≤ n
ν
n−ν
n
n
n+1
(c)
+
=
für ν ≥ 1
ν−1
ν
ν
8
n!
ν!(n−ν)!
für 0 ≤ ν ≤ n
n
(d)
∈ N.
ν
Beweis.
n
(a)
= n(n−1)·...·(n−n+1)
= n·(n−1)·...·1
=1
1·2·...·n
1·2·...·n
n
ν > n =⇒ n + 1 ≤ ν =⇒n +
1 − ν ≤ 0 =⇒ n(n − 1) · . . . · (n − ν + 1)
n
enthält 0 als Faktor =⇒
= ν!0 = 0.
ν
0 ≤ ν < n =⇒ n(n − 1) · . . . · (n − ν + 1) = n·(n−1)·...·(n−ν+1)·(n−ν)·...·2·1
=
(n−ν)·...·2·1
n
n!
n!
=⇒
= (n−ν)!ν!
(n−ν)!
ν
n
n
n!
n!
=
=
(b) Nach (a) ist
= ν!(n−ν)! und
(n−ν)! (n − (n − v))!
n−ν
ν
|
{z
}
ν
n
n!
=
(n−ν)!ν!
ν
n
n
+
= n(n−1)·...·(n−(ν−1)+1)
(c)
+ n(n−1)·...·(n−ν+1)
=
(ν−1)!
ν!
ν
ν−1
n(n−1)·...·(n−(ν−1)+1)·(ν+n−ν+1)
= n(n−1)·...·(n−ν+2)(n+1)
= (n+1)n·...·((n+1)−ν+1)
ν!
ν!
ν!
n+1
=
ν
(d) Beweise die folgende Aussage A(m) für alle natürlichen m durch Induktion nach m:
A(m):
”Für alle Paare n, ν natürlicher Zahlen mit n + ν = m ist
n
∈ N.“
ν
n
0
Induktionsbeginn: m = 0 =⇒ n = ν = 0 =⇒
=
=1∈N
ν
0
Induktionsannahme:
A(x) ist richtig für alle x ∈ N mit x ≤ m, d.h.:
n
∈ N für alle Paare n, ν ∈ N mit n + ν ≤ m.
ν
9
Induktionsschluß: Seien
n, ν natürliche Zahlen mit n + ν = m + 1.
n
∈ N.
Es ist zu zeigen, daß
ν
n
0
Nach Definition ist
= 1 ∈ N, und
= 0 für alle ν ≥ 1
0
ν
nach (a). Also kann man annehmen, daß n ≥ 1 und ν ≥ 1, insbes.
m ≥ 1. Dann sind n − 1, ν − 1 und n − 1, ν Paare natürlicher Zahlen
mit (n − 1) + (ν − 1) = m − 1 ≤ mund (n
− 1) +
ν = m ≤ m.
n−1
n−1
Nach Induktionsannahme sind daher
und
aus N.
ν
−
1
ν
n
n−1
n−1
Aus Regel (c) folgt:
=
+
∈ N.
ν
ν−1
ν
1.4 Der binomische Lehrsatz. Sei n ≥ 1 ganz. Dann gilt für alle a, b ∈ Z
n
(a+b) =
n X
n
ν=0
ν
a
n−ν ν
n
b = a +na
n−1
n n−ν ν
b+. . .+
a b +. . .+nabn−1 +bn
ν
Beweis. Durch Induktion nach n.
1
1
a+
b
Induktionsbeginn: (a + b) = a + b =
1
0
1
Induktionsannahme: 1.4 sei bewiesen für alle Exponenten 1 ≤ x ≤ n:
x
(a + b) =
x X
x
ν=0
ν
ax−ν bν für alle x ∈ N, x ≤ n
10
Induktionsschluß: (von n auf n + 1)
n+1
(a + b)
n
= (a + b)(a + b) = (a + b)
=
=
=
=
=
n X
n
ν=0
ν
an−ν bν =
!
n−1 X
n
n
(an+1 +
aan−ν bν ) +
an−ν bbν + bn+1
ν
ν
ν=1
ν=0
!
n
n X n
X
n
an+1 +
an+1−ν bν +
an−(ν−1) bν + bn+1
ν
ν−1
ν=1
ν=1
n
X n
n
an+1 +
+
a(n+1)−ν bν + bn+1
ν
ν−1
ν=1
n X
n + 1 (n+1)−ν ν
n+1
a
+
a
b + bn+1
ν
ν=1
n+1 X
n + 1 (n+1)−ν ν
a
b
ν
n X
ν=0
Dabei gilt die vorletzte Gleichung wegen 1.3c.
Anmerkung. Da die für a und b verwendeten Rechenregeln (1) bis (9) auch
für reelle Zahlen a und b gelten, ist der binomische Lehrsatz auch für a, b ∈ R
richtig.
11
§2 Teilbarkeit in Z
Bis auf weiteres stehen kleine Buchstaben für ganze Zahlen.
Teilbarkeit. Sei a 6= 0. Eine Zahl b heißt durch a teilbar, wenn es ein q
gibt mit b = qa. Wir sagen dann auch: a teilt b (ist ein Teiler von b) und b
ist ein Vielfaches von a.
Wir schreiben dafür: a | b.
Wenn a die Zahl b nicht teilt, schreiben wir: a - b.
Ist a | b und b = qa, so ist q = ab eindeutig durch das Paar a, b bestimmt.
Die trivialen Teiler von b sind ±b und ±1(b = 1 · b = b · 1 und b =
(−1)(−b) = (−b)(−1)).
2.1 Regel
(a) Aus a | b folgt a | −b und −a | b und −a | −b und |a| |b|.
(b) Aus a | b und b | c folgt a | c.
(c) Aus a | b und c | d folgt ac | bd (insbes.: a | b =⇒ ac | bc).
(d) Aus a | b und a | c folgt a | bx + cy für beliebige x, y.
(e) Aus ac | bc und c 6= 0 folgt a | b.
Beweis.
(a) b = qa =⇒ −b = (−q)a, b = (−q)(−a), −b = q(−a), |b| = |q| |a|
(b) b = qa, c = rb =⇒ c = r(qa) = (rq)a =⇒ a | c
(c) b = qa, d = rc =⇒ bd = (qa)(rc) = (qr)(ac) =⇒ ac|bd
(d) b = qa, c = ra =⇒ bx + cy = qax + ray = (qx + ry)a
1.2
(e) bc = q(ac), c 6= 0 =⇒ b = qa =⇒ a|b
Ist a 6= 0, so kann man b durch a immer mit Rest dividieren.
2.2 Division mit Rest. Sei a 6= 0 und b beliebig. Dann gibt es zu a, b genau
ein Zahlenpaar q, r mit
(∗)
b = qa + r
und 0 ≤ r < |a|
12
(a | b ⇐⇒ r = 0).
Man nennt q den unvollständigen Quotienten von b durch a, und
r den Divisionsrest (Rest bei der Division von b durch a).
Beweis. 1. Existenz. Es genügt, dies für a > 0 zu zeigen, denn: Wenn
a < 0, so ist −a > 0. Aus b = q̃(−a) + r mit 0 ≤ r < |a| = |− a| folgt:
b = qa + r, wobei q := −q̃.
Für u0 = −|b| ist b − u0 a = b + |b|a ≥ 0. Also ist die Menge M := {b − ua |
u ∈ Z und b − ua ≥ 0} ⊆ N nicht leer. Nach dem Prinzip vom kleinsten
Element existiert somit eine kleinste natürliche Zahl r der Form
r = b − qa , q ∈ Z.
Wegen der Minimalität von r ist r − a = b − (q + 1)a < 0, also r < a. Damit
ist, wie gefordert
b = qa + r und 0 ≤ r < a.
2. Eindeutigkeit. Sei b = qa + r = q 0 a + r0 mit 0 ≤ r < |a| und 0 ≤ r0 < a.
Dann ist (q − q 0 )a = r0 − r und |r0 − r| < |a|. Es folgt q − q 0 = 0, und
r0 − r = 0 · a = 0, also r0 = r.
Der größte gemeinsame Teiler von zwei Zahlen.
2.3 Bemerkung. Ist a 6= 0 und b | a, so ist |b| ≤ |a|. Insbesondere kommen
als Teiler von a nur die endlich vielen Zahlen ±1, ±2, . . . , ±a in Frage.
Beweis. b | a =⇒ a = qb, q 6= 0, da a 6= 0 =⇒ |q| ≥ 1 =⇒ |a| = |q| |b| ≥ |b|.
Nach dieser Bemerkung gibt es einen größten gemeinsamen Teiler von zwei
Zahlen a, b, welche nicht beide Null sind. Schreibe für den größten gemeinsamen Teiler (a, b) oder ggT (a, b). Mit anderen Worten:
Der größte gemeinsame Teiler (a, b) von a und b ist die eindeutig bestimmte natürliche Zahl d mit folgenden Eigenschaften:
(i) d | a und d | b
(ii) Gilt t | a und t | b, so ist t ≤ d.
Ist (a, b) = 1 so heißen a und b teilerfremd. In der Tat sind dann +1 und
−1 die einzigen gemeinsamen Teiler von a und b.
2.4 Satz. Seien a und b nicht beide 0 und d = (a, b). Dann gilt:
13
(a) d ist die kleinste positive Zahl der Form ax + by.
(b) Ist (a, b) = 1, so gibt es Zahlen x und y mit
ax + by = 1
(c) Ist t gemeinsamer Teiler von a und b, so ist t ein Teiler von d.
Beweis. M+ = {ax + by | x, y ganz und ax + by > 0} ist nicht leer, da
a2 + b2 ∈ M+ . Sei δ = MinM+ . Zeige zunächst:
(1) δ | a und δ | b
(2) t | a und t | b =⇒ t | δ
Sei δ = ax + by
Zu (1) Dividiere a durch δ mit Rest: a = qδ + r, 0 ≤ r < δ =⇒ r = a − qδ =
a − q(ax + by) = a(1 − qx) + b(−qy) = ax0 + by 0 .
Es folgt r = 0, da δ = MinM+ , und a = qδ, d.h. δ|a. Analog zeigt man, daß
δ | b.
2.1
Zu (2) t | a und t | b =⇒ t | ax + by = δ
Speziell gilt (2) für t = d =⇒ d|δ =⇒ d ≤ δ. Nach (1) ist δ gemeinsamer
Teiler von a und b, somit δ ≤ d. Es folgt d = δ, und (a) ist bewiesen.
(b) folgt aus (a).
Wegen (2) und δ = d gilt auch (c).
2.5 Korollar. M = {ax + by | x, y ∈ Z} ist die Menge der Vielfachen von
(a, b).
2.1
Beweis. (a, b) = d | a und d | b =⇒ d | ax + by, d.h. ax + by ist Vielfaches
von d.
Nach 2.4 ist d von der Form d = ax0 + by0 . Sei v = qd Vielfaches von
d =⇒ v = a(qx0 ) + b(qy0 ) ∈ M .
Das kleinste gemeinsame Vielfache von zwei Zahlen.
b heißt Vielfaches von a, wenn a | b.
Definition. Seien a > 0 und b > 0. Eine Zahl m heißt kleinstes gemeinsames Vielfaches von a und b, wenn m das kleinste unter den gemeinsamen
14
positiven Vielfachen von a und b ist (es gibt solche Vielfache, etwa ab). Schreibe dafür kgV (a, b).
2.6 Bemerkung. Seien a > 0 und b > 0. Dann gilt: Aus a|n und b|n folgt
kgV (a, b) | n. In Worten: Jedes gemeinsame Vielfache von a und b ist ein
Vielfaches von kgV (a, b).
Beweis. Sei m = kgV (a, b). Division mit Rest ergibt n = qm + r, 0 ≤ r <
2.1
m. =⇒ r = n − qm =⇒ a|r und b|r =⇒ r = 0 nach Definition von m.
2.7 Satz. Seien a > 0 und b > 0. Dann gilt
(a, b)kgV (a, b) = ab.
Beweis. Sei m = kgV (a, b). Aus a | ab und b | ab folgt nach 2.6: m | ab und
g = ab
ist ganz. Es ist zu zeigen, daß g = (a, b).
m
a = g · mb , b = g · m
mit m
, m ∈ Z, also gilt
a
a b
(1)
g | a und g | b.
2.6
Aus t | a und t | b folgt bt , at ∈ Z und a | a bt , b | b at =⇒ m |
ab
t
=⇒ t |
ab
m
=g
Also ist gezeigt:
(2)
Aus t | a und t | b folgt t | g, insbesondere t ≤ g.
Aus (1) und (2) ergibt sich: g = (a, b).
Nach Satz 2.7 können wir den Begriff kgV“ eigentlich wieder vergessen. Wir
”
notieren noch
2.8 Regeln für den größten gemeinsamen Teiler. Sei a 6= 0.
(a) 1 ≤ (a, b) ≤ Min(|a|, |b|) falls auch b 6= 0 (folgt aus 2.3)
(b) (a, 1) = 1 (folgt aus a))
(c) (a, 0) = |a|, (−a, b) = (a, b) = (b, a) (klar)
(d) Für c > 0 ist (ac, bc) = c · (a, b)
a
b
(e) ( (a,b)
, (a,b)
)=1
(f) (a, b + ax) = (a, b) für alle x
(g) b | a =⇒ (a, b) = |b|
15
(h) a | bc und (a, b) = 1 =⇒ a|c
Beweis.
2.4
d) d = (a, b) | a und d | b =⇒ dc | ac und dc | bc =⇒ dc | (ac, bc) =: δ
2.4
c | ac und c | bc =⇒ c | δ =⇒ δc ist ganz. Es folgt: δ | ac =⇒ δc | a und
δ | bc =⇒ δc | b
δ
c
2.4
| a und δc | b =⇒ δc | d =⇒ δ | dc
dc | δ und δ | dc =⇒ dc = δ
d)
a
b
a
b
e) (a, b) = ( (a,b)
· (a, b), (a,b)
· (a, b)) = (a, b)( (a,b)
, (a,b)
)
Kürzen ergibt die Behauptung.
2.1
2.1
f) t | a und t | b =⇒ t | a und t | b + ax =⇒ t | a und t | (b + ax) − ax = b.
Also haben die Paare a, b und a, b + ax die gleichen gemeinsamen Teiler
=⇒ (a, b) = (a, b + ax)
d)
b)
g) a = bq =⇒ (a, b) = (bq, b · 1) = |b|(q, 1) = |b|.
c)
h) c = 0 =⇒ a | c. c 6= 0, a | ac, a | bc =⇒ a | (ac, bc) = |c|(a, b) = |c| =⇒
a | c.
2.9 Der euklidische Algorithmus zur Bestimmung des größten gemeinsamen Teilers von a und b.
Nach 2.8 können wir annehmen, daß a > b > 0. Man erhält (a, b) nach dem
folgenden Verfahren: Setze a0 := a und a1 := b.
1. Schritt. Dividiere a0 durch a1 mit Rest:
a0 = q0 · a1 + a2 mit 0 ≤ a2 < a1
2.8
Bleibt kein Rest, so ist a1 | a0 =⇒ (a, b) = (a0 , a1 ) = a1 = b. Sonst gilt
0 < a2 < a 1 < a 0 .
2. Schritt. Dividiere a1 durch a2 mit Rest:
a1 = q1 a2 + a3 , 0 ≤ a3 < a2
Solange ein Rest bleibt fährt man fort und kommt zum k–ten Schritt. Es ist
0 < ak < ak−1 < . . . < a1 < a0 .
16
k–ter Schritt. Dividiere ak−1 durch ak mit Rest:
ak−1 = qk−1 ak + ak+1 , 0 ≤ ak+1 < ak
Wegen 0 ≤ ak+1 < ak < . . . < a1 < a0 = a muß das Verfahren abbrechen
(und zwar nach höchstens a Schritten), d.h.: Es gibt eine Zahl n ≥ 1, so daß
(i) ak−1 = qk−1 ak + ak+1 , 0 < ak−1 < ak für 1 ≤ k ≤ n − 1
(ii) an−1 = qn−1 an (also an |an−1 und daher (an , an−1 ) = an ).
Nach Regel f) gilt: (ak , ak−1 ) = (ak+1 + qk−1 ak , ak ) = (ak+1 , ak ) für 1 ≤ k ≤
n − 1. Also ist
(a, b) = (a1 , a0 ) = (a2 , a1 ) = . . . = (an , an−1 ) = an
Fazit.
(1) Ist b|a, so ist (a, b) = b.
(2) Ist b - a, so ist (a, b) der letzte Divisionsrest, der beim euklidischen
Algorithmus auftritt.
Rechenbeispiel. a = 531, b = 93 (siehe § 2)
531
93
66
27
12
=
=
=
=
=
5 · 93 + 66
1 · 66 + 27
2 · 27 + 12
2 · 12 + 3
4·3
letzter Divisionsrest
Also ist (531, 93) = 3.
Sind a > 0 und b > 0 teilerfremd, so gibt es nach 2.4 b) Zahlen x und y, so
daß ax + by = 1 ist. Mit Hilfe des euklidischen Algorithmus kann man solche
x, y leicht berechnen.
Verfahren zur Lösung der Gleichung aX + bY = 1, wenn (a, b) = 1
ist.
17
1. Schritt. Führe den euklidischen Algorithmus für a, b durch (o.E. a > b).
Erhalte Gleichungen (a0 = a, a1 = b).
a0 = q0 a1 + a2
a1 = q1 a2 + a3
..
.
ak−2 = qk−2 ak−1 + ak
..
.
an−2 = qn−2 an−1 + an
an−1 = qn−1 an
an = (a, b)
Im Falle (a, b) = 1 ist dabei an = 1, qn−1 = an−1 .
2. Schritt. Bestimme rekursiv von unten nach oben für k = n, n − 1, . . . z.
Zahlen xk , yk , so daß
(∗)
xk ak−2 + yk ak−1 = 1
Beginn der Rekursion. k = n : 1 · an−2 + (−qn−2 )an−1 = 1. Im Fall
k = n = 2 ist man fertig. Sei nun n ≥ k ≥ 3 und seien xk , yk mit der
Eigenschaft (∗) schon bestimmt. Setze die Gleichung aus dem Euklidischen
Algorithmus ak−1 = ak−3 − qk−3 ak−2 in (∗) ein und erhalte
1 = xk ak−2 + yk (ak−3 − qk−3 ak−2 )
= (xk − yk qk−3 )ak−2 + yk ak−3 = xk−1 ak−3 + yk−1 ak−2
Am Ende erhält man für k = 2
1 = x2 a0 + y2 a1 = x2 a + y2 b
Rechenbeispiel. Zeige, daß (97, 44) = 1 und
97 = 2 · 44 + 9 9 =
44 = 4 · 9 + 8 8 =


9 = 1·8+1 
y 1 =
löse 97x + 44y = 1
97 − 2 · 44
44 − 4 · 9
9−1·8
Aus 1 = 9 − 1 · 8 und 8 = 44 − 4 · 9 folgt 1 = −44 + 5 · 9. Aus 1 = −44 + 5 · 9
und 9 = 97 − 2 · 44 folgt 1 = 5 · 97 − 11 · 44.
Fazit: x = 5, y = −11 ist eine Lösung der Gleichung 97x + 44y = 1.
18
§3 Primzahlen
Die Zahl 1 hat nur einen positiven Teiler, nämlich 1. Jede Zahl a > 1 hat
mindestens zwei positive Teiler: 1 und a.
Definition. Eine Primzahl ist eine Zahl a > 1, welche nur die Teiler 1 und
a hat.
Beispiele. 2, 3, 5, 7, 11 sind Primzahlen.
Im Folgenden ist der Buchstabe p den Primzahlen vorbehalten; ebenso bedeuten p1 , p2 , . . . oder p0 , p0j , p00j , . . . stets Primzahlen.
3.1 Satz. Jedes a > 1 ist als Produkt von Primzahlen darstellbar (Primfaktorzerlegung von a):
a = p1 · p2 · . . . · pr =
r
Y
pn , r ≥ 1
n=1
Beweis. Für a = p ist die Aussage offenbar wahr. Wir beweisen 3.1 durch
vollständige Induktion nach a.
Induktionsbeginn. a = 2 ist eine Primzahl.
Induktionsannahme. Sei a ≥ 3 und 3.1 bereits bewiesen für alle b mit
1 < b < a.
Induktionsschluß. Ist a Primzahl, so ist 3.1 richtig für a. Sonst gibt es eine
Zerlegung a = a1 a2 mit 1 < a1 < a und 1 < a2 < a.
Nach Induktionsannahme haben a1 und a2 eine Primfaktorzerlegung; also
gilt dies auch für a = a1 a2 .
Frage: Wieviele“ Primzahlen gibt es?
”
3.2 Satz. (Euklid) Es gibt unendlich viele Primzahlen.
Beweis. Es ist zu zeigen: Zu jeder endlichen Menge von Primzahlen kann
man eine weitere Primzahl finden. Seien also r ≥ 1 paarweise verschiedene
Primzahlen p1 . . . pr vorgegeben.
Setze
a := 1 + p1 · . . . · pr
Dann ist a > 1 und p1 , . . . , pr sind keine Teiler von a (denn sonst wäre etwa
pi ein Teiler von 1 = a − p1 · . . . · pr , Widerspruch.) Nach 3.1 ist aber a durch
19
wenigstens eine Primzahl p teilbar. Diese kommt in der Menge {p1 , . . . , pr }
nicht vor.
3.3 Regel.
(a) Aus p - a folgt (p, a) = 1
(b) Aus p | ab folgt: p | a oder p | b.
(c) Für q > 1 gelte: Aus q | ab folgt q | a oder q | b. Dann ist q eine
Primzahl.
(d) Aus p |
r
Q
an folgt: p | an für mindestens ein n.
n=1
(e) Aus p |
r
Q
pn folgt: p = pn für mindestens ein n.
n=1
Beweis.
(a) p hat nur die positiven Teiler 1 und p und p - a. Es folgt (p, a) = 1.
(a)
(p, a) = 1
2.8
=⇒ p | b.
(b) p - a =⇒
p | ab
(c) Ist q > 1 keine Primzahl, so schreibt sich q nach 3.1 in der Form q = p·r,
p Primzahl, r ≥ 2. Also ist q | q = pr und q > p, q > r. Es folgt q - p
und q - r.
(d) folgt aus (b) durch Induktion.
(e) p |
r
Q
(d)
pn =⇒ p | pn für ein n =⇒ p = pn , da p 6= 1 und 1 und pn die
n=1
einzigen positiven Teiler von pn sind.
3.4 Bemerkung. Wegen Regel 3.3(b) und (c) hätte man Primzahl“ auch
”
so definieren können: Eine Zahl p > 1 heißt Primzahl, wenn gilt:
Aus p | ab folgt: p | a oder p | b.
3.5 Satz. Die Zerlegung jeder Zahl a > 1 in ein Produkt von Primzahlen ist
(bis auf die Reihenfolge der Faktoren) eindeutig.
20
Beweis. Es genügt zu zeigen:
r
r0
Q
Q
Aus a =
pn =
p0n mit p1 ≤ p2 ≤ . . . ≤ pr und p01 ≤ p02 ≤ . . . ≤
p0r0 folgt:
n=1
n=1
r = r0 und pn = p0n für alle n,
1 ≤ n ≤ r.
Beweis durch Induktion nach a.
Induktionsbeginn. Für a = 2 muß offenbar r = r0 = 1 und p1 = p01 = 2
sein.
Induktionsannahme. Sei a > 2 und die Behauptung bereits bewiesen für
2, 3, . . . , a − 1.
Induktionsschluß. Ist a eine Primzahl, so ist r = r0 = 1 und p1 = p01 = a,
denn a hat keine echten Teiler (dies sind die von ±1 und ±a verschiedenen
Teiler).
Andernfalls sind r > 1 und r0 > 1 und
p01
r
r0
Y
Y 0
p
,
p
pn .
n
1 n=1
n=1
Nach 3.3(e) gibt es dann n, m mit p01 = pn und p1 = p0m . Wegen p1 ≤ pn =
p01 ≤ p0m = p1 folgt p1 = p01 . Wegen 1 < p1 < a folgt
0
r
r
Y
Y
a
=
pn =
p0n =: a0 < a, also 1 < a0 < a
1<
p1 n=2
n=2
Wende die Induktionsannahme an auf a0 und erhalte r = r0 und pn = p0n für
2 ≤ n ≤ r.
p1 = p01 wurde bereits gezeigt. Damit ist alles bewiesen.
Man kann in der Primfaktorzerlegung noch gleiche Faktoren zusammenfassen
und erhält:
3.6 Korollar. Jede Zahl a > 1 besitzt genau eine Zerlegung
m2
mr
1
a = pm
1 · p2 · . . . · pr , p1 < p2 < . . . pr ; mρ ≥ 1 für ρ = 1, . . . , r.
(Wir sprechen auch von der kanonischen Zerlegung von a.)
21
3.7 Bemerkung. Sei a =
r
Q
n=1
n
pm
n die kanonische Zerlegung von a > 1. Dann
sind die gesamten positiven Teiler von a die Zahlen
r
Y
plnn , wobei 0 ≤ ln ≤ mn für 1 ≤ n ≤ r.
n=1
Insbesondere besitzt a genau
r
Q
(mn + 1) verschiedene positive Teiler.
n=1
Beweis. Offenbar sind die angegebenen Zahlen Teiler von a; sie sind nach 3.6
paarweise verschieden; also stimmt die Anzahlaussage, falls a keine weiteren
positiven Teiler hat. Ist nun d | a, also a = dq, so gehen in d und in q nur
Primzahlen auf, die auch in a aufgehen. Also gilt
d=
r
Y
plnn ,
q=
n=1
r
Y
pknn
=⇒
n=1
r
Y
plnn +kn
= dq = a =
n=1
r
Y
n
pm
n .
n=1
Wegen der Eindeutigkeit der Zerlegung folgt mn = ln + kn . Also ist
0 ≤ ln = mn − kn ≤ mn für n = 1, . . . r.
Wenn die Primfaktorzerlegungen von a ≥ 1, b ≥ 1 schon vorliegen, so läßt
sich (a, b) leicht bestimmen, ohne den euklidischen Algorithmus zu bemühen.
3.8 Satz. Seien p1 , . . . pr die verschiedenen Primteiler von ab, a > 1 und
b > 1. Dann kommen auch in den Zerlegungen von a bzw. b höchstens die
Primzahlen p1 , . . . , pr vor.
Schreibe:
mr
1
a = pl11 · . . . · plrr , b = pm
1 · . . . · pr , ln ≥ 0 , mn ≥ 0.
Dann gilt:
Min(l1 ,m1 )
(a) (a, b) = p1
Max(l1 ,m1 )
(b) kgV (a, b) = p1
Min(lr ,mr )
· . . . · pr
Max(lr ,mr )
· . . . · pr
Beweis.
22
(a) Nach dem Beweis von 3.7 sind die positiven Teiler von a bzw. b die
Zahlen
r
Y
n=1
r
Y
pknn , 0 ≤ kn ≤ ln
pknn , 0 ≤ kn ≤ mn
für alle 1 ≤ n ≤ r,
bzw.
für alle 1 ≤ n ≤ r.
n=1
Die gemeinsamen positiven Teiler von a und b sind also die Zahlen
r
Y
pknn mit 0 ≤ kn ≤ Min(ln , mn ) für alle 1 ≤ n ≤ r
n=1
Die größte dieser Zahlen ist offenbar
r
Q
Min(ln ,mn )
pn
.
n=1
(b) folgt aus (a) und der Formel kgV (a, b) · (a, b) = ab, denn Min(ln , mn ) +
Max(ln , mn ) = ln + mn .
Der größte gemeinsame Teiler von mehr als zwei Zahlen.
Bezeichnung. Sind die Zahlen a1 , . . . , ar (r ≥ 1) nicht alle 0, so wird ihr
größter gemeinsamer Teiler mit (a1 , . . . , an ) bezeichnet. δ = (a1 , . . . , ar ) ist
also die größte ganze Zahl mit δ | a1 , . . . , δ | ar−1 und δ | ar .
3.9 Satz. Seien a1 > 0, . . . , ar > 0, r ≥ 2. Dann gilt
(a) (a1 , . . . , ar ) = ((a1 , . . . , ar−1 ), ar )
(b) Jeder gemeinsame Teiler von a1 , . . . , ar teilt (a1 , . . . ar ).
Beweis. (Induktion nach r). Für r = 2 ist (a) trivial und (b) gilt nach 2.4.
Induktionsannahme. Sei r ≥ 3, (a) und (b) bewiesen für alle k mit 2 ≤
k ≤ r − 1.
Induktionsschluß. Ist t gemeinsamer Teiler von a1 , . . . , ar , so auch von
a1 , . . . , ar−1 . Nach Induktionsannahme (b) ist daher t ein Teiler von
(a1 , . . . , ar−1 ) = a0 . Ferner gilt t | ar . Nach (2.4) ist daher t | (a0 , ar ) =
((a1 , . . . , ar−1 ), ar ). Setze δ := ((a1 , . . . , ar−1 ), ar ). Wegen t | δ ist t ≤ δ.
Ferner gilt: δ | (a1 , . . . , ar−1 ) und δ | ar und daher δ | a1 , . . . , δ | ar−1 und
23
δ | ar , d.h.: δ ist gemeinsamer Teiler von a1 , . . . , ar .
Damit ist gezeigt, daß δ der größte gemeinsame Teiler von a1 , . . . , ar ist, und
(a) ist bewiesen.
Im Beweis haben wir gesehen, daß jeder gemeinsame Teiler t von a1 , . . . , ar
auch δ teilt. Damit ist auch (b) bewiesen.
3.10 Korollar. Unter den Voraussetzungen von 3.9 ist (a1 , . . . , ar ) die kleinste positive Zahl, welche sich in der Form schreibt
a1 x1 + . . . + ar xr
mit x1 , . . . , xr ∈ Z.
Beweis. (Induktion nach r.) Für r = 2 wurde dies in 2.4 gezeigt.
Nach Induktionsannahme ist δ 0 := (a1 , . . . , ar−1 ) die kleinste positive Zahl
der Form δ 0 = a1 y1 + . . . + ar−1 yr−1 . Ferner ist nach Induktionsbeginn δ =
(a1 , . . . , ar ) = ((a1 , . . . , ar−1 ), ar ) = (δ 0 , ar ) von der Form δ = δ 0 x + ar xr .
Es folgt: δ = a1 (y1 x) + . . . + ar−1 (yr−1 x) + ar xr ist von der gewünschten
Gestalt.
Ist d = a1 x1 + . . . + ar xr > 0 mit x1 , . . . , xr ∈ Z, so ist wegen δ | a1 , . . . , δ | ar
auch δ | d , also δ ≤ d.
Mit Hilfe von 3.9 kann man auch (a1 , . . . , ar ) für r ≥ 3 (iterativ) mit Hilfe
des euklidischen Algorithmus bestimmen.
(a1 , a2 , a3 ) = ((a1 , a2 ), a3 )
(a1 , a2 , a3 , a4 ) = (((a1 , a2 ), a3 ), a4 ) usw.
Wir erwähnen noch ohne Beweis:
3.10 Korollar. Seien p1 , . . . , ps die verschiedenen Primteiler des Produkts
a1 · . . . · ar von positiven Zahlen a1 , . . . , ar und
l
an = p11,n · . . . · plss,n
,
lm,n ≥ 0 für 1 ≤ m ≤ s, 1 ≤ n ≤ r.
Setze lm := Min(lm,1 , . . . , lm,r ), 1 ≤ m ≤ s. Dann gilt
(a1 , . . . , ar ) = pl11 · . . . · plss .
24
§4 Vollkommene Zahlen
Sei a > 0
T (a) bezeichnet die Anzahl der positiven Teiler von a.
S(a) bezeichnet die Summe der positiven Teiler von a.
Es ist also T (1) = S(1) = 1.
Jede Zahl a > 1 hat eine eindeutige Darstellung
Y
a=
plp , wobei lp ≥ 1, falls p | a.
p|a
Im §3 haben wir gesehen: Die positiven Teiler von a sind die Zahlen der Form
Y
(∗)
pmp
,
wobei 0 ≤ mp ≤ lp
für p | a.
p|a
Insbesondere gilt deshalb und wegen der Eindeutigkeit der Primfaktorzerlegung für a > 1
Y
(lp + 1).
T (a) =
p|a
Es folgt: T (ab) = T (a)T (b) falls a, b ≥ 1 und (a, b) = 1.
4.1 Satz.
S(a) =
Y plp +1 − 1
p|a
p−1
Insbesondere: S(ab) = S(a)S(b), wenn (a, b) = 1.
Beweis. Nach der Formel für eine endliche geometrische Reihe gilt (vgl. §1):
lp
X
pmp =
mp =0
plp +1 − 1
p−1
Wegen (∗) folgt

Yp
p|a
lp

X Y
− 1 Y  X mp 
(∗) X
=
p
=
pmp =
t = S(a)
p−1
m =0
0≤m ≤l
lp +1
p|a
p
p
p|a
25
p
p|a
t|a
t>0
Dabei gilt das zweite Gleichheitszeichen nach dem Distributivgesetz.
Eine Zahl a heißt gerade, falls 2 | a und ungerade, falls 2 - a.
Definition. Eine Zahl a > 1 heißt vollkommen, wenn a mit der Summe
seiner Teiler t 6= a übereinstimmt, wenn also S(a) = 2a.
Beispiele. 6 = 1 + 2 + 3 und 28 = 1 + 2 + 4 + 7 + 14 sind vollkommen. Sie
sind die einzigen vollkommenen Zahlen ≤ 30. (Man prüfe das nach.)
Frage 1. Wieviele vollkommene Zahlen gibt es?
Es ist 6 = 3·4
und 28 = 7·8
mit Primzahlen 3 = 22 − 1 und 7 = 23 − 1. Beide
2
2
vollkommenen Zahlen ≤ 30 sind also gerade und von der Form
a=
p(p + 1)
mit einer Primzahl p = 2n − 1,
2
n ∈ N.
Wie der nächste Satz zeigt, ist dies kein Zufall.
eine vollkommene
4.2 Satz. Ist p = 2n − 1, n ∈ N eine Primzahl, so ist p(p+1)
2
gerade Zahl. Sonst gibt es keine vollkommenen geraden Zahlen.
n
n
Beweis. a = p(p+1)
= (2 −1)2
= 2n−1 · (2n − 1) = 2n−1 · p ist die kanonische
2
2
Zerlegung von a. Die verschiedenen Primteiler von a sind 2 und p. Nach 4.1
ist daher
S(a) =
2n − 1 p2 − 1
·
= (2n − 1)(p + 1) = p(p + 1) = 2a
2−1 p−1
Sei umgekehrt a gerade und vollkommen. Dann schreibt a in der Form a =
2n−1 · u, wobei n > 1 und u ungerade ist. Nach 4.1 gilt
2n u = 2a = S(a) =
Es folgt: S(u) =
2n ·n
2n −1
u
2n −1
n
=u+
2n − 1
· S(u) = (2n − 1)S(u)
2−1
u
.
2n −1
Insbesondere ist
= S(u) − u ganz, also ist t0 = 2nu−1 ein Teiler von u.
Wegen n > 1 ist 2 − 1 > 1 und t0 ist ein von u verschiedener Teiler von u.
X
X
S(u) = u + t0 =
t=u+
t
t|u
Also hat u nur zwei Teiler, nämlich u und t0 .
26
t|u
t6=u
Es folgt: u ist eine Primzahl und t0 = 1. =⇒ u = 2n − 1, u ist eine Primzahl
u = (u+1)u
.
und a = 2n−1 u = (u+1)
2
2
Definition. Die Primzahlen der Form p = 2n − 1 heißen Mersennesche
Primzahlen.
Gäbe es unendlich viele Mersennesche Primzahlen, so gäbe es auch unendlich
viele vollkommene Zahlen.
Frage 2: Gibt es unendlich viele Mersennesche Primzahlen?
Diese Frage kann bis heute nicht beantwortet werden.
22 − 1 = 3 und 23 − 1 = 7 sind Primzahlen, 24 − 1 = 15 ist keine.
25 − 1 = 31; 26 − 1 = 63 keine Primzahl, 27 − 1 = 127 Primzahl.
Man könnte vermuten: 2n − 1 ist Primzahl ⇐⇒ n ist Primzahl. Dies ist
falsch: 211 − 1 = 2047 = 23 · 89 ist zusammengesetzt.
Beispiel einer sehr großen Mersenneschen Primzahl:
2132049 − 1 ist eine Primzahl mit 39751 Stellen.
Die eine Richtung der obigen Vermutung ist jedoch richtig:
4.3 Satz. Ist n keine Primzahl, so ist auch 2n − 1 keine Primzahl.
Beweis. Schreibe n = uv mit u > 1 und v > 1. Dann ist
2n − 1 = 2uv − 1 = (2u )v − 1
Nach der Formel für die geometrische Reihe (x = 2u ) gilt:
1 + 2u + (2u )2 + . . . + (2u )v−1 =
(2u )v − 1
, also ist
2u − 1
2n − 1 = (2u − 1)(1 + 2u + (2u )2 + . . . + (2u )v−1 ).
Wegen u > 1 und v > 1 ist 2u − 1 > 1 und 1 + 2u + . . . + (2u )v−1 > 1, und
2n − 1 ist zusammengesetzt.
Anmerkung. Man kennt bis heute nur endlich viele Mersennesche Primzahlen, also auch nur endlich viele gerade vollkommene Zahlen. Ungerade
vollkommene Zahlen sind überhaupt keine bekannt. Man weiß auch nicht, ob
es welche gibt oder nicht.
27
Multiplikative zahlentheoretische Funktionen
Definition. Sei f : N\{0} −→ C (eine zahlentheoretische Funktion“.) Man
”
nennt f multiplikativ, wenn
f (ab) = f (a)f (b), falls (a, b) = 1
Beispiele.
(a) Die konstante Funktion f (n) = 1 ist offenbar multiplikativ.
(b) Die Identität f (n) = n ist offenbar multiplikativ.
(c) Wie oben gezeigt wurde, sind S und T multiplikativ.
Die Aussage (c) folgt auch leicht aus dem folgenden allgemeinen Satz.
4.4 Satz. Ist f (n) eine
P multiplikative zahlentheoretische Funktion, so gilt
dies auch für F (n) =
f (d).
d|n
d>0
4.5 Korollar. S und T sind multiplikativ.
Beweis.
1. Für T wende
P man 4.4 an auf f (n) = 1:
F (n) =
f (d) = T (n) wegen f (n) ≡ 1. Aus f (n) multiplikativ
d|n,d>0
folgt F (n) multiplikativ.
2. Für S wende
P man Satz
P 4.4 an auf f (n) = n:
F (n) =
f (d) =
d = S(n). Schließe wie in 1.
d|n
d>0
d|n
d>0
Beweis von Satz 4.4 Für die Nullfunktion f (n) ≡ 0 ist nichts zu zeigen.
Ist f 6= 0 und speziell n ∈ N mit f (n) 6= 0, so gilt
f (n) = f (n · 1) = f (n) · f (1).
Es folgt f (1) = 1 nach der Kürzungsregel.
Ist nun m = 1 oder n = 1, so ist F (n) = f (1) = 1 oder F (m) = f (1) = 1.
In jedem Fall ist F (m · n) = F (m) · F (n). Sei also m > 1 und n > 1. Dann
schreibt man im Fall (m, n) = 1
28
α
α
r+1
r+s
· . . . · pr+s
n = pα1 1 · pα2 2 · . . . · pαr r , m = pr+1
mit paarweise verschiedenen Primzahlen p1 , . . . , pr+s und αi ≥ 1 für
i = 1, . . . , r + s.
Die Teiler von n sind pβ1 1 · . . . · pβr r , 0 ≤ βi ≤ αi , i = 1, . . . , r.
βr+1
βr+s
, 0 ≤ βi ≤ αi , i = r + 1, . . . , r + s.
· . . . · pr+s
Die Teiler von m sind pr+1
Durchläuft nun d1 die Teiler von n und d2 die Teiler von m, so durchläuft
d1 d2 die Zahlen
β
β
r+1
r+s
d = d1 d2 = pβ1 1 . . . pβr r pr+1
· . . . · pr+s
,
0 ≤ βi ≤ αi , i = 1, . . . , r + s.
α
r+s
Dies sind gerade die Teiler von mn = pα−1
· . . . · pr+s
.
1
Mit anderen Worten
X
f (d1 d2 ) =
d1 |n,d2 |m
d1 >0,d2 >0
X
f (d) = F (nm)
d|nm
d>0
Für d1 | n und d2 | m gilt wegen (n, m) = 1 auch (d1 , d2 ) = 1. Nach Voraussetzung ist daher f (d1 d2 ) = f (d1 )f (d2 ) und



P
f (d1 d2 )
d1 |n,d2 |m
d1 >0,d2 >0
=
P
f (d1 )f (d2 )
d1 |n,d2 |m
d1 >0,d2 >0
=
P
 P

f (d1 ) 
f (d2 )

d1 |n
d1 >0
d2 |m
d2 >0
= F (n) · F (m). Damit ist F (nm) = F (n)F (m) gezeigt.
Anmerkung. Von ähnlicher Bauart wie die Mersenneschen Primzahlen sind
die Fermatschen Primzahlen: Eine Primzahl der Form p = 2s + 1 heißt
Fermatsche Primzahl.
Beispiele. 2 = 20 + 1, 3 = 21 + 1, 5 = 22 + 1, 17 = 24 + 1, 257 = 28 + 1
sind Primzahlen, aber 9 = 23 + 1, 33 = 25 + 1, 65 = 26 + 1, 129 = 27 + 1
sind keine.
Allgemein gilt
4.6 Satz. Ist s > 0 keine Potenz von 2, so ist 2s + 1 keine Primzahl.
Beweis. Schreibe s = 2t · v mit t ≥ 0, v > 1 ungerade und setze k := 2t .
Die Formel für die geometrische Reihe für x = −2k ergibt
1 − xv
= 1 + x + x2 + . . . + xv−1 = 1 − 2k + 22k − + . . . + 2k(v−1) ∈ Z
1−x
29
und
1 − xv
1 − (−2k )v
1 + 2kv
1 + 2s
=
=
=
.
1−x
1 − (−2k )
1 + 2k
1 + 2k
Also gilt 1 + 2k | 1 + 2s .
Zeige noch, daß 1+2k 6= 1 und 1+2k 6= 1+2s . Dann ist 1+2s keine Primzahl.
Aus v > 1, k ≥ 1 folgt 1 < 1 + 2k < 1 + (2k )v = 1 + 2kv = 1 + 2s .
t
Fazit. Als Fermatsche Primzahlen kommen nur die Zahlen 2(2 ) + 1, t =
0, 1, 2, . . . in Frage.
Stand der Wissenschaft: 3, 5, 17, 257, 216 + 1 = 65537 sind die einzigen
bekannten Fermatschen Primzahlen. (232 + 1 ist zum Beispiel durch 641 teilbar.)
Ein ungelöstes Problem: Gibt es unendlich viele Fermatsche Primzahlen?
30
§5 Kongruenzrechnung
Sei m > 0 fest vorgegeben. Nach §2 wissen wir: Jede Zahl a läßt sich auf
eindeutige Weise durch m mit Rest dividieren, d.h.: Es gibt genau ein Zahlenpaar q, r mit der Eigenschaft
(∗)
a = qm + r , 0 ≤ r < m
Definition. Zwei Zahlen heißen kongruent modulo m, wenn sie bei der
Division durch m den gleichen Rest lassen.
Schreibe für a ist kongruent zu b modulo m“ kurz
”
a ≡ b mod m.
Wenn klar ist, welches m gemeint ist auch: a ≡ b.
a 6≡ b mod m bedeutet, daß a und b nicht kongruent modulo m (oder in”
kongruent modulo m“) sind.
Offenbar gilt
5.1 Bemerkung. Kongruenz modulo m“ ist eine Äquivalenzrelation, d.h.
”
(a) a ≡ a mod m (Reflexivität)
(b) a ≡ b mod m =⇒ b ≡ a mod m (Symmetrie)
(c) a ≡ b mod m und b ≡ c mod m =⇒ a ≡ c mod m (Transitivität)
5.2 Bemerkung. Genau dann ist a ≡ b mod m, wenn m ein Teiler von a − b
ist.
Beweis. Sei a ≡ qm + r, 0 ≤ r < m.
a ≡ b mod m =⇒ b = q 0 m + r =⇒ a − b = (q − q 0 )m =⇒ m | a − b.
m | a − b =⇒ a − b = v · m =⇒ b = a − vm = (q − v)m + r =⇒ a ≡ b mod m.
Die möglichen Divisionsreste modulo m sind die m Zahlen 0, 1, . . . , m − 1.
Wir können also die ganzen Zahlen nach ihren Divisionsresten in m Klassen
einteilen:
Definition. Die Restklasse von a modulo m besteht aus allen Zahlen,
welche modulo m den gleichen Divisionsrest haben wie a. Demnach gibt es
31
genau m verschiedene Restklassen modulo m und verschiedene Klassen haben
keine Elemente gemeinsam.
Rest 0 haben
Rest 1 haben
..
.
: 0, ±m, ±2m, ±3m, . . .
: 1, 1 ± m, 1 ± 2m, 1 ± 3m, . . .
Rest r haben
..
.
: r, r ± m, r ± 2m, r ± 3m, . . .
Rest m − 1 haben
: m − 1, m − 1 ± m, m − 1 ± 2m, m − 1 ± 3m, . . .
(für 0 ≤ r < m)
Zahlenbeispiele. m = 2
Rest 0 mod 2 : 0, ±2, ±4, . . .
haben die geraden Zahlen
Rest 1 mod 2 : 1, 1 ± 2, 1 ± 4, . . . haben die ungeraden Zahlen.
m = 7 : Identifiziere Z mit den Tagen von Ewigkeit zu Ewigkeit“, genauer
”
von −∞ bis +∞, wobei 0 mit dem Tag der Geburt Christi gleichgesetzt wird.
n>0:
n = n–ter Tag nach Christi Geburt.
− n = n–ter Tag vor Christi Geburt.
Wir nehmen an: 0 war ein Sonntag (so genau weiß das niemand.)

−7, −14, −21, . . .

0
Restklasse von 0 mod 7: Sonntage

+7, +14, +21, . . .
1,
8,
15,
22, . . .
Restklasse von 1 mod 7: Montage
−6, −13, −20, −27, . . .
..
.
6, 13,
20, . . .
Restklasse von 6 mod 7: Samstage
−1, −8, −15, . . .
Definitionsgemäß gehören a und b zur gleichen Restklasse modulo m wenn
a ≡ b mod m.
Man spricht daher auch von Kongruenzklassen modulo m anstelle von
Restklassen modulo m.
Nach 5.2 gilt: a ≡ b mod m ⇐⇒ m | a − b, d.h.
Es gibt ein q ∈ Z mit b = a + qm.
Damit gilt
5.3 Bemerkung. Die Restklasse von a modulo m ist die Menge
{a + mq | q ∈ Z}
32
Schreibe dafür auch a + mZ oder a + (m).
Wir haben gesehen:
(1) Es gibt genau m verschiedene Restklassen modulo m.
(2) Jede Zahl gehört zu genau einer Restklasse modulo m.
Die Aussagen (1) und (2) zusammengenommen kann man auch so ausdrücken:
Die Menge aller ganzen Zahlen ist die disjunkte Vereinigung der verschiedenen Restklassen modulo m.
Definition. Eine Menge von Zahlen a1 , . . . , am heißt vollständiges Repräsentantensystem (Restsystem) modulo m, wenn a1 +(m), a2 +(m), . . . ,
am + (m) gerade die m verschiedenen Restklassen modulo m sind, d.h. wenn
ai 6≡ aj mod m , falls i 6= j.
Beispiel. 0, 1, 2, . . . , m−1 ist ein vollständiges Repräsentantensystem modulo m. (Diese Zahlen sind selbst Divisionsreste und voneinander verschieden.)
5.4 Satz.
(a) Ist a1 , . . . , am ein vollständiges Restsystem modulo m, so gilt dies auch
für a1 + c, . . . , am + c(c ∈ Z). Insbesondere gilt nach dem Beispiel:
Je m aufeinander folgende Zahlen bilden ein vollständiges Restsystem
modulo m. Ein solches System mit – dem Betrag nach – möglichst
kleinen Elementen ist die Menge der ganzen Zahlen größer als − m2 und
kleiner oder gleich + m2 .
Für ungerades m sind dies die Zahlen
− m−1
, − m−1
+ 1, . . . , −1, 0, 1, . . . ,
2
2
m−1
2
und für gerades m die Zahlen
− m2 + 1, − m2 + 2, . . . , −1, 0, 1, . . . ,
m = 7 : −3, −2, −1, 0, 1, 2, 3
m = 8 : −3, −2, −1, 0, 1, 2, 3, 4
m
2
(b) Ist a1 , . . . , am ein vollständiges Restsystem modulo m und ist (k, m) =
1, so ist auch a1 k, a2 k, . . . , am k ein vollständiges Restsystem modulo
m.
33
Beweis.
5.2
(a) Für i 6= j ist ai 6≡ aj mod m, also m - (ai − aj ) = (ai + c) − (aj + c) =⇒
(ai + c) 6≡ (aj + c) mod m.
(b) ai k ≡ aj k mod m =⇒ m | (ai k − aj k) = (ai − aj )k, also m | (ai − aj )k
und (m, k) = 1. Nach 2.8 h gilt daher m | (ai − aj ) d.h. ai ≡ aj mod
m. Nach Voraussetzung ist dann i = j.
Im folgenden sei m > 0 fest vorgegeben, und a ≡ b“ bedeute immer a ≡ b
”
mod m. Es soll gezeigt werden, daß man mit ≡“ wie mit einem Gleich”
heitszeichen umgehen darf.
5.5 Satz.
(a) a ≡ b =⇒ a ± c ≡ b ± c
(b) a ≡ b =⇒ ac ≡ bc
(c) a ≡ b =⇒ an ≡ bn für alle n ∈ N.
(d) Ist f (x) = c0 + c1 x + . . . + cn xn eine Polynomfunktion in der Variablen
x, so folgt aus a ≡ b schon f (a) ≡ f (b).
Beweis.
+
+
+
+
(a) a ≡ b =⇒ m | (a − b) = (a (−) c) − (b (−) c) =⇒ a (−) c ≡ b (−) c
(b) a ≡ b =⇒ m | a − b =⇒ m | (a − b)c = ac − bc =⇒ ac ≡ bc
(c) (Induktion nach n) n = 0 : a0 = 1 ≡ 1 = b0
Induktionsannahme. Sei n ≥ 1 und an−1 ≡ bn−1 schon bewiesen.
(b)
Mit (b) folgt an = an−1 a ≡ an−1 b ≡ bn−1 b = bn .
(d) Sei a ≡ b. Nach (b) und (c) gilt: cν aν ≡ cν bν , ν = 0, . . . , n
Induktion nach n. n = 0 : f (x) = c0 für alle x =⇒ f (a) = c0 = f (b).
Sei n ≥ 1 und c0 + c1 a + . . . + cn−1 an−1 ≡ c0 + c1 b + . . . + cn−1 bn−1 schon
gezeigt. Nach (a) gilt dann
f (a) = (c0 + c1 a + . . . + cn−1 an−1 ) + cn an ≡ (c0 + c1 b + . . . + cn−1 bn−1 )
+cn an ≡ (c0 + c1 b + . . . + cn−1 bn−1 ) + cn bn = f (b)
34
5.6 Satz. Sei c > 0 und a, b beliebig.
(a) ac ≡ bc mod m und (c, m) = 1 =⇒ a ≡ b mod m
(b) a ≡ b mod m ⇐⇒ ac ≡ bc mod cm
(c) a ≡ b mod m, n | m und n > 0 =⇒ a ≡ b mod n
(d) a ≡ b mod m =⇒ (a, m) = (b, m)
Beweis.
2.8
(a) m | (ac − bc) = (a − b)c und (c, m) = 1 =⇒ m | a − b =⇒ a ≡ b mod m
(b) m | (a − b) ⇐⇒ cm | c(a − b) = ac − bc ⇐⇒ ac ≡ bc mod cm
(c) m | (a − b) und n | m =⇒ n | (a − b) =⇒ a ≡ b mod n
2.8(f )
(d) b = a + mq =⇒ (b, m) = (a + mq, m) = (a, m).
Beispiel für die Anwendung der Kongruenzrechnung.
Behauptung. 232 + 1 ist durch 641 teilbar und somit keine Primzahl.
Beweis. 641 = 640 + 1 = 5 · 27 + 1 =⇒ 5 · 27 ≡ −1 mod 641
=⇒ 54 · 228 = (5 · 27 )4 ≡ (−1)4 = 1 mod 641
=⇒
641 = 625 + 16 = 54 + 24 =⇒ 54 ≡ −24 mod 641
−232 = (−24 ) · 228 ≡ 54 · 228 ≡ 1 mod 641 =⇒ 641 | 232 + 1
n
Fermat glaubte noch, daß alle Zahlen der Form Fn = 22 + 1 Primzahlen
sind. Dies ist auch für n = 0, 1, 2, 3, 4 richtig:
F0 = 21 + 1 = 3, F1 = 22 + 1 = 5, F2 = 24 + 1 = 17, F3 = 28 + 1 = 257,
F4 = 216 + 1 = 65537, aber
641 | (232 + 1) = F5
Fermat hatte Unrecht. Bis heute sind keine weiteren Fermatschen Primzahlen
bekannt.
35
§6 Lineare Kongruenzen
Sei m > 0 und a, b beliebig. Wir wollen die Frage untersuchen, unter welchen
Bedingungen an a, b und m eine Zahl x0 existiert, so daß
ax0 ≡ b mod m.
Wenn ein solches x0 existiert, sagen wir: Die lineare Kongruenz
(∗)
aX ≡ b mod m
in einer Unbestimmten X ist lösbar (und zwar, indem man die Unbestimmte
durch die Zahl x0 ersetzt).
Ist x0 eine Lösung der Kongruenz (∗), so ist auch jedes y aus der Restklasse
von x0 modulo m eine Lösung von (∗). (y ≡ x0 =⇒ ay ≡ ax0 ≡ b mod m
nach 5.5 (b)).
Deshalb versteht man unter der Anzahl der Lösungen von (∗) die Anzahl
der verschiedenen Restklassen von Lösungen. Dies ist auch die Anzahl der
Lösungen x0 mit 0 ≤ x0 < m.
Ferner gilt: Ist a0 ≡ a und b0 ≡ b, so hat a0 X ≡ b0 mod m die gleiche
Lösungsmenge wie (∗). (Beweis: Übungsaufgabe.)
6.1 Satz. Ist (a, m) = 1 so hat die lineare Kongruenz aX ≡ b mod m genau
eine Lösung. Also gibt es genau ein x0 , 0 ≤ x0 < m mit ax0 ≡ b mod m.
Beweis. Nach 5.4(b) bilden a · 0, a · 1, . . . , a(m − 1) ein vollständiges Restsystem modulo m. Also gibt es genau ein x0 mit 0 ≤ x0 < m, so daß
ax0 ≡ b mod m.
Beispiel. a = 10, b = 4, m = 7 : (a, m) = 1
Betrachte die Kongruenz 10X ≡ 4 mod 7. Sie ist wegen 10 ≡ 3 mod 7
äquivalent zur Kongruenz
3X ≡ 4 mod 7.
Berechne den Divisionsrest von 3x0 modulo 7 für 0 ≤ x0 < 7.
36
3·0
3·1
3·2
3·3
3·4
3·5
3·6
=
=
=
=
=
=
=
0
3
6
9
12
15
18
6=
6
=
6
=
≡
≡
≡
≡
4
4
4
2 6= 4
5 6= 4
1 6= 4
4 mod 7
Also ist x0 = 6 die einzige Zahl x0 mit 0 ≤ x0 < 7 und 3x0 ≡ 4 mod 7.
Alle Lösungen bilden die Restklasse von 6 modulo 7. Die Lösungsmenge ist
also
. . . , −22, −8, −1, 6, 13, 20, 27, . . .
6.2 Satz. Die Kongruenz aX = b mod m ist genau dann lösbar, (a, m) | b.
Zusatz. Ist (a, m) | b, so bilden die Lösungen von aX ≡ b mod m genau eine
m
.
Restklasse modulo (a,m)
Zum Beweis von 6.2 zeigen wir zunächst
6.3 Lemma. Sei n ≥ 2 und a1 , . . . , an nicht alle Null. Genau dann ist die
lineare Gleichung
(∗∗)
a1 X1 + . . . + an Xn = c
in den Unbestimmten X1 , . . . , Xn ganzzahlig lösbar, wenn (a1 , . . . , an ) | c.
Insbesondere gilt
• aX + bY = c lösbar ⇐⇒ (a, b) | c
• Ist (a, b) = 1, so ist aX + bY = c lösbar für alle c
Beweis von 6.3 Nach 3.10 besteht die Menge M = {a1 x1 + . . . + an xn |
x1 , . . . , xn ∈ Z} gerade aus den Vielfachen von (a1 , . . . , an ). Also gilt:
(∗∗)
ist lösbar ⇐⇒ c ist Vielfaches von (a1 , . . . an ), d.h. (a1 , . . . , an ) | c.
Beweis des Satzes. aX ≡ b mod m ist lösbar ⇐⇒ Es gibt ein x0 mit
ax0 ≡ b mod m ⇐⇒. Es gibt ein x0 mit m | ax0 − b ⇐⇒. Es gibt x0 , y0 mit
37
ax0 − b = my0 ⇐⇒. Es gibt x0 , y0 mit ax0 + (−m)y0 = b. Letzteres ist nach
6.3 damit äquivalent, daß (a, m) = (a, −m) | b.
Beweis des Zusatzes. Sei (a, m) | b ; Wir setzen
a0 =
a
b
m
, b0 =
und m0 =
(a, m)
(b, m)
(a, m)
Dann gilt nach 5.6(b): ax0 ≡ b mod m ⇐⇒ a0 x0 ≡ b0 mod m0 . Also stimmen
die Lösungsmengen von aX ≡ b mod m und von a0 X ≡ b0 mod m0 überein.
a
m
Nun gilt aber nach 2.8(e) (a0 , m0 ) = ( (a,m)
, (a,m)
) = 1. Aus 6.1 folgt: Die
0
0
0
Lösungsmenge von a X ≡ b mod m besteht aus genau einer Restklasse
modulo m0 .
6.4 Korollar. Wir betrachten die lineare Gleichung
(L)
aX + bY = c ,
a 6= 0 oder b 6= 0
(i) Ist d = (a, b) kein Teiler von c, so ist (L) nicht (ganzzahlig) lösbar.
(ii) Ist d | c (etwa wenn (a, b) = 1), so ist (L) lösbar.
(iii) Aus einem Lösungspaar x0 , y0 bekommt man wie folgt die Gesamtheit
aller Lösungen:
x = x0 + h ·
b
d
,
y = y0 − h ·
a
d
durchläuft die Gesamtheit aller Lösungen von (L), wenn h alle ganzen
Zahlen durchläuft.
Beweis von (iii). ax + by = ax0 + h ab
+ by0 − h ab
= ax0 + by0 = c, somit
d
d
sind die angegebenen Paare Lösungen. Wir zeigen nun, daß jede Lösung von
(L) die angegebene Gestalt hat. Dazu können wir b 6= 0 annehmen.
Sei x, y ein Lösungspaar von (L), also ax + by = c = ax0 + by0 =⇒
6.2
ax ≡ c mod |b|
=⇒
Zusatz
x ≡ x0 mod |b|
=⇒
d
ax0 ≡ c mod |b|
x = x0 + h db mit h ∈ Z =⇒ by = c − ax =
= c − a(x0 + h db ) = (c − ax0 ) − b ha
= by0 − b ha
=
d
d
ha
ha
= b(y0 − d ) und b 6= 0 =⇒ y = y0 − d .
38
Spezialfall (a, b) = 1: Ist (a, b) = 1 und x0 , y0 eine beliebige Lösung von
aX + bY = c, so wird die Lösungseinheit beschrieben durch die Formeln
x = x0 + hb,
y = y0 − ha,
h ∈ Z beliebig.
Der Chinesische Restsatz.
6.5 Lemma. Seien m1 > 0 und m2 > 0 teilerfremd. Dann haben die Kongruenzen
X ≡ a1 mod m1
X ≡ a2 mod m2
eine gemeinsame Lösung.
Zusatz. Die Menge aller gemeinsamen Lösungen besteht aus genau einer
Restklasse modulo m1 m2 .
Beweis. Es sind z1 , z2 zu finden, so daß a1 + z1 m1 = a2 + z2 m2 . Dann ist
x = a1 +z1 m1 = a2 +z2 m2 eine gemeinsame Lösung der obigen Kongruenzen.
Äquivalente Aufgabe: Finde z1 , z2 , so daß
a2 − a1 = z1 m1 + z2 (−m2 ).
Wegen (m1 , −m2 ) = (m1 , m2 ) = 1 gibt es nach 6.3 solche z1 , z2 .
Eindeutigkeit der Lösung modulo m1 m2 : Wegen (m1 , m2 ) = 1 gilt nach
2.7: m1 m2 = kgv(m1 , m2 ).
Sind x und y Lösungen der beiden Kongruenzen, so ist x ≡ y ≡ a1 mod m1
und x ≡ y ≡ a2 mod m2 , also m1 | (x − y) und m2 | (x − y). Nach 2.6 gilt
daher m1 m2 = kgV (m1 , m2 ) | (x − y) =⇒ x ≡ y mod m1 m2 .
Ist umgekehrt x gemeinsame Lösung und y ≡ x mod m1 m2 , so gilt auch
y ≡ x mod m1 und y ≡ x mod m2 . Es folgt y ≡ x ≡ a1 mod m1 , y ≡ x ≡ a2
mod m2 , d.h. y ist ebenfalls gemeinsame Lösung.
6.6 Der Chinesische Restsatz. Sei r ≥ 2 und seien m1 , . . . , mr positiv und
paarweise teilerfremd (d.h. (mi , mj ) = 1 falls i 6= j). Dann hat das System
von Kongruenzen


 X ≡ a1 mod m1

 X ≡ a2 mod m2
(∗)
..

.


 X ≡ a mod m
n
n
39
eine gemeinsame Lösung.
Zusatz. Die Menge der gemeinsamen Lösungen besteht aus genau einer Restklasse modulo m = m1 · . . . · mr .
Beweis. (Induktion nach r.) Für r = 2 bewiesen in 6.5. Sei r > 2 und die
Behauptung sei für r − 1 schon bewiesen.
Induktionsschluß: Es gibt nach Annahme eine Zahl a0 , so daß die Lösungsmenge von


 X ≡ a1 mod m1
..
(∗∗)
.

 X ≡ a
r−1 mod mr−1
aus allen Zahlen x mit x ≡ a0 mod (m1 · . . . · mr−1 ) besteht.
M.a.W.: Die Lösungsmenge von (∗∗) stimmt mit der Lösungsmenge der Kongruenz X ≡ a0 mod (m1 · . . . · mr−1 ) überein.
Daher stimmt die Lösungsmenge von (∗) mit der Lösungsmenge von
X ≡ a0 mod (m1 · . . . · mr−1 )
(∗ ∗ ∗)
X ≡ ar mod mr
überein. Dabei ist auch (m1 · . . . · mr−1 , mr ) = 1, da (mi , mr ) = 1 für i =
1, . . . , r − 1. Nach 6.5 ist daher (∗ ∗ ∗) lösbar, und die Lösungsmenge besteht
aus genau einer Restklasse modulo (m1 · . . . · mr−1 )mr = m.
Verfahren zur Lösung einer simultanen linearen Kongruenz.
Seien Kongruenzen X ≡ ai mod mi , i = 1, . . . , r vorgegeben mit paarweise
teilerfremden m1 , . . . , mr .
Q
1. Schritt. Setze bi =
mj für i = 1, . . . , r
j6=i
2. Schritt. Löse die Kongruenzen bi Xi ≡ ai mod mi . Dies ist nach 6.1
möglich, denn (bi , mi ) = 1.
3. Schritt. Berechne x := b1 x1 + b2 x2 + . . . + br xr .
Behauptung: x löst das obige System von linearen Kongruenzen.
Beweis. mi |bj für i 6= j =⇒ bj xj ≡ 0 mod mi für i 6= j =⇒ x ≡ bi xi ≡ ai
mod mi für i = 1, . . . , r.
Beispiel. X ≡ 1 mod 2, X ≡ 2 mod 3, X ≡ 4 mod 5.
40
1. m = 2 · 3 · 5 = 30, b1 = 15, b2 = 10, b3 = 6. Löse
2. (a) 15X1 ≡ 1 mod 2 : x1 = 1
(b) 10X2 ≡ 2 mod 3 : x2 = 2
(c) 6X3 ≡ 4 mod 5 : x3 = 4
3. x = b1 x1 + b2 x2 + b3 x3 = 15 · 1 + 10 · 2 + 6 · 4 = 59
x = 59 ist eine Lösung. Die allgemeine Lösung ist
59 + λ · 30,
λ∈Z
Also sind z.B. auch 59 − 30 = 29 und 59 − 60 = −1 Lösungen.
Anmerkung. Beim obigen Lösungsverfahren waren im 2. Schritt Kongruenzen der Form bi X ≡ ai mod mi mit (bi , mi ) = 1 aufgetreten. Bei großen
Zahlen hilft Probieren nicht viel:
Verfahren zur Lösung einer Kongruenz cX ≡ d mod n mit (c, n) = 1.
1. Schritt. Finde (mit Hilfe des euklidischen Algorithmus) Zahlen y und z,
so daß
cy + nz = 1
2. Schritt. Setze x := y · d. Dann ist cx ≡ d mod n. (Ändert man x um ein
Vielfaches von n ab (x0 = x+kn), so gilt ebenfalls: cx0 = cx+(ck)n ≡ cx ≡ d
mod n.)
Beweis. cy + nz = 1 =⇒ cy ≡ 1 mod n =⇒ cx = (cy)d ≡ 1 · d = d mod n,
also cx ≡ d mod n.
Beispiel. 44X ≡ 5 mod 97 : c = 44, d = 5, n = 97
1. Schritt. Zeige daß (44, 97) = 1 und löse 44Y + 97Z = 1 (vgl. §2)
97 = 2 · 44 + 9
44 = 4 · 9 + 8
9 = 1·8+ 1
=⇒ (44, 97) = 1
Liest man den Algorithmus von unten nach oben, so ergibt sich eine Lösung
von 44Y + 97Z = 1 (vgl. §2)
41
1 = 9−1·8
8 = 44 − 4 · 9
1 = 9 − 1(44 − 4 · 9) = (−1) · 44 + 5 · 9
1 = (−1)44 + 5 · 9
9 = 97 − 2 · 44
1 = (−1) · 44 + 5(97 − 2 · 44) = 44 · (−11) + 97 · 5
Es folgt y = −11.
2. Schritt. X = y · d = (−11) · 5 = −55
Dann ist auch −55 + 97 = 42 eine Lösung.
Fazit: 44 · 42 ≡ 5 mod 97
Probe: 44 · 42 = 1848
1848 : 97 = 19 Rest 5, d.h. 1848 ≡ 5 mod 97.
42
§7 Der kleine Satz von Fermat
Polynomkongruenz modulo p. Sei p eine Primzahl, n ≥ 0 und c0 , . . . , cn ∈
Z. Wir betrachten die Kongruenz
(∗)
c0 + c1 X + . . . + cn−1 X n−1 + cn X n ≡ 0 mod p
d.h.: Wir suchen alle x ∈ Z mit der Eigenschaft
c0 + c1 x + . . . + cn−1 xn−1 + cn xn ≡ 0 mod p
Diese Zahlen nennt man die Lösungen von (∗). Nach 5.5 gilt:
(1) Die Lösungsmenge ändert sich nicht, wenn man in (∗) die Koeffizienten
c0 , . . . , cn ersetzt durch c00 , . . . , c0n mit ci ≡ c0i mod p.
(2) Mit einer Lösung x0 von (∗) ist auch jedes y mit y ≡ x0 mod p eine
Lösung von (∗). Also ist die Lösungsmenge von (∗) die Vereinigung von
vollen Restklassen modulo p. Daher erklärt man:
Definition. Die Anzahl der Lösungen von (∗) ist die Anzahl der Restklassen
modulo p, deren Elemente (∗) lösen. (Diese ist auch gleich der Anzahl der
Lösungen x0 mit 0 ≤ x0 < p.)
7.1 Satz. Ist p kein Teiler von cn , so hat (∗) höchstens n Lösungen.
Beweis. Für n = 0 ist dies klar.
Induktionsannahme. 7.1 ist richtig für n − 1.
Schluß von n − 1 auf n. Angenommen (∗) habe n + 1 paarweise inkongruente Lösungen x0 , x1 , . . . , xn . Setze für x ∈ Z
f (x) := c0 + c1 x + . . . + cn xn . Es gilt dann (wie man nachrechnet) n
P
i−1
f (x) − f (x0 ) = (x − x0 )
ci (xi−1 + x0 xi−2 + . . . + xi−2
0 x + x0 )
i=1
= (x − x0 )g(x),
wobei g(X) ein Polynom der Form
g(X) = b0 + b1 X + . . . + bn−1 X n−1 ist mit bn−1 = cn , p - bn−1
Speziell gilt für X = xk , k = 1, . . . , n:
(xk − x0 )g(xk ) = f (xk ) − f (x0 ) ≡ 0 − 0 ≡ 0 mod p, und nach Annahme ist
43
(xk − x0 ) 6≡ 0 mod p. Also gilt
p|(xk − x0 )g(xk ), p - xk − x0 =⇒ p|g(xk ), d.h. die Kongruenz
g(X) ≡ 0 mod p
hat die n paarweise inkongruenten Lösungen x1 , . . . , xn . Dies widerspricht
der Induktionsannahme. Also ist die Annahme falsch, daß (∗) n + 1 inkongruente Lösungen hat.
Definition. Die zahlentheoretische Funktion
ϕ : N\{0} −→ N\{0}
m 7−→ Anzahl der zu m teilerfremden Zahlen b mit 1 ≤ b ≤ m
heißt Eulersche ϕ–Funktion.
Beispiele. ϕ(1) = 1, ϕ(2) = 1, ϕ(3) = 2, ϕ(4) = 2, ϕ(5) = 4,
ϕ(6) = 2, ϕ(7) = 6, ϕ(8) = 4, ϕ(9) = 6, ϕ(10) = 4, ϕ(11) = 10, ϕ(12) = 4, . . .
Ist b ≡ a mod m, so gilt nach 5.6: (a, m) = (b, m).
Insbesondere: Es gibt genau ϕ(m) Restklassen modulo m, welche sämtlich
aus zu m teilerfremden Zahlen bestehen. Sie heißen die primen Restklassen
modulo m. Die übrigen Zahlen haben mit m einen Teiler t > 1 gemeinsam.
Definition. Ein reduziertes Restsystem modulo m ist ein System von
ϕ(m) paarweise inkongruenten Zahlen mod m, welche zu m teilerfremd sind.
Man erhält es aus einem vollständigen Restsystem modulo m, indem man
alle Zahlen wegläßt, die mit m einen Teiler t > 1 gemeinsam haben.
Beispiele.
m=6:
1
1
m=7: 0 1
− 1
2
−
2
2
3
−
3
3
4
−
4
4
5
5
5
5
6
−
6
6
ist
ist
ist
ist
ein
ein
ein
ein
vollständiges Restsystem
reduziertes Restsystem
vollständiges Restsystem
reduziertes Restsystem
Es soll nun gezeigt werden, daß ϕ multiplikativ ist.
7.2 Bemerkung. Ist a1 , . . . , aϕ(m) ein reduziertes Restsystem mod m und
ist (a, m) = 1, so ist auch a1 a, . . . , aϕ(m) a ein solches. Dies ergibt sich sofort
aus 5.6.
44
7.3 Lemma. Seien a > 0, b > 0 und (a, b) = 1. Durchläuft x ein vollständiges
Restsystem mod b und y ein vollständiges Restsystem mod a, so durchläuft
ax + by ein vollständiges Restsystem mod ab.
Beweis. Zu zeigen: Die ab Zahlen ax+by sind paarweise inkongruent modulo
ab, wenn x und y vollständige Restsysteme mod b bzw. mod a durchlaufen;
M.a.W.: Aus ax1 + by1 ≡ ax2 + by2 mod ab folgt x1 ≡ x2 mod b und y1 ≡ y2
mod a.
Bew.: ax1 + by1 ≡ ax2 + by2 mod ab =⇒ a1 x1 + by1 ≡ ax2 + by2 mod
b =⇒ ax1 ≡ ax2 mod b. Wegen (a, b) = 1 folgt nach 5.6: x1 ≡ x2 mod b.
Analog zeigt man y1 ≡ y2 mod a.
7.4 Lemma. Das Lemma 7.3 bleibt richtig, wenn man darin vollständig“
”
durch reduziert“ ersetzt.
”
Beweis. Nach dem Beweis von 7.3 ist noch zu zeigen:
(x, b) = (y, a) = 1 ⇐⇒ (ax + by, ab) = 1
⇐=“: Sei d := (x, b) > 1 : d | x und d | b =⇒ d | ax + by
”
und d | ab =⇒ (ax + by, ab) ≥ d > 1.
Analog folgt (ax + by, ab) > 1 aus (y, a) > 1.
=⇒“: Sei (x, b) = (y, a) = 1. Zu zeigen: (ax + by, ab) = 1
”
Angenommen die Primzahl p teilt (ax + by, ab) =⇒ p | ab. Wir können
annehmen, daß p | a.
Aus p | ax + by
und p | a folgt p | by
=⇒ p | y
Wegen (a, b) = 1 und p | a folgt p - b
p | a und p | y widerspricht aber (y, a) = 1.
Aus 7.3 und 7.4 ergibt sich
7.5 Korollar. Seien a > 0, b > 0 und (a, b) = 1. Dann ist
ϕ(ab) = ϕ(a)ϕ(b)
ϕ ist also eine multiplikative Funktion. Zur Berechnung von ϕ(a) müssen wir
also nur noch ϕ(pn ), n ≥ 1, p Primzahl kennen.
7.6 Satz. Für a = pn , n ≥ 1 ist
1
ϕ(pn ) = pn (1 − )
p
45
Beweis. Für k ∈ N = {1, 2, . . . , pn } gilt offenbar:
(k, pn ) 6= 1 ⇐⇒ p | k ⇐⇒ k ∈ {1 · p, 2 · p, 3 · p, . . . , pn−1 · p} =: M
M hat pn−1 Elemente; also besteht N \M aus pn − pn−1 Elementen und
N \M = {x | 1 ≤ x ≤ pn und (x, pn ) = 1}, d.h. ϕ(pn ) = Anzahl der
Elemente von N \M = pn − pn−1 = pn (1 − p1 ).
7.7 Korollar. Sei a > 1 und a = pe11 · . . . · perr , r ≥ 1, eρ ≥ 1 die kanonische
Zerlegung von a. Dann ist
ϕ(a) = a
r Y
ρ=1
1
1−
pρ
Beweis. Da die Zahlen pe11 , . . . , perr paarweise teilerfremd sind, ergibt sich
rekursiv aus 7.5
ϕ(a) = ϕ(pe11 ) · . . . · ϕ(perr )
Wegen 7.6 gilt daher
ϕ(a) =
r Y
1
1
1
er
1−
· . . . · pr 1 −
=a
1−
p1
pr
pρ
ρ=1
pe11
7.8 Der kleine Satz von Fermat. Sei m > 1. Dann gilt
(a, m) = 1 ⇐⇒ aϕ(m) ≡ 1 mod m
Beweis. ⇐=“: Sei aϕ(m) ≡ 1 mod m. Dann ist nach 5.6
”
1 = (1, m) = (aϕ(m) , m), also auch (a, m) = 1.
=⇒“: Sei (a, m) = 1 und a1 , . . . , aϕ(m) ein reduziertes Restsystem mod m.
”
Dann ist nach 7.2 auch aa1 , . . . , aaϕ(m) ein solches.
Nach 5.6 folgt:
aϕ(m)
ϕ(m)
Q
n=1
ϕ(m)
Q
ϕ(m)
Q
n=1
ϕ(m)
Q
n=1
(aan ) ≡
an ≡ 1 ·
an mod m, also
an mod m. Kürzen ergibt aϕ(m) ≡ 1 mod m.
n=1
(Kürzen ist nach 5.6 wegen (a1 · . . . · aϕ(m) , m) = 1 erlaubt.)
Mit ϕ(p) = p − 1 ergibt sich
46
7.9 Korollar. Für p - a ist ap−1 ≡ 1 mod p. Also gilt
ap ≡ a mod p für jedes a ∈ Z.
7.10 Satz von Wilson. (p − 1)! ≡ −1 mod p.
M.a.W.: (p − 1)! + 1 ist ein Vielfaches von p.
Beweis. Betrachte das Polynom
f (X) := X p−1 − 1 −
p−1
Y
(X − m)
m=1
Es ist von der Form f (X) = c0 + c1 X + . . . + dp−2 X p−2 .
Betrachte die Polynomkongruenz
(∗)
c0 + c1 X + . . . + cp−2 X p−2 ≡ 0 mod p
Nach 7.9 gilt für a ∈ {1, 2, . . . p − 1} : f (a) = ap−1 − 1 ≡ 0 mod p,
d.h.: (∗) hat p − 1 Lösungen.
Nach 7.1 kann das nur sein, wenn ci ≡ 0 mod p für i = 0, 1, . . . , p − 2.
p−1
Q
Nun gilt aber c0 = f (0) = −1 −
(−m) = −1 − (−1)p−1 (p − 1)!
m=1
Also ist −1 ≡ (−1)p−1 (p − 1)! mod p.
Für p ≥ 3 ist p ungerade, also (−1)p−1 = 1 und daher
(−1) ≡ (p − 1)! mod p
Es gilt auch (2 − 1)! ≡ −1 mod 2, da 1 ≡ −1 mod 2.
7.11 Satz. Sei d ein positiver Teiler von p − 1. Dann hat die Kongruenz
X d ≡ 1 mod p genau d Lösungen.
Beweis. Sei e ∈ N mit de = p − 1. Nach der Formel für die geometrische
Reihe ist
p−1
d
d
d 2
d e−1
X
−1= X −1 1+X + X
+ ... + X
= X d − 1 g (X)
Wegen (X d )e−1 = X d(e−1) ist g(X) ein Polynom vom Grad d(e − 1). Es folgt
47
(1) g(X) ≡ 0 mod p hat höchstens d(e − 1) Lösungen (7.1)
(2) X p−1 − 1 ≡ 0 mod p hat genau ϕ(p) = p − 1 Lösungen (7.8)
a löst X p−1 − 1 ≡ 0 mod p =⇒ p | ap−1 − 1 = (ad − 1)g(a) =⇒ p | ad − 1
oder p | g(a) =⇒ a löst X d − 1 ≡ 0 mod p oder a löst g(x) ≡ 0 mod p.
Also gilt
(3) a ist Lösung von X p−1 − 1 ≡ 0 mod p =⇒ a ist Lösung von
g(X) ≡ 0 mod p oder von X d − 1 ≡ 0 mod p.
Aus (1) bis (3) folgt: X d −1 ≡ 0 mod p hat mindestens p−1−d(e−1)
Lösungen. Aber p − 1 − d(e − 1) = d und nach 7.1 gilt auch
(4) X d − 1 ≡ 0 mod p hat höchstens d Lösungen.
p−1
7.12 Korollar. Sei p ≥ 3. Dann hat X 2 ≡ 1 mod p genau
nämlich die Restklassen von 12 , 22 , . . . , ( p−1
)2 .
2
p−1
2
Lösungen,
p−1
Beweis. (a2 ) 2 ≡ ap−1 ≡ 1 mod p für a 6≡ 0 mod p nach 7.8.
Nach 7.11 ist noch zu zeigen: 12 , 22 , . . . , ( p−1
)2 sind paarweise inkongruent
2
modulo p.
Angenommen es existieren a, b mit 1 ≤ a < b ≤ p−1
mit a2 ≡ b2 mod p.
2
Dann ist (p − a)2 ≡ (−a)2 ≡ a2 mod p und die Kongruenz X 2 ≡ a2 mod p
hat die drei Lösungen a, b und p − a, wobei
p>p−a≥p−
p−1
p+1
=
> b > a ≥ 1,
2
2
im Widerspruch zu 7.1.
7.13 Korollar. Sei p ≥ 3. Die Kongruenz X 2 ≡ −1 mod p hat eine Lösung
(das ist ein x mit p | x2 + 1), wenn p ≡ 1 mod 4 und keine Lösung, wenn
p ≡ 3 mod 4.
Beispiele.
3 ≡ 3 mod 4 : 3 - 12 + 1, 3 - 22 + 1 = 5,
also gibt es kein x ∈ N mit x2 ≡ −1 mod 3
5 ≡ 1 mod 4 : 5 | 22 + 1 = 5
22 ≡ −1 mod 5
13 ≡ 1 mod 4 : 13 | 26 = 52 + 1
52 ≡ −1 mod 5
7 ≡ 3 mod 4 : Nach 7.13 gibt es kein x ∈ N mit
7 | x2 + 1, d.h. x2 ≡ 1 mod 7
48
Beweis von 7.13 Ist a2 ≡ −1 mod p, so ist nach 7.8
(−1)
p−1
2
≡ ap−1 ≡ 1 mod p
p−1
Dann muß (−1) 2 = 1 sein, da −1 6≡ 1 mod p für p ≥ 3. Es folgt: p−1
ist
2
gerade, d.h. 4 | p − 1, d.h. p ≡ 1 mod 4. Ist umgekehrt p ≡ 1 mod 4, so ist
p−1
(−1) 2 ≡ 1 mod p und daher −1 ≡ a2 mod p mit 1 ≤ a ≤ p−1
nach 7.12.
2
Wir haben also gesehen:
Zusatz. Ist p ≡ 1 mod 4, so gibt es ein a mit 1 ≤ a ≤
mod p.
p−1
,
2
so daß a2 ≡ −1
7.14 Teilersummenformel. Für alle m ≥ 1 gilt
X
ϕ(d) = m
d|m
d≥1
Beweis.
Nach 7.5 ist ϕ multiplikativ. Nach Satz 4.4 ist dann auch φ(m) =
P
ϕ(d) multiplikativ, ebenso wie χ(m) = m.
d|m
d≥1
Es genügt daher, 7.14 für Primzahlpotenzen pn , n ≥ 1 zu beweisen. Es ist
ϕ(1) = 1 und ϕ(pk ) = pk − pk−1 für 1 ≤ k ≤ n. Daher ist
φ(pn ) = ϕ(1) + ϕ(p) + . . . + ϕ(pn−1 ) + ϕ(p)
= 1 + (p − 1) + . . . + (pn−1 − pn−2 ) + (pn − pn−1 ) = pn
49
§8 Summen von Quadraten
A. Summen von zwei Quadraten. Sei p eine Primzahl.
Beispiele. 2 = 12 + 12 , 5 = 12 + 22 , 13 = 22 + 32
Aber 3 und 7 sind nicht Summen von zwei Quadraten.
8.1 Satz. Genau dann ist p Summe von zwei Quadraten, wenn entweder
p = 2 oder p ≡ 1 mod 4(p = 2, 5, 13, 17, 29, . . .).
Beweis. 2 = 12 + 12 . Sei p ≥ 3. Dann ist p ≡ 1 mod 4 oder p ≡ 3 mod 4.
1. Sei p ≡ 3 mod 4
z ≡ 1 mod 4 =⇒ z 2 ≡ 1 mod 4
z ≡ 2 mod 4 =⇒ z 2 ≡ 0 mod 4
z ≡ 3 mod 4 =⇒ z 2 ≡ 9 ≡ 1 mod 4
z ≡ 0 mod 4 =⇒ z 2 ≡ 0 mod 4
Also ist für alle z ∈ Z : z 2 ≡ 0 mod 4 oder z 2 ≡ 1 mod 4.
Es folgt: x2 + y 2 ≡ 0, 1 oder 2 mod 4 für alle x, y ∈ Z. Insbesondere ist
x2 + y 2 6= p für alle x, y ∈ Z.
2. Sei p ≡ 1 mod 4. Nach 7.13 gibt es ein x0 ∈ Z, 0 < x0 <
2
x20 ≡ −1 mod p, d.h. x20 + 1 = k · p < p4 + 1 < p2 .
Kürzen ergibt 1 ≤ k < p.
Damit gibt es ein k, 1 ≤ k < p, so daß die Gleichung
(1)
p
2
mit
X 2 + Y 2 = kp
eine Lösung x0 , y0 aus Z hat.
Sei k die kleinste positive ganze Zahl, so daß (1) lösbar ist.
Behauptung. k = 1, und somit existieren x0 , y0 mit x20 + y02 = p.
Beweis. Angenommen k ≥ 2. (Wir konstruieren ein l, 1 ≤ l < k, so daß
X 2 + Y 2 = lp eine Lösung hat, im Widerspruch zur Minimalität von k. Also
ist tatsächlich k = 1.)
Seien also x0 , y0 ∈ N mit x20 + y02 = kp und 2 ≤ k < p. Angenommen k | x0
und k | y0 . Dann ist k 2 | kp, im Widerspruch zu 2 ≤ k < p. Wir können
also annehmen, daß k - x0 . Nach 5.4 existiert ein vollständiges Restsystem
modulo k aus Zahlen z mit |z| ≤ k2 . Wähle also x1 und y1 mit
x0 ≡ x1 mod k, y0 ≡ y1 mod k und |x1 | ≤
50
k
k
, |y1 | ≤
2
2
Dann gilt: x21 + y12 ≡ x20 + y02 ≡ 0 mod k, wobei x1 6= 0, da k - x0 .
Es folgt: x21 + y12 = lk mit lk ≥ x21 ≥ 1 und lk = x21 + y12 ≤ 2( k2 )2 =
l ≤ k2 < k.
k
2
· k, also
Verwende nun die folgende allgemeine Identität für Quadrupel z1 , z2 , z3 , z4
von Zahlen (nachrechnen!)
(2)
(z12 + z22 )(z32 + z42 ) = (z1 z3 + z2 z4 )2 + (z1 z4 − z2 z3 )2
Wende (2) an auf das Quadrupel x0 , y0 , x1 , y1 . Erhalte
(3) kplk = (x20 + y02 )(x21 + y12 ) = (x0 x1 + y0 y1 )2 + (x0 y1 − x1 y0 )2 = x̃22 + ỹ22 ,
wobei
x̃2 = x0 x1 + y0 y1 ≡ x20 + y02 ≡ 0 mod k
ỹ2 = x0 y1 − x1 y0 ≡ x0 y0 − x0 y0 ≡ 0 mod k
wegen x0 ≡ x1 und y0 ≡ y1 mod k.
Also ist x̃2 = kx2 , ỹ2 = ky2 . Kürze in (3) durch k 2 , erhalte x22 + y22 = pl, 1 ≤
l < k < p, im Widerspruch zur Minimalität von k. Also ist die Annahme
falsch und k = 1.
Frage: Wann schreibt sich ein beliebiges m ∈ N (m kein Quadrat) als Summe
von zwei Quadraten?
8.2 Satz. Sei m > 1 kein Quadrat. Schreibe m in der Form
m = 2e dp1 · . . . · pr , d = n2 ≥ 1, 2 < p1 < · . . . · < pr Primzahlen, r ≥ 0, e ∈
{0, 1}. Genau dann ist m die Summe von zwei Quadraten, wenn pi ≡ 1 mod
4 für alle i = 1, . . . , r.
Beweis. ⇐=“ Sei p1 ≡ p2 ≡ . . . ≡ pr ≡ 1 mod 4.
”
Nach 8.1 sind dann p1 , . . . , pr Summen von zwei Quadraten, ebenso wie d =
n2 + 02 und 2 = 12 + 12 .
Aus (2) folgt durch Induktion nach r :
m = 2 · . . . · 2 · d · p1 · . . . · pr ist ebenfalls Summe von zwei Quadraten.
=⇒“ Angenommen 8.2 ist falsch. Dann gibt es ein kleinstes Gegenbeispiel
”
zu 8.2, also eine kleinste Zahl m > 1 der Form m = 2e dp1 · . . . · pr wie im
Satz und ohne Einschränkung p1 = p ≡ 3 mod 4, so daß m Summe von zwei
Quadraten ist. Schreibe m in der Form
(4)
m = l · p2k+1 , k ≥ 0, (l, p) = 1, p ≡ 3 mod 4
51
Nach Annahme ist x2 + y 2 = m, p | m.
m ist kein Quadrat, also x 6= 0 und y 6= 0.
Es folgt p - y, da sonst p | y, p | x und daher p2 | x2 + y 2 = m und
( xp )2 +( yp )2 = pm2 = m0 ; m0 erfüllt die Voraussetzungen von 8.2, im Widerspruch
zur Minimalität von m. Also ist p - y. Aus 7.9 folgt: y p−1 ≡ 1 mod p.
Wegen x2 ≡ −y 2 mod p folgt
(xy p−2 )2 ≡ x2 (y p−2 )2 ≡ −y 2 (y p−2 )2 = −(y p−1 )2 ≡ −1 mod p
Setze z := xy p−2 . Dann ist z 2 ≡ −1 mod p und p ≡ 3 mod 4. Dies ist ein
Widerspruch zu 7.13. Also gilt 8.2.
B. Summen von vier Quadraten.
8.3 Satz. Jede Zahl m ≥ 0 ist die Summe von vier Quadraten.
Beweis. Wir benutzen die folgende Identität (nachrechnen!)
(5)
(x21 + x22 + x23 + x24 )(y12 + y22 + y32 + y42 ) = d21 + d22 + d23 + d24 , wobei
d1
d2
d3
d4
=
=
=
=
x1 y1 + x2 y2 + x3 y3 + x4 y4
x1 y2 − x2 y1 + x3 y4 − x4 y3
x1 y3 − x3 y1 + x4 y2 − x2 y4
x1 y4 − x4 y1 + x2 y3 − x3 y2
Insbesondere folgt induktiv: Sind a1 , . . . , an Summen von vier Quadraten, so
ist auch a1 · . . . · an Summe von vier Quadraten.
Ferner ist 0 = 02 + 02 + 02 + 02 , 1 = 12 + 02 + 02 + 02 , 2 = 12 + 12 + 02 + 02 .
Wegen (5) genügt es also, die Behauptung für Primzahlen p ≥ 3 zu beweisen.
I. Sei p ≥ 3 vorgegeben. Konstruiere ein k, 1 ≤ k < p, so daß X12 + X22 +
X32 + X42 = kp lösbar ist:
)2 } paarweise
Nach 7.12 sind die Elemente von S = {02 , 12 , . . . ( p−1
2
inkongruent modulo p. Dies gilt dann auch für die Menge T = {−1−q |
q ∈ S}. Offenbar sind S und T disjunktiv. Daher besteht S ∪ T aus
p + 1 Elementen. Zwei davon müssen kongruent sein, und sie können
nicht beide zu S bzw. T gehören. Also existieren s ∈ S und t ∈ T mit
s ≡ t mod p. Es folgt
und x2 ≡ −1 − y 2 mod p, d.h.
s = x2 , t = −1 − y 2 , 0 ≤ x, y ≤ p−1
2
p−1
2
2
2
2
x + y + 1 + 0 = kp, 0 ≤ x, y ≤ 2 , k ≥ 1
52
Aus 0 ≤ x, y <
p
2
2
folgt 1 ≤ kp < 2 p4 + 1 =
p2
2
+ 1 < p2 , also 1 ≤ k < p.
Also gibt es ein k mit 1 ≤ k < p, so daß
(6)
X12 + X22 + X32 + X42 = kp
eine Lösung hat. Wähle k minimal mit diesen Eigenschaften.
II. Es ist noch zu zeigen, daß k = 1 ist.
Wir nehmen an, daß k ≥ 2 ist und führen dies zum Widerspruch. Wähle
x1 , x2 , x3 , x4 aus N mit x21 + x22 + x23 + x24 = kp.
Teilt k alle xi , so teilt k 2 auch kp, und k = p, Widerspruch. Also ist
o.E. k - x1 .
1. Fall: k ist gerade. Wegen x21 + x22 + x23 + x24 = kp gerade können 3
Fälle auftreten:
(i) alle xi gerade
(ii) alle xi ungerade
(iii) x1 , x2 gerade und x3 , x4 ungerade (nach evtl. Umordnung)
In jedem Fall sind x1 + x2 , x1 − x2 , x3 + x4 , x3 − x4 sämtlich gerade.
2
2
4
4
=⇒ y1 = x1 +x
, y2 = x1 −x
, y3 = x3 +x
, y4 = x3 −x
sind alle ganz, und
2
2
2
4
1
k
2
2
2
2
2
2
2
2
y1 + y2 + y3 + y4 = 2 (x1 + x2 + x3 + x4 ) = 2 · p mit 1 ≤ k2 < k < p, im
Widerspruch zur Minimalität von k.
2. Fall: k ist ungerade. Nach 5.4 existieren y1 , y2 , y3 , y4 mit xi ≡ yi mod
k und |yi | ≤ k−1
, i = 1, 2, 3, 4.
2
Nach Voraussetzung ist y1 6= 0. Es folgt
y12 + y22 + y32 + y42 ≡ x21 + x22 + x23 + x24 ≡ 0 mod k, also
0 6= k 0 k = y12 + y22 + y32 + y42 < 4( k2 )2 = k 2 und 1 ≤ k 0 < k.
Nach (5) gilt d21 + d22 + d23 + d24 = (x21 + x22 + x23 + x24 )(y12 + y22 + y32 + y42 ) =
kpk 0 k.
Wegen x1 ≡ yi mod k gilt:
53
d1 ≡ x1 y1 + x2 y2 + x3 y3 + x4 y4 ≡ x21 + x21 + x23 + x24 ≡ 0 mod k und
d2 ≡ x1 x2 − x2 x1 + x3 x4 − x4 x3 ≡ 0 mod k, d.h. k | d1 und k | d2
Analog zeigt man: k | d3 und k | d4 .
Also sind die zi = dki ganz und
z12 + z22 + z32 + z42 = k 0 p, 1 ≤ k 0 < k < p
im Widerspruch zur Minimalität von k. Also ist k = 1 und es existieren
x1 , x2 , x3 , x4 ∈ N mit
x21 + x22 + x23 + x24 = p
C. Summen von drei Quadraten.
8.4 Satz. Eine ganze Zahl m > 1 ist genau dann Summe von drei Quadraten,
wenn sie nicht von der Form
m = 4a b, a ≥ 0 und b ≡ 7 mod 8
ist. (Beispielsweise ist 7 = 40 · 7 ist nicht Summe von drei Quadraten.)
Wir beweisen nur die einfache Richtung: Sei m = 4a b, a ≥ 0, b ≡ 7 mod 8.
Zeige, daß m nicht Summe von drei Quadraten ist.
Induktion nach a. a = 0 =⇒ m = b ≡ 7 mod 8.
Modulo 8 gerechnet gilt:
z ≡ 0 ⇒ z 2 ≡ 0; z ≡ 1 ⇒ z 2 ≡ 1; z ≡ 2 ⇒ z 2 ≡ 4, z ≡ 3 ⇒ z 2 ≡ 1
z ≡ 4 ⇒ z 2 ≡ 0; z ≡ 5 ⇒ z 2 ≡ 1; z ≡ 6 ⇒ z 2 ≡ 4, z = 7 ⇒ z 2 ≡ 1
Also ist für alle n ∈ Z n2 ≡ 0, 1, oder 4 modulo 8 und x2 + y 2 + z 2 ≡
0, 1, 2, 3, 4, 5 oder 6 modulo 8; insbesondere ist x2 +y 2 +z 2 6≡ 7 für x, y, z ∈ N.
Schluß von a − 1 auf a. Sei a ≥ 1 und sei schon gezeigt, daß 4a−1 b nicht
Summe von 3 Quadraten ist, wenn b ≡ 7 mod 8. Angenommen 4a b = x2 +
y 2 + z 2 ⇒ x2 + y 2 + z 2 ≡ 0 mod 4(a ≥ 1).
Im Beweis von 8.1 gesehen: x2 , y 2 , z 2 ≡ 0 oder 1 mod 4.
Zusammen mit x2 + y 2 + z 2 ≡ 0 mod 4 ergibt sich:
x2 ≡ y 2 ≡ z 2 ≡ 0 mod 4. Dann sind aber x2 , y2 , z2 ganz und 4a−1 b = ( x2 )2 +
( y2 )2 + ( z2 )2 , im Widerspruch zur Induktionsannahme.
54
§9 Pythagoras–Tripel
Nach Pythagoras gilt: In einem rechtwinkligen Dreieck mit den Katheden
a und b und der Hypothenuse c ist
a2 + b2 = c2
Speziell gilt die sogenannte
Zimmermannsregel. Drei Latten der Länge 3, 4 und 5 Meter ergeben zusammengenagelt ein rechtwinkliges Dreieck, denn
32 + 42 = 52
Definition. x, y, z heißt Pythagoras–Tripel, wenn x, y und z positive Zahlen sind mit
x2 + y 2 = z 2
Frage: Wie verschafft man sich einen Überblick über alle Pythagoras–Tripel?
9.1 Bemerkung. Sei x, y, z ein Pythagoras–Tripel. Dann gilt:
(a) x und y sind nicht beide ungerade.
(b) Für alle λ ≥ 1 aus N ist auch λx, λy, λz ein Pythagoras–Tripel.
(c) Teilt t die Zahlen x, y und z, so ist auch xt , yt , zt ein Pythagoras–Tripel.
Beweis. x, y ungerade =⇒ x2 ≡ y 2 ≡ 1 mod 4 (siehe den Beweis von 8.1)
=⇒ x2 + y 2 ≡ 2 mod 4 =⇒ x2 + y 2 ist kein Quadrat (Beweis von 8.1).
(b) und (c) sind offensichtlich richtig.
Zur Beantwortung obiger Frage genügt es danach, diejenigen Pythagoras–
Tripel zu finden mit
(i) x, y, z sind (paarweise) teilerfremd,
(ii) x ist gerade und y ungerade.
Die übrigen Pythagoras–Tripel entstehen aus solchen durch Streckung und
evtl. Vertauschung von x und y.
55
9.2 Satz. (Indische Formeln für Pythagoras–Tripel.)
(a) Sind a ≥ 1, b ≥ 1 mit a > b, a−b ungerade und (a, b) = 1, so bilden x =
2ab, y = a2 −b2 , z = a2 +b2 ein Pythagoras–Tripel mit den Eigenschaften
(i) und (ii) (ein sogenanntes normiertes“ Pythagoras–Tripel).
”
(b) Jedes normierte Pythagoras–Tripel x, y, z ist von der Form x = 2ab, y =
a2 − b2 , z = a2 + b2 mit a ≥ 1, b ≥ 1, a − b ungerade und (a, b) = 1
(c) Es gibt unendlich viele Pythagoras–Tripel.
Beweis.
(a) Seien a, b, x, y, z wie in (a) beschrieben. Dann ist x gerade und
x2 + y 2 = 4a2 b2 + a4 − 2a2 b2 + b4 = a4 + 2a2 b2 + b4 = (a2 + b2 )2 = z 2
Wegen a − b ungerade ist auch a + b ungerade und daher
y = (a + b)(a − b) ungerade.
Noch zu zeigen: x und y sind teilerfremd.
Angenommen die Primzahl p teilt x und y. =⇒ p ist ungerade und
p2 | x2 + y 2 = z 2 =⇒ p ungerade, p | z =⇒ p | y + z = 2a2 und
p | y − z = 2b2 und p ungerade =⇒ p | a und p | b, im Widerspruch zu
(a, b) = 1.
(c) a durchlaufe alle ungeraden Primzahlen p und b := 2. Das Paar a, b
erfüllt dann die Voraussetzungen von (a).
Also ist nach (a) 4p, p2 − 4, p2 + 4 ein normiertes Pythagoras–Tripel für
alle Primzahlen p ≥ 3.
(b) Sei x, y, z ein normiertes Pythagoras–Tripel. =⇒ z 2 = x2 + y 2 ist
ungerade =⇒ z ungerade; y ungerade =⇒ z ± y gerade =⇒ x2 =
(z − y)(z + y) = 4x20 =⇒ x = 2x0 , x20 = z−y
· z+y
, x0 ≥ 1.
2
2
z+y
,
=1
Behauptung. z−y
2
2
Beweis. Angenommen die Primzahl p teilt
p2 | (z − y)(z + y) = z 2 − y 2 = x2 .
z±y
2
=⇒ p |
z+y
2
−
z−y
2
= y und
Es folgt p | y und p | x, im Widerspruch zur Voraussetzung. Also ist
z−y z+y
, 2
= 1 und z−y
· z+y
= x20 ist ein Quadrat =⇒ z−y
= b2 , z+y
=
2
2
2
2
2
2
a , a ≥ 1, b ≥ 1.
56
− z−y
= a2 − b2 , z =
=⇒ x20 = a2 b2 =⇒ x0 = ab =⇒ x = 2x0 = 2ab, y = z+y
2
2
z+y
z−y
2
2
+ 2 = a + b =⇒ (a, b) | x und (a, b) | y und (x, y) = 1 =⇒ (a, b) = 1
2
y = a2 − b2 = (a + b)(a − b) ungerade =⇒ a − b ungerade.
Der große Satz von Fermat. (Andrew Wiles, Oktober 1994)
Für n ≥ 3 hat die Gleichung X n +Y n = Z n keine positive ganzzahlige Lösung
x, y, z. (Ein Highlight der Mathematik.)
Wir notieren hier den Beweis für n = 4, der von Fermat stammt. Euler hat
den (schwierigen) Fall n = 3 bewiesen.
9.3 Satz. Die Gleichung
(1)
X4 + Y 4 = Z4
hat keine Lösung in positiven ganzen Zahlen x, y, z.
Beweis. Zeige, daß die Gleichung
(2)
X4 + Y 2 = Z4
keine positive ganzzahlige Lösung hat.
Dann hat auch (1) keine solche Lösung; denn wäre x4 + y 4 = z 4 so wäre
x4 + w2 = z 4 mit w = y 2 eine Lösung von (2).
Angenommen es gibt positive ganze Zahlen x, y, z mit x4 + y 2 = z 4 .
Sei x, y, z ein solches Tripel mit minimalen z.
Dann sind x, y, z paarweise teilerfremd, denn: Ist p ein gemeinsamer Primteiler von zwei dieser Zahlen, so teilt p (wegen x4 + y 2 = z 4 ) auch die dritte
4 2 4
y
x
z
4
2
4
4
2
+
, im
Zahl und p | y = z − x , also p | y. Es folgt
=
2
p
p
p
Widerspruch zur Minimalität von z.
Es gilt: (x2 )2 +y 2 = (z 2 )2 , also ist x2 , y, z 2 ein Pythagoras Tripel aus paarweise
teilerfremden Zahlen.
1. Fall. x2 ist ungerade und y gerade. Nach 9.2 gibt es a ≥ 1, b ≥ 1 mit
(a, b) = 1, a − b > 0 ungerade und x2 = a2 − b2 , y = 2ab und z 2 = a2 + b2 .
Es folgt: (xz)2 = x2 z 2 = (a2 − b2 )(a2 + b2 ) = a4 − b4 , also b4 + (xz)2 = a4 und
a2 < z 2 , also a < z. Dies steht im Widerspruch zur Minimalität von z.
2. Fall. x2 ist gerade und y ungerade. Nach 9.2 gibt es a > b ≥ 1 mit
x2 = 2ab, y = a2 − b2 , z 2 = a2 + b2 und (a, b) = 1. Wegen y = a2 − b2 ungerade
57
ist a − b ungerade. Es folgt x2 = 2αβ, z 2 = α2 + β 2 , α ungerade, β gerade
(α, β) = 1.
Wende erneut 9.2 an und zwar auf z 2 = α2 + β 2 : Es gibt r ≥ 1, s ≥ 1 mit
z = r2 + s2 , α = r2 − s2 , β = 2rs und (r, s) = 1. Es ist x2 = 2αβ = (r2 −
s2 )4rs; r, s und r2 − s2 sind paarweise teilerfremd. Wegen x2 = (r2 − s2 )4 · r · s
sind daher r, s und r2 − s2 Quadrate: r = u2 , s = v 2 , r2 − s2 = w2 , also
v 4 + w2 = s2 + (r2 − s2 ) = r2 = u4 .
Ferner ist u ≤ u4 = r2 < r2 + s2 = z. Dies steht im Widerspruch zur
Minimalität von z.
Also ist die zu Anfang des Beweises gemachte Annahme falsch und (2) hat
keine positive ganzzahlige Lösung.
9.4 Korollar.
x
∈ R2 | x2 + y 2 = 1} liegen unendlich
(a) Auf dem EinheitskreisK = {
y
viele Punkte mit rationalen Koordinaten x und y.
x
∈ R2 | x4 +y 4 = 1} liegen außer den Punkten
(b) Auf der Kurve C = {
y
0
±1
und
keine Punkte mit rationalen Koordinaten.
±1
0
Beweis.
(a) Ist a, b, c ein normiertes Pythagoras–Tripel,
so ist a2 + b2 = c2 , also
!
a
c
b
c
( ac )2 + ( cb )2 = 1, d.h. der Punkt
rationale Koordinaten.
liegt auf dem Kreis K und hat
Für verschiedene normierte
Pythagoras
Tripel a, b, c und a0 , b0 , c0 sind
!
!
0
auch die Punkte
a
c
b
c
und
a
c0
b0
c0
0
0
verschieden, denn die Brüche ac , cb , ac0 , cb0
liegen in gekürzter Form vor. Aus 9.2(c) folgt die Behauptung.
x
(b) Sei
∈ C mit rationalen Zahlen x, y. Ist x = 0 oder y = 0, so ist
y
y = ±1 oder x = ±1.
58
Angenommen x 6= 0 und y 6= 0, o.E. x > 0 und y > 0. Sei d der
Hauptnenner von x und y: Dann ist d4 (x4 + y 4 ) = d4 · 1 = d4 , also
(dx)4 + (dy)4 = d4
mit positiven ganzen Zahlen dx, dy und d, im Widerspruch zu 9.3.
59
Kapitel II.
Algebraische Grundbegriffe
§1 Ringe und Körper
Für das Rechnen in Z haben wir in Kap. I, §1 Regeln aufgestellt, welche auch
in Q und R gelten. Damit werden Z, Q und R zu Ringen im folgenden Sinn:
Sei R eine Menge, auf der zwei Verknüpfungen, + ( plus“, Addition) und
”
· ( mal“, Multiplikation), erklärt sind.
”
Definition. R, zusammen mit den Operationen + und ·, (kurz (R, +, ·)“)
”
heißt ein Ring, wenn Addition und Multiplikation den folgenden sechs Axionen genügen:
Addition +“
Multiplikation ·“
”
”
1. Eindeutige Ausführbarkeit
Zu je zwei Elementen a, b ∈ R existiert in R in eindeutiger Weise
die Summe a + b
das Produkt a · b
Für a, b, c ∈ R gelten die folgenden Gesetze:
2. Assoziativgesetze
(a + b) + c = a + (b + c)
(a · b) · c = a · (b · c)
a+b=b+a
3. Kommutativgesetze
a·b=b·a
4. Existenz neutraler Elemente
Es gibt ein Element n ∈ R, Es gibt ein Element e ∈ R,
so daß für jedes a ∈ R gilt: so daß für jedes a ∈ R gilt:
a+n=a
a·e=a
Es ist e 6= n.
60
Bezeichnung: n nennt man Nullelement und e Einselement des Rings R.
Bemerkung: Sind 3. und 4. erfüllt, so hat R genau ein Nullelement und
genau ein Einselement. Wir bezeichnen diese (wie in Z) mit 0 ( Null“) und
”
1 ( Eins“).
”
Beweis. Sind n und ñ Nullelemente, so gilt wegen 3. und 4. ñ = ñ + n =
n + ñ = n. Entsprechend schließt man für Einselemente.
5. Umkehrung der Addition Zu jedem a ∈ R gibt es ein
0
Element a ∈ R, so daß gilt
a + a0 = 0
0
a heißt ein Negatives
von a
Bemerkung. Zu jedem a gibt es genau ein Negatives. Wir bezeichnen es
mit −a.
Schreibe für a + (−b) auch a − b und nenne a − b die Differenz zwischen a
und b.
Beweis. Seien a0 und ã Negative von a. Dann gilt a + a0 = 0 = a + ã =⇒
ã = ã + 0 = ã + (a + a0 ) = (ã + a) + a0 = (a + ã) + a0 = 0 + a0 = a0 + 0 = a0 .
6. Das Distributivgesetz
(a + b) · c = (a · c) + (b · c)
für alle a, b, c aus R.
Zur Vereinfachung der Schreibweise führen wir folgende Konventionen ein:
Schreibe ab für a · b
Punktrechnung geht vor Strichrechnung, d.h.
ab + c := (a · b) + c
a + bc := a + (b · c)
Damit schreibt sich z. B. das Distributivgesetz in der Form
(a + b)c = ac + bc
Bei mehrfachen Summen bzw. Produkten werden Klammern weggelassen
(weil es wegen der Assoziativgesetze nicht auf die Art der Klammerung ankommt.) Also
a + b + c := (a + b) + c, abc := (a · b) · c
61
u.s.w.
Definition. Ein Ring R heißt Integritätsbereich, wenn zusätzlich gilt:
7. Nullteilerfreiheit. Aus ab = 0 folgt: a = 0 oder b = 0.
Definition. Ein Ring heißt Körper, wenn zusätzlich zu den Axiomen 1 bis
6 noch gilt:
8. Umkehrbarkeit der Multiplikation. Zu jedem a 6= 0 aus R gibt es ein
Element ã mit aã = 1.
Bemerkung. In einem Körper gibt es zu jedem a ∈ R, a 6= 0 genau ein
ã ∈ R mit aã = 1.
Wir bezeichnen dieses Element mit a−1 und nennen es das zu a reziproke
Element (Kehrwert von a).
˜ =⇒ ã
˜ = ã
˜ · 1 = ãaã
˜ = aãã
˜ = 1ã = ã1 = ã
Beweis. aã = 1 = aã
Wir schreiben für a−1 auch
1
a
und für b · a−1 auch ab .
Beispiele.
a) Z ist ein Ring, sogar ein Integritätsbereich; Z ist kein Körper, denn 2
hat in Z kein Reziprokes.
b) Q und R sind Körper.
c) N ist kein Ring: 1 hat in N kein Negatives.
d) Jeder Körper ist ein Integritätsbereich. (Der Beweis wird später geführt.)
Unterringe. Sei R ein Ring und S ⊆ R eine Teilmenge mit 1 ∈ S. Man
nennt S einen Unterring von R, wenn S abgeschlossen ist unter Addition,
Multiplikation und der Bildung des Negativen, d.h.: Sind a, b ∈ S, so sind
auch −a, a + b und ab aus S. Offenbar ist jeder Unterring eines Rings selbst
ein Ring.
62
Beispiele von Unterringen.
a) Z ist ein Unterring von Q und Q ist ein Unterring von R; N ist kein
Unterring von Z.
b) Reell–quadratische Zahlbereiche. Sei m > 1 eine quadratfreie ganze Zahl, d.h. m = p1 · . . . · pr mit paarweise verschiedenen
√ Primzahlen.
Die positive Quadratwurzel von m bezeichnen wir mit m.
(i) Betrachte in R die Teilmenge
√
√
S = Z[ m] := {a + b m | a, b ∈ Z}
√
√
Offenbar ist 1 = 1 + 0 m ∈ Z[ m]; es ist sogar Z ⊆ S.
S ist abgeschlossen unter den oben
drei
√ genannten
√ Operationen:
0
0
0 0
aus S:
Seien a, b, a , b ∈ Z und r √
= a + b m, s = a + b m √
0
0
r + s = (a + a ) + (b + b ) m ∈
√ S, −r = (−a) + (−b) m ∈ S
rs = (aa0 + bb0 m) + (ab0 + a0 b) m ∈ S.
Also ist S ein Unterring von R.
√
√
(ii) K = Q[ m] := {a + b m | a, b ∈ Q} ist sogar ein Körper:
Wie in
√ (i) zeigt man,2 daß 2K ein Ring ist. Ferner gilt, falls s =
a + b m 6= 0 ist:√a 6= b m, da m kein Quadrat ist. Also
√ ist
x = a2 −b1 2 m · (a − b m) ein wohldefiniertes Element von Q[ m]
√
√
√
√ m)
und (a + b m) · a2 −b1 2 m · (a − b m) = (a+b am)(a−b
=
2 −b2 m
√
2
2
a −b m
−1
= a2 −b2 m = 1 und s = x ∈ Q[ m].
Weitere abkürzende Schreibweisen: Sei m ≥ 1 aus Z und a ∈ R. Schreibe
ma := a
. . + a}
, am := a
. . · a}, ferner
| · .{z
| + .{z
m-mal
m-mal
0 · a := 0, (−m)a := m(−a); a0 := 1, a−m := (a−1 )m , falls a−1 existiert.
Für m · 1 schreiben wir auch kurz m.
Es ergeben sich folgende Regeln:
(−m)a = −(ma); a−m = (am )−1 , falls a−1 existiert, ferner am an =
am+n , an bn = (ab)n , (an )m = an·m für alle n, m ∈ N.
Dies ergibt sich leicht durch Induktion.
1.1 Bemerkung. Sei R ein Ring und a, b, c ∈ R. Dann gilt
63
a) Es gibt genau ein x ∈ R mit x + b = a, nämlich x = a − b.
b) a(b−c) = ab−ac, a0 = 0, −(−a) = a, (−a)b = a(−b) = −ab, (−a)(−b) =
ab.
c) Kürzungsregeln.
(i) Für jeden Ring gilt:
Aus a + b = a + c folgt b = c
(ii) Ist R ein Integritätsbereich, so gilt:
Aus ab = ac und a 6= 0 folgt b = c
d) Ist R ein Körper und b 6= 0, so gibt es zu jedem a genau ein x ∈ R mit
xb = a, nämlich x = ab−1 .
e) Jeder Körper ist ein Integritätsbereich.
Beweis.
a) (a − b) + b = a + (−b + b) = a + (b + (−b) = a + 0 = a
Eindeutigkeit. x+b = a =⇒ x = x+0 = x+(b−b) = (x+b)−b = a−b
c)(i) a + b = a + c =⇒ b + a = c + a =⇒ b = b + a − a = c + a − a = c
b) a · 0 = a · 0 = a(0 + 0) = a0 + a0 =⇒ 0 = a · 0 nach c)(i).
−a + a = a + (−a) = 0 =⇒ a = −(−a)
ab + a(−b) = a(b + (−b)) = a0 = 0 =⇒ a(−b) = −ab
Analog zeigt man (−a)b = −ab
a(b − c) = a(b + (−c)) = ab + a(−c) = ab + (−ac) = ab − ac
c)(ii) ab = ac =⇒ a(b − c) = ab − ac = ab − ab = 0 =⇒ b − c = 0, da a 6= 0
und R nach Voraussetzung ein Integritätsbereich ist.
b − c = 0 =⇒ c = (b − c) + c = b + (−c + c) = b + (c − c) = b + 0 = b
d) (ab−1 )b = a(b−1 b) = a1 = a
Eindeutigkeit. xb = a =⇒ x = x · 1 = x(bb−1 ) = (xb)b−1 = ab−1
e) ab = 0 und a 6= 0 =⇒ a−1 existiert und 0 = a−1 0 = a−1 ab = 1b = b
64
Nullteiler und Einheiten.
Definition. Sei R ein Ring und a ∈ R.
a) a ∈ R heißt Nullteiler von R, wenn es ein b 6= 0 in R gibt mit ab = 0.
(Ein Ring ist also genau ein Integritätsbereich, wenn 0 der einzige Nullteiler von R ist. Man spricht daher bei einem Integritätsbereich auch
von einem nullteilerfreien“ Ring.)
”
b) Ein Element a ∈ R heißt Einheit von R, wenn es ein b ∈ R gibt mit
ab = 1.
(Ein Ring R ist also genau dann ein Körper, wenn jedes a 6= 0 eine
Einheit von R ist.)
Ist a Einheit von R, so ist das b mit ab = 1 eindeutig bestimmt (Beweis!). Es wird mit a−1 bezeichnet. Mit Rt imes bezeichnen wir die
Menge aller Einheiten von R.
Beispiele.
a) In Z sind 1 und −1 die einzigen Einheiten.
b) In Q sind alle Elemente außer der 0 Einheiten.
1.2 Regel.
a) 1 ist eine Einheit.
b) Sind a und b Einheiten, so sind auch a−1 , b−1 und ab Einheiten. (Induktiv folgt daraus: am bn sind Einheiten für alle m, n ∈ Z.)
Beweis.
a) 1 · 1 = 1
b) (ab)(b−1 a−1 ) = a(bb−1 )a−1 = a1a−1 = aa−1 = 1
(a−1 )a = a(a−1 ) = 1.
65
§2 Restklassenringe und Polynomringe
Sei m > 1 ganz und mZ := {mx | x ∈ Z}.
Nach I. 5.3 gilt: Die verschiedenen Restklassen von Z modulo m sind
mZ, 1 + mZ, . . . , (m − 1) + mZ.
Für die Gesamtheit aller Restklassen modulo m schreiben wir
Z/mZ = {a + mZ | a ∈ Z} = {mZ, 1 + mZ, . . . , (m − 1) + mZ}.
Wir wollen die Menge Z/mZ zu einem Ring machen, indem wir Addition
und Multiplikation von Restklassen erklären.
Definition. Seien a, b ∈ Z. Setze
(a + mZ) + (b + mZ) := (a + b) + mZ
(a + mZ) · (b + mZ) :=
ab + mZ
2.1 Bemerkung.
a) Addition und Multiplikation sind unabhängig von der Wahl der Repräsentanten wohl definiert.
b) Z/mZ ist ein Ring mit Null = mZ, Eins = 1 + mZ.
c) Ist m = p eine Primzahl, so ist Z/pZ ein Körper.
d) Ist m > 1 keine Primzahl, so ist Z/mZ kein Integritätsbereich.
Beweis. Schreibe im Folgenden ≡“ für ≡ mod m“.
”
”
0
a) Sei a + mZ = a + mZ und b + mZ = b0 + mZ.
Zu zeigen: (a + b) + mZ = (a0 + b0 ) + mZ und
ab + mZ
=
a0 b0 + mZ
Nach Voraussetzung ist also a ≡ a0 und b ≡ b0 . Aus I. 5.5 folgt
a + b ≡ a0 + b0 und ab ≡ a0 b0 , d.h. (a + b) + mZ = (a0 + b0 ) + mZ und
ab + mZ = a0 b0 + mZ.
66
b) mZ = 0 + mZ und 1 + mZ sind offenbar neutral bezüglich der Addition
bzw. Multiplikation, und (−a) + mZ ist ein Negatives von a + mZ.
Von den Rechenregeln zeigen wir exemplarisch das Distributivgesetz;
für die übrigen Gesetze wären analoge Rechnungen durchzuführen.
(a + mZ) ((b + mZ) + (c + mZ)) = (a + mZ) ((b + c) + mZ) =
= a(b + c) + mZ = (ab + ac) + mZ = (ab + mZ) + (ac + mZ) =
= (a + mZ)(b + mZ) + (a + mZ)(c + mZ).
c) Noch zu zeigen: Ist a + pZ 6= pZ, so gibt es ein b mit (a + pZ)(b + pZ) =
1 + pZ:
I.6.4
a + pZ 6= pZ =⇒ p - a =⇒ (p, a) = 1 =⇒ Es gibt x, y ∈ Z mit
px + ay = 1 =⇒ ay ≡ 1 mod p =⇒ ay + pZ = 1 + pZ, also auch
(a + pZ)(y + pZ) = ay + pZ = 1 + pZ.
d) Ist m = ab mit 0 < a ≤ b < m, so ist
(a + mZ)(b + mZ) = ab + mZ = m + mZ = mZ = 0, aber
a + mZ 6= 0 und b + mZ 6= 0.
Schreibe 1 für die Eins 1 + mZ von Z/mZ und
k für 1| + .{z
. . + 1} = (1 + mZ) + . . . + (1 + mZ) = k + mZ
k−mal
Dann ist Z/Z = {0, 1, 2, . . . , m − 1} und die Addition und Multiplikation in
Z/mZ kann auch wie folgt beschrieben werden:
+
+
(k · l ∈ Z/mZ) = (Divisionsrest modulo m von k · l ∈ Z)
Beispiele für Verknüpfungstabellen
m=3
+
0
1
2
0
0
1
2
1
1
2
0
2
2
0
1
m=4
+
0
1
2
3
0
0
1
2
3
1
1
2
3
0
2
2
3
0
1
·
0
1
2
3
3
0
1
2
0
0
0
0
·
0
1
2
3
0
0
0
0
0
67
1
0
1
2
1
0
1
2
3
2
0
2
1
1·1 = 1
2·2 = 1
2
0
2
0
2
3
0
3
2
1
1·1 = 1
3·3 = 1
2·2 = 0
Einheiten und Nullteiler in Z/mZ. Setze a := a + mZ
Beispiele.
m = 2 : 1 ist Einheit; 0 ist Nullteiler; ϕ(2) = 1
m = 3 : 1, 2 sind Einheiten; 0 ist Nullteiler; ϕ(3) = 2
m = 4 : 1, 3 sind Einheiten; 0, 2 sind Nullteiler; ϕ(4) = 2
m = 6 : 1 · 1 = 1, 2 · 3 = 6 = 0, 4 · 3 = 0, 5 · 5 = 25 = 1 =⇒
1, 5 sind Einheiten; 0, 2, 3, 4 sind Nullteiler; ϕ(6) = 2
m = p Primzahl: Z/pZ ist ein Körper mit p Elementen =⇒ 0 ist Nullteiler,
die übrigen p − 1 Elemente sind Einheiten; ϕ(p) = p − 1.
Diese Rechnungen führen zur
Vermutung. ϕ(m) = Anzahl der Einheiten von Z/mZ.
2.2 Satz. In Z/mZ gibt es genau ϕ(m) Einheiten, nämlich die primen
Restklassen modulo m. (Dies sind die a+mZ mit (a, m) = 1). Die übrigen
Restklassen sind Nullteiler.
Beweis. Sei (a, m) = 1. Nach I.7.8 gilt dann aϕ(m) ≡ 1 mod m, d.h.
(a + mZ)(aϕ(m)−1 + mZ) = aϕ(m) + mZ = 1 + mZ = 1.
Damit ist a + mZ Einheit in Z/mZ.
Sei (a, m) = d > 1; m = dd0 , a = d00 d. Dann gilt ad0 = d00 dd0 = d00 m ≡ 0
mod m und 1 ≤ d0 < m. Also ist d0 + mZ 6= 0, aber (a + mZ)(d0 + mZ) =
ad0 + mZ = 0 + mZ = 0.
2.3 Korollar. Das Produkt aller von Null verschiedenen Elemente von Z/pZ
ist −1. Für alle r ∈ Z/pZ ist rp = r.
Beweis. Nach dem Satz von Wilson ist (p − 1)! ≡ −1 mod p, d.h.
(1 + pZ) · (2 + pZ) · . . . · ((p − 1) + pZ) = (p − 1)! + pZ = −1 + pZ = −1
Sei r = a + pZ. Nach 7.9 gilt ap ≡ a mod p, d.h.
rp = (a + pZ)p = ap + pZ = a + pZ = r.
Polynomringe.
Definition. Sei R ein Ring. Ein Polynom (in einer Unbestimmten X) über
R ist ein Ausdruck
2
n
f = a0 + a1 X + a2 X + . . . + an X =
n
X
i=0
68
ai X i
wobei n ∈ N und ao , . . . , an Elemente aus R sind. Wir setzen noch ai = 0 für
alle i ∈ N mit i > n. Die Elemente ai , i ∈ N nennt man die Koeffizienten
∞
P
von f . Ein Polynom über R ist also ein Ausdruck der Form f =
ai X i mit
i=0
Elementen ai ∈ R, wobei ai 6= 0 nur für endlich viele Indizes i gilt.
Beispiele. 1 + 1 · X + 3 · X 2 und 1 + 0 · X + 0 · X 2 + 1 · X 3 sind Polynome
über Z. Man schreibt dafür auch kürzer 1 + X + 3X 2 bzw. 1 + X 3 , kann also
in einem Polynom Summanden ai X i mit ai = 0 weglassen und X i anstelle
von 1 · X i schreiben.
Definition. Polynome sind gleich, wenn sie die gleichen Koeffizienten haben.
In Formeln:
∞
X
i=0
ai X i =
∞
X
bi X i genau dann, wenn ai = bi für i = 0, 1, 2, . . .
i=0
Auswertung von Polynomen. Sei f =
n
P
ai X i ein Polynom über R und
i=0
b ∈ R. Der Wert von f an der Stelle b ist das Element
n
f (b) := a0 + a1 b + . . . + an b =
n
X
ai bi ∈ R
i=0
Bemerkung. Es kann vorkommen, daß verschiedene Polynome an allen
Stellen von R den gleichen Wert annehmen.
Beispiel. R = Z/2Z = {0, 1}. Die Polynome X, X 2 , X 3 , . . . haben an der
Stelle 0 den Wert 0 und an der Stelle 1 den Wert 1.
Bezeichne die Menge aller Polynome über R mit R[X]. Wir wollen R[X] zu
einem Ring machen, in dem wir eine geeignete Addition bzw. Multiplikation
von Polynomen einführen.
Vorbetrachtung. Seien a0 , . . . , an bzw. b0 , . . . , bm Elemente aus R. Setzt
man noch aj = 0 für j > n und bj = 0 für j > m, so gilt nach den Rechengesetzen für R:
(1)
(a0 + a1 y + . . . + an y n ) + (b0 + b1 y + . . . + bm y m ) =
= (a0 + b0 ) + (a1 + b1 )y + . . . + (al + bl )y l , wenn l = Max (n, m)
69
und
(2)
(a0 + a1 y + . . . + an y n ) · (b0 + b1 y + . . . + bm y m ) =
= c0 + c1 y + . . . + cn+m y n+m , wobei
c0 = a0 b0 , c1 = a0 b1 + a1 b0 , c2 = a0 b2 + a1 b1 + a2 b0 , . . .
ck = a0 bk + a1 bk−1 + . . . + ak−1 b1 + ak b0 , für k = 0, . . . , n + m.
Wegen aj = 0 für j > n und bj = 0 für j > m ist
cn+m = a0 bn+m + . . . + an bm + an+1 bm−1 + . . . + an+m b0 = an bm .
Definiere nun Addition und Multiplikation von Polynomen so, als wäre X
ein Element von R.
∞
X
i
ai X +
i=0
∞
X
!
ai X i
i=0
∞
X
i=0
∞
X
bi X
i
!
bi X i
Insbesondere ist
n
P
i=0
ai X
:=
i=0
m
P
i
bi X i
i=0
∞
X
(ai + bi )X i
ck X k , wobei
k=0
ck = a0 bk + a1 bk−1 + . . . + ak−1 b1 + ak b0
:=
∞
X
=
k
X
ai bk−i .
i=0
= c0 + c1 X + . . . + cn+m X n+m mit
i=0
c0 = a0 b0 , c1 = a0 b1 + a1 b0 , und cn+m = an bm (siehe (2)). Damit wird R[X]
zu einem Ring mit Eins = 1 und Null = 0. Durch Vergleich mit (1) und (2)
sieht man:
2.4 Bemerkung. Sind f, g Polynome aus R[X] und ist y ∈ R, so gilt
(f + g)(y) = f (y) + g(y) und (f g)(y) = f (y) · g(y).
Offenbar ist R ⊆ R[X] ein Unterring (bestehend aus den konstanten Polynomen a = a + 0 · X + 0 · X 2 + . . . , a ∈ R).
∞
P
Das konstante Polynom 0 =
0 · X i heißt auch Nullpolynom.
i=0
Definition. Sei f = a0 + a1 X + . . . + an X n , n ≥ 0, an 6= 0, ein von 0
verschiedenes Polynom. Dann nennt man n den Grad von f und an den
Leitkoeffizienten von f .
2.5 Bemerkung. Sei R ein Integritätsbereich. Dann gilt
70
a) R[X] ist ebenfalls ein Integritätsbereich.
b) Sind f, g ∈ R[X] von Null verschiedene Polynome, so ist
Grad f g = Grad f + Grad g.
c) Die Einheiten von R[X] sind die Einheiten von R.
Beweis.
a) b) Seien f = a0 +a1 X +. . .+an X n 6= 0 und g = b0 +b1 X +. . .+bm X m 6= 0
mit n ≥ 0, m ≥ 0, an 6= 0, bm 6= 0.
Dann ist f g = c0 + c1 X + . . . + cn+m X n+m , cn+m = an bm . Da R integer
ist, gilt cn+m = an bm 6= 0. Es folgt f g 6= 0 und Grad f g = n + m =
Grad f + Grad g.
c) Sei f ∈ R[X] eine Einheit. Dann gibt es ein g ∈ R[X] mit f g = 1.
Es folgt Grad f + Grad g = Grad f g = Grad 1 = 0 und daher Grad
f = Grad g = 0, d.h. f = a0 , g = b0 und f g = a0 b0 = 1. Es folgt
f = a0 ∈ R× . Umgekehrt ist jedes konstante Polynom f = a0 mit
a0 ∈ R× in R[X] eine Einheit.
Division von Polynomen mit Rest. Sei K ein Körper.
2.6 Satz. Seien f und g Polynome aus K[X], g 6= 0. Dann gibt es eindeutig
bestimmte Polynome q, r ∈ K[X] mit
(i)
(ii)
f =g·q+r
r = 0 oder r 6= 0 und Grad r < Grad g.
Beweis. Existenz: Es ist f = g · 0 + f . Also ist die Menge {r0 ∈ K[X] | Es
gibt ein q 0 ∈ K[X] mit f = g · q 0 + r0 } = M nicht leer (f ∈ M ).
1. Fall. Ist 0 ∈ M , so ist f = gq + 0 und wir sind fertig.
2. Fall. Sei 0 6∈ M . Dann hat jedes r0 ∈ M einen Grad.
Wähle ein r ∈ M von kleinstmöglichem Grad. Es gibt dann nach Definition
von M ein q ∈ R[X] mit
f = gq + r.
Noch zu zeigen: Grad r < Grad g
71
Angenommen n := Grad r ≥ m := Grad g, r =
n
P
ai X i und g =
i=0
m
P
bi X i .
i=0
n−m
Dann hat das Polynom an b−1
g den Grad (n − m) + m = n = Grad r
m X
und den Leitkoeffizienten
an b−1
= an , also r = an X n + an−1 X n−1 + . . . und
m bm
n−m
−1
= an X n + a0n−1 X n−1 + . . .
a n bm bm X
n−m
Setze r0 := r − an b−1
g = (an−1 − a0n−1 )X n−1 + niedriger Terme. Ferner
m X
ist
n−m
f = gq + r = g(q + an b−1
) + r0 und daher r0 ∈ M , also r0 6= 0. Wir
m X
haben also ein r0 ∈ M gefunden mit Grad r0 ≤ n − 1 < n = Grad r, im
Widerspruch zur Minimalität von Grad r.
Eindeutigkeit. Angenommen f = gq + r = gq 0 + r0 mit Grad r < Grad g
oder r = 0, und Grad r0 < Grad g oder r0 = 0.
Es folgt g(q − q 0 ) = r0 − r.
Wäre q 6= q 0 , so wäre r0 − r = g(q − q 0 ) 6= 0 und daher Grad (r0 − r) = Grad
g+ Grad (q − q 0 ) ≥ Grad g, im Widerspruch zur Wahl von r und r0 .
Also ist q = q 0 , somit auch r0 − r = g · 0 = 0, also r0 = r.
Rechenbeispiele. Betrachte die Polynome f = X 3 + X 2 − 2X − 2 und
g = X2 + 1
(i) als Polynome in Q[X]
(ii) als Polynome in Z/3Z[X]
Zu (i):
(X 3 + X 2 − 2X − 2) : (X 2 + 1) = X + 1
X 2 − 3X − 2
−3X − 3
=⇒ (X 3 + X 2 − 2X − 2) : (X 2 + 1) = X + 1 Rest −3X − 3, d.h.
X 3 + X 2 − 2X − 2 = (X 2 + 1)(X + 1) − 3X − 3
Zu (ii): Mithilfe der obigen Verknüpfungsstabellen für Z/3Z erhält man
(X 3 + X 2 − 2X − 2) : (X 2 + 1) = X + 1
X3
+ X
—————————–
X2
+1
72
da in Z/3Z gilt: 3 = 0 und 1 = −2
=⇒ (X 3 + X 2 − 2X − 2) : (X 2 + 1) = X + 1, d.h.
X 3 + X 2 − 2X − 2 = (X 2 + 1)(X + 1)
73
§3 Gruppen
Definition. Eine Gruppe ist eine Menge G zusammen mit einer Verknüpfung
◦, so daß gilt: (Axiome)
(1) Zu je zwei Elementen a, b ∈ G gibt es ein eindeutig bestimmtes Element
a ◦ b ∈ G (Eindeutige Ausführbarkeit)
(2) Für alle a, b, c ∈ G gilt
(a ◦ b) ◦ c = a ◦ (b ◦ c)
(Assoziativgesetz)
(3) Es gibt ein Element e ∈ G, so daß für alle a ∈ G
a◦e=a
(Neutrales Element)
(4) Zu jedem a ∈ G gibt es ein a0 ∈ G mit
a ◦ a0 = e
(Existenz des Inversen)
Die Gruppe G heißt kommutativ (oder abelsch), wenn
(5)
a◦b=b◦a
(Kommutativgesetz)
für alle a, b ∈ G.
Wie bei Ringen zeigt man: Neutrales Element und inverses Element zu a sind
eindeutig bestimmt.
Beispiele.
a) (Z, +) ist eine abelsche Gruppe, 0 ihr neutrales Element.
b) Ist V ein Vektorraum, so ist (V, +) eine abelsche Gruppe.
c) Ist K ein Körper (etwa Q oder Z/pZ), so ist (K\{0}, ·) eine abelsche
Gruppe.
d) Ist R ein Ring, so ist (R, +) eine abelsche Gruppe. Insbesondere ist
(Z/mZ, +) eine abelsche Gruppe mit m Elementen.
e) Ist R ein Ring, so ist (R× , ·) eine abelsche Gruppe, die sogenannte
Einheitengruppe von R.
74
Insbesondere ist ((Z/mZ)× , ·) eine abelsche Gruppe mit ϕ(m) Elementen,
die sogenannte prime Restklassengruppe modulo m.
Beweis. a) bis d) sind klar.
Zu e). Nach 1.2 gilt: 1 ∈ R× ist neutrales Element.
a ∈ R× =⇒ a−1 existiert und a−1 ∈ R×
a, b ∈ R =⇒ ab ∈ R× .
Assoziativ- und Kommutativgesetz gelten schon in R.
Nach 2.2 besteht (Z/mZ)× aus ϕ(m) Elementen.
Konventionen und Schreibweisen. Gewöhnlich wählen wir als Verknüpfungszeichen unserer Gruppe den Malpunkt und lassen diesen meistens sogar weg
(ab bedeutet a · b). Das neutrale Element wird dann mit 1 (oder e) bezeichnet
und das zu a inverse mit a−1 .
Mit l > 0 aus Z verwenden wir abkürzend
a0 = e, a1 = a, a2 = aa, . . . , al = a
· · a}, . . .
| ·{z
l−mal
a−2 := (a−1 )2 = (a2 )−1 , . . . , a−l := (a−1 )l = (al )−1 , . . .
Durch Induktion zeigt man für alle a, b ∈ G und l, m ∈ Z:
al am = al+m und (al )m = alm (nachprüfen).
Zusätzlich gilt, wenn G abelsch ist: (ab)l = al bl .
Die vorgestellte Notation nennt man die multiplikative Schreibweise.
Additive Schreibweise. Ist + das Verknüpfungszeichen von G, so wird das
neutrale Element mit 0 bezeichnet und das zu a inverse mit −a. Ferner sei
für l > 0 aus Z
0 · a := 0, 1 · a := a, 2 · a := a + a, . . . , l · a := a
· + a}.
| +{z
l−mal
(−2) · a := (−a) + (−a) = −(a + a), . . . , (−l) · a := l · (−a) = −(l · a).
Es folgt: l · a + ma = (l + m)a, l · (m · a) = (l · m) · a.
Die additive Schreibweise wird nur für abelsche Gruppen verwandt, es gilt
somit auch l · (a + b) = l · a + l · b.
75
Untergruppen. Sei (G, ·) eine Gruppe.
Definition. Eine Teilmenge H von G heißt Untergruppe von G, wenn H
bzgl. der Verknüpfung von G selbst eine Gruppe ist, d.h. wenn
1∈H
aus a, b ∈ H bereits ab ∈ H folgt und
für a ∈ H auch a−1 zu H gehört .
Beispiele.
a) G ist eine Untergruppe von G, ebenso {1}.
b) Sei a ∈ G beliebig. Setze
< a >:= {an | n ∈ Z}
Wegen a0 = 1, al am = al+m und (an )−1 = a−n ist < a > eine Untergruppe
G. Sie heißt die von a erzeugte Untergruppe von G. Wird G = (G, +)
additiv geschrieben, so ist natürlich < a >= {na | n ∈ Z}.
Beispielsweise ist in G = (R, +)
Z = {1 · n | n ∈ Z} =< 1 >
und in (Z, +) für jedes m ∈ Z
mZ = {m · n | n ∈ Z} =< m >.
Zyklische Gruppen.
Definition. Eine Gruppe heißt zyklisch, wenn es ein a ∈ G gibt, so daß
G =< a >. In diesem Fall heißt a ein erzeugendes Element (Generator)
von G.
Beispiele.
a) (Z, +) ist zyklisch mit Generator 1.
b) Die Menge der bijektiven Selbstabbildungen von R2 bilden eine Gruppe
S(R2 ) mit neutralem Element id = identische Abbildung.
c) Für α ∈ R sei dα die Drehung der Ebene mit Mittelpunkt 0 um den
Winkel α, gemessen im Uhrzeigersinn. G = {dα | α ∈ R} ⊆ S(R2 )
ist eine Gruppe, denn: d0 = id, dα ◦ dβ = dα+β , dα ◦ d−α = dα−α =
d0 , dα ◦ d0 = dα .
und a = dα . Zur Veranschaulichung zeichnen wir eine Uhr
d) Sei α = 2π
12
mit Mittelpunkt 0 ∈ R2 und Radius 1.
76
Die Anwendung von a bedeutet, daß die Uhr um eine Stunde vorgestellt
wird. Für k ≥ 1 stellt ak = a
. . ◦ a} die Uhr um k Stunden vor, a−1
| ◦ .{z
k−mal
bedeutet die Rückstellung um eine Stunde. Offenbar ist a12 = d2π die
identische Abbildung. Es folgt für k ∈ Z:
ak = ar mit 0 ≤ r ≤ 11, wenn r der Divisionsrest von k modulo 12 ist:
k = q · 12 + r =⇒ aq·12+r = aq·12 · ar = (a12 )q · ar = (id)q · ar = ar . Damit
ist gezeigt
U =< a >= {id, a, a2 , . . . , a11 } ist eine zyklische Gruppe mit 12 Elementen.
e) Sei m > 1 ganz. Dann ist Z/mZ = {mZ, 1 + mZ, . . . , (m − 1) + mZ}
bezüglich der Addition eine zyklische Gruppe der Ordnung m mit Generator 1 + mZ, denn
n(1+ mZ) = (1+mZ)+. . .+(1+ mZ) = n +mZ, für n = 1, . . . , m −1.
f) Betrachte die Einheitengruppe von Z/5Z. Nach 2.2 gilt G = (Z/5Z)× =
{1 + 5Z, 2 + 5Z, 3 + 5Z, 4 + 5Z}. G ist zyklisch mit Erzeuger a = 2 + 5Z :
a = 2 + 5Z, a2 = 4 + 5Z,a3 = 8 + 5Z = 3 + 5Z, a4 = a · a3 =
(2 + 5Z)(3 + 5Z) = 6 + 5Z = 1 + 5Z.
e) Betrachte die Einheitengruppe von Z/8Z. Nach 2.2 ist
(Z/8Z)× = {1 + 8Z, 3 + 8Z, 5 + 8Z, 7 + 8Z}
(Z/8Z)× ist nicht zyklisch, denn: < 1 + 8Z >= {1 + 8Z}
(3 + 8Z)2 = 1 + 8Z =⇒< 3 + 8Z >= {1 + 8Z, 3 + 8Z}
(5 + 8Z)2 = 1 + 8Z =⇒< 3 + 8Z >= {1 + 8Z, 5 + 8Z} und
(7 + 8Z)2 = 1 + 8Z =⇒< 7 + 8Z >= {1 + 8Z, 7 + 8Z}.
77
Im Verlauf soll gezeigt werden: (Z/pZ)× ist zyklisch für Primzahlen p.
Fortan sei G eine (multiplikativ geschriebene) endliche abelsche Gruppe. Die
Anzahl der Elemente von G nennt man die Ordnung von G; schreibe dafür
ord G.
Beispiel. ord (Z/mZ)× = ϕ(m). Wir haben gesehen: aϕ(m) = 1, also aordG =
1 für alle a ∈ (Z/mZ)× = G.
Allgemein gilt:
3.1 Satz. Für alle a ∈ G ist aordG = 1.
Beweis. Sei m = ord G, G = {a1 , . . . , am }. Ist a ∈ G beliebig, so gilt nach
der Kürzungsregel: aa1 , . . . , aam sind paarweise verschieden (aai = aaj =⇒
ai = a−1 (aai ) = a−1 (aaj ) = (a−1 a)aj = aj ). Also ist G = {aa1 , . . . , aam }. Es
folgt:
Y
a1 · . . . · am =
g = (aa1 ) · . . . · (aam ) = am (a1 · . . . · am )
g∈G
Kürzen ergibt: 1 = am .
Ist G endlich, ord G = m, so ist nach 3.1 {ν | aν = 1, ν ≥ 1} nicht leer, also
existiert Min {ν | aν = 1, ν ≥ 1}. Diese Zahl nennt man die Ordnung von
a; schreibe dafür auch ord a.
3.2 Satz. Es ist ord < a >= ord a und < a >= {1, a, a2 , . . . , ad−1 }, wenn
d = ord a.
Beweis. Sei d = ord a. Schreibe n ∈ Z in der Form n = qd + r,
0 ≤ r ≤ d − 1, q ∈ Z. Dann ist an = aqd ar = (ad )q ar = 1 · ar = ar , somit
H :=< a >= {1, a, a2 , . . . , ad−1 } und ord < a >≤ ord a. Andererseits gilt
nach 3.1 aordH = 1, also auch ord a ≤ ord H nach Definition von ord a.
Insbesondere gilt:
Ist 0 ≤ i < j ≤ d − 1, so ist ai 6= aj und < a > besteht
aus den d paarweise verschiedenen Elementen
1, a, a2 , . . . , ad−1
3.3 Korollar. Für jedes a ∈ G ist ord a ein Teiler von ord G.
78
Beweis. Nach 3.1 ist aordG = 1. Zeige nun allgemeiner:
Aus an = 1 folgt d := ord a | n.
Beweis. Schreibe n = qd + r, 0 ≤ r ≤ d − 1 =⇒ 1 = an = (ad )q ar = ar =⇒
r = 0 (nach dem Beweis von 3.2); also ist n = qd, d.h. d | n.
3.4 Satz. Ist G zyklisch, so ist auch jede Untergruppe H von G zyklisch,
und ord H | ord G.
Beweis. Sei ord G = d, also G =< a >= {1, a, . . . , ad−1 }. Wegen {1} =< 1 >
können wir o.E. H 6= {1} annehmen. Sei t = Min {s | s ≥ 1 und as ∈ H}.
Zeige nun, daß H =< at >.
Bew. Sei as ∈ H, as 6= 1, 1 ≤ s ≤ d − 1. Es folgt s ≥ t.
Noch zu zeigen: t | s, somit s = t · q und as = (at )q ∈< at >.
Angenommen t - s : s = tq + r, 1 ≤ r ≤ t − 1 =⇒ as = (at )q ar =⇒ ar =
as (at )−q ∈ H, da as ∈ H und at ∈ H.
ar ∈ H mit 1 ≤ r < t steht aber im Widerspruch zur Minimalität von t.
Nach 3.2 und 3.3 ist schießlich wegen H =< at >
ord H = ord at | ord G.
79
§4 Die prime Restklassengruppe modulo p
Sei K ein Körper. Wir wollen zeigen:
4.1 Satz. Jede endliche Untergruppe von K × ist zyklisch. Insbesondere ist
(Z/pZ)× zyklisch, wenn p eine Primzahl ist.
Dazu betrachten wir den Polynomring K[X]. Sei f ∈ K[X].
Definition. y ∈ K heißt Nullstelle von f , wenn f (y) = 0.
4.2 Lemma. Ist y eine Nullstelle von f , so gibt es ein g ∈ K[X] mit
f = g · (X − y)
Beweis. Nach 2.6 gibt es Polynome g, r ∈ K[X] mit f = g · (X − y) + r,
wobei r = 0 oder Grad r < Grad (X − y) = 1.
In jedem Fall ist r ∈ K. Setze y für X ein und erhalte 0 = f (y) = g(y) · (y −
y) + r = 0 + r = r, also f = g · (X − y).
4.3 Korollar. Sind y1 , . . . , ym paarweise verschiedene Nullstellen von f , so
gibt es ein g ∈ K[X] mit
f = (X − y1 ) · . . . · (X − ym ) · g
Beweis. (Induktion nach m). m = 1 gilt nach 4.2.
Schluß von m − 1 auf m. Nach Induktionsannahme gibt es ein h ∈ K[X]
mit
f = (X − y1 ) · . . . · (X − ym−1 ) · h
Werte beide Seiten an der Stelle X = ym aus:
0 = f (ym ) = (ym − y1 ) · . . . · (ym − ym−1 ) h(ym ). Es folgt h(ym ) = 0.
{z
}
|
6=0
Nach 4.2 gibt es ein g mit h = (X − ym ) · g und somit
f = (X − y1 ) · . . . · (X − ym−1 )(X − ym ) · g
4.4 Korollar. Sei f 6= 0 ein Polynom vom Grad n ≥ 0. Dann hat f höchstens
n Nullstellen.
80
Beweis. Seien y1 , . . . , ym paarweise verschiedene Nullstellen von f . Nach 4.3
gibt es ein Polynom g mit
f = (X − y1 ) · . . . · (X − ym )g
Es folgt g 6= 0 und Grad g = n − m, also n − m ≥ 0 und m ≤ n.
4.5 Korollar. Sei K ein unendlicher Körper (etwa K = R) und f, g Polynome aus K[X]. Ist dann f (a) = g(a) für alle a ∈ K, so ist f = g.
Beweis. f −g ist ein Polynom h mit h(a) = f (a)−g(a) = 0 für alle (unendlich
vielen) a ∈ K. Nach 4.4 muß dann h = 0 sein, d.h. f = g.
Anmerkung. Im Körper K = Z/pZ gilt nach Fermat ap = a für a ∈ K.
Also gilt für das Polynom f = X p − X 6= 0 : f (a) = ap − a = 0 für alle
a ∈ K.
Zum Beweis von 4.1 benötigen wir einen weiteren Hilfssatz.
4.6 Lemma. Sei G eine endliche abelsche Gruppe. Es gelte
(∗)
| {x ∈ G | xd = 1} |≤ d für alle d | ord G
Dann ist G zyklisch.
Schließe zunächst 4.1 aus 4.6. Sei G ⊆ K × eine endliche Untergruppe.
Nach 4.4 gilt: Ist d | ord G, so hat X d − 1 höchstens d Nullstellen in K, also
auch in G. Also ist (∗) erfüllt und G ist zyklisch nach 4.6.
Beweis von 4.6. Jedes a ∈ G eine wohlbestimmte Ordnung d ≥ 1, und nach
3.3 ist d | m := ord G. Setze
Gd := {x ∈ G | ord x = d}.
Zeige, daß Gm 6= ∅. Dann gibt es ein x ∈ G mit ord x = m und daher
ord < x >= ord x = ord G nach 3.2, d.h. < x >= G.
Wegen ord a | ord G (nach 3.3) gilt
G=
[
Gd
d|m
Setze ψ(d) := |Gd |. Wegen (1) gilt:
81
(1)
m=
X
ψ(d)
(2)
Ferner gilt nach der Teilersummenformel
X
m=
ϕ(d)
(3)
d|m
d|m
Zeige noch
ψ(d) ≤ ϕ(d) für alle d | m
(4)
Wenn (4) gezeigt ist, folgt zusammen mit (2) und (3)
X
X
ϕ(d) und ψ(d) ≤ ϕ(d) für alle d | m
ψ(d) =
d|m
d|m
Dann muß aber ψ(d) = ϕ(d) sein für alle d | m. Insbesondere ist dann
ψ(m) = ϕ(m) 6= 0 und Gm =
6 ∅.
Beweis von (4). Im Fall ψ(d) = 0 ist nichts zu zeigen. Sei also ψ(d) ≥ 1.
Dann existiert ein a ∈ G mit ord a = d. Dann ist ord < a >= d und es gilt
xd = 1 für alle x ∈ H =< a > (nach 3.2 und 3.1). Es ist also ord H = d und
H ⊆ H̃ = {x ∈ G | xd = 1}. Nach Voraussetzung (∗) ist aber ord H̃ ≤ d,
somit H = H̃ und H = {a, a2 , . . . , ad = 1} = {x ∈ G | xd = 1}. Insbesondere
ist Gd ⊆ H.
n
Für alle n mit 1 ≤ n ≤ d und (n, d) = t > 1 gilt x = a t ∈ H, somit
n
d
(an ) t = (a t )d = xd = 1, also ord an ≤ dt < d, also an 6∈ Gd . Es folgt Gd ⊆
{an | 1 ≤ n ≤ d und (n, d) = 1}. Also gilt ψ(d) = |Gd | ≤| {n | 1 ≤ n ≤ d
und (n, d) = 1} |= ϕ(d).
Wir haben also gesehen:
4.7 Korollar. Die multiplikative Gruppe K × eines endlichen Körpers ist
zyklisch.
4.8 Korollar. Die prime Restklassengruppe modulo p ist zyklisch von der
Ordnung p − 1.
Definition. Eine Zahl m ∈ Z heißt Primitivwurzel modulo p, wenn p - m
und m + pZ ein Generator der primen Restklassengruppe modulo p ist.
82
4.9 Korollar. Es gibt genau ϕ(p − 1) Primitivwurzeln modulo p, die paarweise inkongruent modulo p sind.
Beweis. G = (Z/pZ)∗ erfüllt wegen 4.4 die Voraussetzung (∗) von 4.6. Im
Verlauf des Beweises von 4.6 haben wir gezeigt: Die Menge Gd = {x ∈ G |
ord x = d} besteht aus ϕ(d) Elementen, wenn d | ord G. Insbesondere besteht
Gp−1 = {x ∈ G | ord x = p − 1} aus ϕ(p − 1) Elementen.
Wegen ord x = ord < x > sind die Elemente von Gp−1 gerade die Erzeuger
von G.
Nach 2.3 ist das Produkt der von 0 verschiedenen Elemente von Z/pZ gleich
−1. Wir zeigen noch, daß die entsprechende Aussage für jeden endlichen
Körper gilt.
Q
4.10 Satz. Ist K ein endlicher Körper, so ist
a = −1.
a∈K ×
Beweis. Sei q = ord K, d.h. q − 1 =ord K × . Nach 3.1 ist aq−1 = 1 für alle
a ∈ K × ; m.a.W.: Jedes a ∈ K × ist Nullstelle des Polynoms f = X q−1 − 1.
Sei K × = {y1 , . . . , yq−1 }. Nach 4.3 gilt dann mit einem c ∈ K
f = X q−1 − 1 = (X − y1 ) · . . . · (X − yq−1 ) · c
= X q−1 · c + . . . + (−y1 )(−y2 ) · . . . · (−yq−1 ) · c
Es folgt durch Koeffizientenvergleich c = 1 und
−1 = (−y1 )(−y2 ) · . . . · (−yq−1 ) = (−1)q−1 y1 · . . . · yq−1 = y1 · . . . · yq−1 , denn
(−1)q−1 = 1 nach 3.1.
83
Kapitel III.
Aufbau des Zahlensystems
§1 Addition und Multiplikation natürlicher
Zahlen
Wir wollen erklären, wie man natürliche Zahlen addiert und multipliziert und
dabei nur den Begriff das Zählens verwenden.
Heuristische Überlegung. Die natürlichen Zahlen entstehen durch fortschreitendes Zählen. Man kann diesen Vorgang beispielsweise anhand einer
Strichliste dokumentieren:
leer, |, ||, |||, . . . , n, n0 , (n0 )0 , . . .
Anstelle von Strichfolgen kann man die Zahlen etwa durch arabische oder
römische Zifferen ausdrücken:
0, 1, 2, 3, 4, 5, 6, 7, 8, 9, 10, 11 . . .
I, II, III, IV, V, V I, V II, V III, IX, X, XI, . . .
Beim Zählen folgt auf eine Zahl genau eine nächste. Nenne diese den Nachfolger; schreibe n0 für den Nachfolger von n. Es gilt:
(α) Verschiedene natürliche Zahlen haben auch verschiedene Nachfolger.
(β) 0 ist kein Nachfolger einer natürlichen Zahl.
(γ) Jede natürliche Zahl wird erreicht, wenn man lange genug“ zählt.
”
In der Sprache der Mengenlehre bedeutet dies:
Definition. Die natürlichen Zahlen bilden eine Menge N, zusammen mit
(1) einem ausgezeichneten Element 0, und
(2) einer Abbildung
S : N −→ N
n 7−→ S(n) =: n0
Es soll gelten (Axiome der natürlichen Zahlen.)
(A) S ist injektiv, d.h.: Aus n 6= m folgt n0 6= m0 .
84
(B) 0 6∈ S(N), d.h.: Für alle n ∈ N ist n0 6= 0.
(C) (Induktionsaxiom.) Sei M ⊆ N eine Menge mit den Eigenschaften
(i) 0 ∈ M ; (ii) Aus x ∈ M folgt x0 ∈ M
Dann ist M = N.
n0 wird als Nachfolger von n bezeichnet.
Alle weiteren Aussagen über natürliche Zahlen und deren Beweise lassen
sich einzig und allein auf diese drei Axiome gründen. Wir werden diese
Rückführung auf die Axiome an Beispielen demonstrieren.
1.1 Satz. Jede von 0 verschiedene Zahl ist Nachfolger einer natürlichen Zahl.
Beweis. Sei M = {0} ∪ S(N). Zeige, daß M = N ist. Es gilt
(i) 0 ∈ M
(ii) Sei x ∈ M ; dann ist x0 = S(x) ∈ S(N) ⊆ M , also x0 ∈ M .
Nach Axiom (C) ist M = N.
I. Addition natürlicher Zahlen.
1.2 Satz. Zu jedem x ∈ N existiert genau eine Funktion ax : N −→ N mit
folgenden Eigenschaften:
(1) ax (0) = x
(2) ax (y 0 ) = ax (y)0 für alle y ∈ N.
Schreibe x + y := ax (y) für alle x ∈ N.
Wegen (1) und (2) gilt: x + 0 = x und x + y 0 = (x + y)0 für alle x, y ∈ N.
Definition. Die gemäß 1.2 eindeutig existierende Verknüpfung
+ : N × N −→ N, (x, y) 7−→ x + y
mit den Eigenschaften x + 0 = x und x + y 0 = (x + y)0 heißt Addition
natürlicher Zahlen.
Beweis von 1.2. Halte x fest. Seien ax , bx : N −→ N Funktionen mit den
Eigenschaften (1) und (2), d.h.
(1) ax (0) = bx (0) = x,
85
(2) ax (y 0 ) = ax (y)0 und bx (y 0 ) = bx (y)0 für alle y, x ∈ N.
Sei M = {y ∈ N | ax (y) = bx (y)}. Zu zeigen: M = N.
(i) ax (0) = x = bx (0), also 0 ∈ M (wegen (1)).
(ii) Sei y ∈ M , d.h. ax (y) = bx (y). Es folgt mit (2):
ax (y 0 ) = (ax (y))0 = (bx (y))0 = bx (y 0 ) also y 0 ∈ M
Nach (C) ist M = N.
Existenz. Sei M = {x ∈ N | Es existiert eine Funktion ax : N −→ N mit
den Eigenschaften (1) und (2) }. Zu zeigen: M = N.
(i) Setze a0 (y) := y. Dann gilt (1) a0 (0) = 0
Also ist 0 ∈ M .
(2) a0 (y 0 ) = y 0 = (a0 (y))0
(ii) Sei x ∈ M und ax : N → N die wegen x ∈ M in der (bereits bewiesenen) Eindeutigkeitsaussage eindeutig bestimmte Abbildung mit (1),
(2). Dann ist ax (0) = x und ax (y 0 ) = (ax (y))0 für alle y ∈ N.
Setze ax0 (y) := (ax (y))0 für alle y ∈ N.
Da ax die Bedingungen (1) und (2) erfüllt, folgt
ax0 (0) = (ax (0))0 = x0
und
0
0
0
ax0 (y ) = (ax (y )) = (ax (y)0 )0 = (ax0 (y))0
Also erfüllt auch ax0 (1) und (2), d.h. x0 ∈ M .
Nach (C) ist (wegen (i) und (ii)) M = N.
1.3 Satz. Für alle x, y, z ∈ N gilt
(x + y) + z = x + (y + z)
x+y = y+x
(Assoziativgesetz)
(Kommutativgesetz)
Beweise nur das Assoziativgesetz. Halte x, y fest. Sei M = {z | (x + y) + z =
x + (y + z)}. Zu zeigen: N = M .
(1)
(1)
(i) (x + y) + 0 = x + y = x + (y + 0), also 0 ∈ M .
86
(2)
(ii) Sei z ∈ M , d.h. (x + y) + z = x + (y + z). Dann ist (x + y) + z 0 =
(2)
(2)
((x + y) + z)0 = (x + (y + z))0 = x + (y + z)0 = x + (y + z 0 ), also z 0 ∈ M .
Nach (C) folgt M = N.
(2)
(1)
Schreibe 1 für 00 . Dann gilt: x + 1 = x + 00 = (x + 0)0 = x0 .
II. Multiplikation natürlicher Zahlen.
1.4 Satz. Zu jedem x ∈ N gibt es genau eine Funktion mx : N → N mit den
Eigenschaften
(3) mx (0) = 0
(4) mx (y 0 ) = mx (y) + x für alle y ∈ N.
Der Beweis verläuft analog zum Beweis von 1.2 und wird daher weggelassen.
Setze x · y := mx (y) für alle x, y ∈ N.
(3) und (4) bedeuten somit
x · 0 = 0 und x · (y + 1) = (x · y) + x für alle x, y ∈ N.
Ferner ist x · 1 = x · (00 ) = x · 0 + x = 0 + x = x + 0 = x.
Definition. Die Verknüpfung
· : N × N −→ N, (x, y) 7−→ x · y
heißt Multiplikation natürlicher Zahlen.
1.5 Satz Für alle natürlichen Zahlen x, y, z gilt
x·y = y·x
(Kommutativgesetz)
x · (y + z) = (x · y) + (x · z) (Distributivgesetz)
x · (y · z) = (x · y) · z
(Assoziativgesetz)
Beweise nur das Distributivgesetz. Halte x, y fest und setze
M := {z | x · (y + z) = (x · y) + (x · z)}. Zu zeigen: M = N
(i) x · (y + 0) = x · y = (x · y) + 0 = (x · y) + (x · 0), also 0 ∈ M.
(ii) Sei z ∈ M , d.h. x · (y + z) = (x · y) + (x · z). Es folgt
x · (y + z 0 ) = x · ((y + z)0 ) = (x · (y + z)) + x = ((x · y) + (x · z)) + x =
(x · y) + ((x · z) + x) = (x · y) + (x · z 0 ), also z 0 ∈ M .
87
Nach (C) gilt daher M = N.
Konvention. Wir lassen künftig den Malpunkt weg und schreiben kurz
x+y+z
xyz
xy + z
z + xy
für
für
für
für
x + (y + z) = (x + y) + z,
x(yz) = (xy)z;
(xy) + z
z + (xy) (Punktrechnung vor Strichrechnung).
III. Der Rekursionssatz.
Bei der Definition von Addition und Multiplikation sind wir nach dem folgenden Schema vorgegangen:
(i) Man definiert ax (0) bzw. mx (0).
(ii) Man gibt an, wie ax (y 0 ) bzw. mx (y 0 ) aus ax (y) bzw. mx (y) zu berechnen
ist.
Diese Vorgehen nennt man rekursive (induktive) Definition. Sie funktioniert ganz allgemein:
1.6 Rekursionssatz. (ohne Beweis.) Sei A eine Menge, g : A −→ A eine
Abbildung und α ∈ A ein Element. Dann gilt es genau eine Funktion
f : N −→ A mit folgenden Eigenschaften:
(i) f (0) = α; (ii) f (n0 ) = g(f (n)) für alle n ∈ N.
(f (0) = α, f (1) = g(f (0)) = g(α), f (2) = g(f (1)) = g(g(α), f (3) = g(g(g(α))), . . .).
Beispiele.
a) A = N, g(a) = a0 , α ∈ N : f (n) = aα (n) = α + n
b) A = N, g = ax , α = 0 : f (n) = mx (n) = x · n
Rekursive Folgen. Man nennt eine Abbildung f : N → A auch eine Folge
von Elementen aus A und schreibt auch fn für f (n), (fn )n∈N oder f0 , f1 , f2 , . . .
für f .
Beispiele. f = c : N → N n 7→ c ist die konstante Folge c, c, c, . . ..
f : N → N, n 7→ 2n ist die Folge 1, 2, 4, 8, 16, 32, . . .
Eine Folge f , die wie in 1.6 definiert ist, nennt man eine rekursive Folge.
88
Es wird in diesem Fall eine Abbildung g : A → A und ein α ∈ A vorgegeben
und erklärt
(i) f0 = α ; fn+1 = g(fn )
Man nennt α das Anfangsglied und g eine Rekursionsgleichung für die
Folge f .
1
Beispiel. Sei g : R → R, g(x) = 1+x
; α = 1. Dann ist f0 = 1, fn+1 =
Die Folgenglieder berechnen sich nacheinander als
f0 = 1, f1 =
1
1
1
= , f2 =
1+1
2
1+
1
2
2
1
= , f3 =
3
1+
2
3
1
.
1+fn
3
= ,...
5
Allgemeiner kann man rekursive Folgen definieren, indem man angibt, wie
ein Folgenglied aus den k vorangegangenen berechnet werden soll (k–fache
Rekursion):
(1.6)’Satz. Sei g : Rk → R eine Funktion in k Variablen. Dann wird durch
Vorgabe von f0 , . . . , fk−1 und die Rekursionsvorschrift
fn+k = g(fn , . . . , fn+k−1 ) für n = 0, 1, 2, . . .
eine eindeutig bestimmte Folge definiert.
Beispiel. k = 2, g(x, y) = x + y;
f0 = 0, f1 = 1 : fn+2 = g(fn , fn+1 ) = fn + fn+1 = Summe der beiden
vorangegangenen Folgenglieder. Die Folge beginnt mit
0, 1, 1, 2, 3, 5, 8, 13, 21, 34, 55, 89, 144, . . .
Es handelt sich um die berühmte Fibonacci–Folge.
Das Induktionsaxiom (C) läßt sich auch etwas anders formulieren:
(V) Beweisprinzip der vollständigen Induktion. Sei A = A(n) eine
Aussage über natürliche Zahlen. Es sei bekannt:
(a) (Induktionsbeginn) A(0) ist richtig.
(b) (Induktionsschluß) Aus der Gültigkeit von A(x) (Induktionsannahme)
folgt stets die Gültigkeit von A(x + 1). Dann ist A allgemein (d.h. für
alle n ∈ N) richtig.
Beweis. Sei M := {x | x ∈ N und A(x) gilt}. Zu zeigen: M = N.
89
(i) 0 ∈ M , da A(0) gilt.
(ii) Sei x ∈ M , dann gilt A(x). Nach Voraussetzung gilt dann auch A(x+1).
Also ist x + 1 ∈ M .
Nach (C) ist daher M = N.
IV. Die Anordnung der natürlichen Zahlen.
1.7 Satz. Für alle x, y, z ∈ N gilt:
a) Aus x + y = x + z folgt y = z ( Kürzungsregel“).
”
b) Aus x + y = 0 folgt: x = 0 und y = 0.
Beweis.
a) Zu zeigen: Aus y 6= z folgt x + y 6= x + z.
Halte y, z mit y 6= z fest; setze M = {x | x + y 6= x + z}
Zeige mit Hilfe von (C), daß M = N.
(i) 0 + y = y 6= z = 0 + z, also 0 ∈ M .
(ii) Sei x ∈ M , d.h. x + y 6= x + z =⇒ (x + y)0 6= (x + z)0 nach (A).
Aber (x + y)0 = (y + x)0 = y + x0 und (x + z)0 = (z + x) = z + x0 ;
also ist y + x0 6= z + x0 und somit x0 ∈ M .
Nach (C) folgt M = N.
b) Aus y 6= 0 folgt y = w0 mit w ∈ N nach 1.1, und x + y = x + w0 =
(x + w)0 6= 0 nach (B). Analog schließt man, wenn x 6= 0 ist.
1.8 Korollar. Für x, y ∈ N tritt genau einer der folgenden Fälle ein:
(1) x = y
(2) Es gibt ein u 6= 0 in N mit x = y + u
(3) Es gibt ein v 6= 0 in N mit y = x + v
Beweis. Unvereinbarkeit: Wegen 1.7a) ist y + u 6= y für u 6= 0. Also sind
(1) und (2) unvereinbar. Entsprechend zeigt man dies für (1) und (3). Aus
(2) und (3) folgt x = y + u = (x + v) + u = x + (v + u), also v = u = 0 nach
1.7a) und b); Widerspruch.
90
Eintreffen eines der drei Fälle: Halte x fest, zeige induktiv die Aussage
A(y) : Für x, y gilt (1), (2) oder (3).
Induktionsbeginn: y = 0 : x = y + x =⇒ (1) oder (2) gilt für x, y
Induktionsannahme: Für x, y gilt (1), (2) oder (3).
Schluß von y auf y + 1: Wir unterscheiden zwei Fälle:
a) (1) oder (3) gilt für x, y =⇒ y = x+v, v ∈ N =⇒ y +1 = x+(v +1) =⇒
v + 1 6= 0 nach 1.7b) und (3) gilt für x, y + 1.
1.1
b) (2) gilt für x, y =⇒ x = y + u, u 6= 0 =⇒ u = w0 = w + 1, w ∈ N =⇒
x = (y + 1) + w, w ∈ N =⇒ (1) oder (2) gilt für x, y + 1.
Definition. x < y := y = x + v mit v 6= 0
(Fall (3))
Im Fall (2) ist daher y < x. Aus 1.8 ergibt sich
1.9 Korollar. Für x, y tritt genau einer der Fälle x = y, x < y oder y < x
ein.
Im Fall x < y (sprich x kleiner als y“) schreibt man auch y > x (sprich y
”
”
größer als x“).
Definition.
x ≥ y := x > y oder x = y
x ≤ y := x < y oder x = y
1.10 Korollar. Die Relation ≤“ ist eine lineare Ordnung (oder Total”
ordnung) auf N, d.h.:
(1) x ≤ x
(Reflexivität)
(2) Aus x ≤ y und y ≤ x folgt x = y (Antisymmetrie)
(3) Aus x ≤ y und y ≤ z folgt x ≤ z (Transitivität)
Ist dabei x 6= y oder y 6= z, so ist auch x 6= z.
(4) Es gilt x ≤ y oder y ≤ x.
Beweis. (1), (2) und (4) sind klar nach 1.9.
Zu (3): y = x + v, z = y + w =⇒ z = x + (v + w) =⇒ x ≤ z.
1.7b)
Dabei: v 6= 0 oder w 6= 0 =⇒ v + w 6= 0 =⇒ x < z.
1.11 Korollar. (Monotonie) Für alle x, y, z ∈ N gilt
91
a) Aus x ≤ y folgt x + z ≤ y + z
b) Aus x ≤ y folgt x · z ≤ y · z
Beweis.
a) y = x + u =⇒ y + z = x + u + z = (x + z) + u
b) y = x + n =⇒ yz = (x + u)z = xz + uz
Definition. Sei A ⊆ N nicht leer. Ein Element a0 ∈ A heißt Minimum von
A (oder kleinstes Element von A) wenn a0 ≤ a für alle a ∈ A (Schreibe dann
a0 = MinA).
1.12 Prinzip vom kleinsten Element. Jede nicht leere Menge natürlicher
Zahlen besitzt ein Minimum. Dieses ist eindeutig bestimmt. Die Eindeutigkeit
folgt aus 1.10 (2).
Beweis. 0 ∈ A =⇒ 0 = Min A, denn 0 ≤ n für alle n ∈ N (wg. n = 0 + n).
Sei nun 0 6∈ A. Angenommen A besitze kein Minimum. Setze B := {n | n ∈ N
und n < x für alle x ∈ A}. Es folgt B ∩ A = ∅. Wegen A 6= ∅ folgt B 6= N.
Zeige, daß auch B = N, Widerspruch.
(i) Wegen 0 6∈ A ist 0 < x für alle x ∈ A, also 0 ∈ B.
(ii) Sei n ∈ B. Dann ist n < x für alle x ∈ A. Es folgt n + 1 ≤ x für alle
x ∈ A.
Da A kein Minimum besitzt ist n + 1 6∈ A. Also gilt n + 1 < x für alle x ∈ A,
d.h. n + 1 ∈ B.
Nach (C) gilt daher B = N.
1.13 Satz. N ist nullteilerfrei, d.h.
Aus x 6= 0 und y 6= 0 folgt: x · y 6= 0
Beweis. y = z 0 , z ∈ N (nach 1.1). Also gilt wegen x 6= 0
xy = xz 0 = xz + x 6= 0 nach 1.7b).
92
§2 Der Ring der ganzen Zahlen
Die additive Gruppe der ganzen Zahlen.
Zielvorstellung. Die ganzen Zahlen sollen eine abelsche Gruppe (G, +) bilden, welche N umfaßt und so daß gilt:
(i) Jedes Element aus G schreibt sich in der Form a − b mit a, b ∈ N.
(ii) 0 ∈ N ist das neutrale Element von G.
Es folgt: Sind a, b, c, d ∈ N so gilt
(∗)
a − b = c − d in G ⇐⇒ a + d = c + b in N.
Wir wollen uns eine Gruppe mit diesen Eigenschaften schaffen.
Betrachte die Menge N×N = {(a, b) | a ∈ N und b ∈ N} der Paare natürlicher
Zahlen. Führe auf N × N eine Äquivalenzrelation ein; dabei soll am Ende die
Äquivalenzklasse von (a, b) der Zahl a − b in der zu konstruierenden Gruppe
G entsprechen.
Definition. Die Paare (a, b) und (c, d) aus N × N heißen äquivalent, wenn
a + d = b + c.
Schreibe dann (a, b) ∼ (c, d) “.
”
2.1 Bemerkung. ∼ ist eine Äquivalenzrelation, d.h.
(1) (a, b) ∼ (a, b)
(Reflexivität)
(2) (a, b) ∼ (c, d) =⇒ (c, d) ∼ (a, b) (Symmetrie)
(3) (a, b) ∼ (c, d) und (c, d) ∼ (e, f ) =⇒ (a, b) ∼ (e, f ) (Transitivität)
Beweis. (1) und (2) sind klar.
(3) (a, b) ∼ (c, d) ∼ (e, f ) =⇒ a + d = c + b und c + f = e + d
=⇒ a + d + f = b + c + f = b + d + e. Nach der Kürzungsregel 1.7a) folgt
a + f = b + e, d.h. (a, b) ∼ (e, f ).
Seien a, b ∈ N.
Definition. Die Menge [a, b] := {(c, d) | (a, b) ∼ (c, d)} nennt man die
Äquivalenzklasse von (a, b) modulo ∼.
Also ist [a, b] = [c, d] ⇐⇒ (a, b) ∼ (c, d) ⇐⇒ a + d = c + b.
93
Wir bezeichnen Z := Menge der Äquivalenzklassen modulo ∼
= {[a, b] | (a, b) ∈ N × N} als Menge der ganzen Zahlen.
Heuristische Überlegung. In der zu konstruierenden Gruppe G soll gelten:
s.o
a − b = c − d ⇐⇒ a + d = c + b ⇐⇒ [a, b] = [c, d]. Identifiziere daher a − b
mit [a, b]. Bei der Addition auf G sollte gelten:
(a − b) + (c − d) = (a + c) − (b + d).
Nachdem a − b mit [a, b] identifiziert ist bedeutet die letzte Gleichung: [a, b] +
[c, d] = [a + c, b + d]. Wir erheben dies zur Definition der Addition auf G.
Definition. [a, b] + [c, d] := [a + c, b + d] (Addition ganzer Zahlen.)
Es ist zu zeigen, daß dies eine sinnvolle Definition ist, d.h. [a, b] = [a0 , b0 ] und
[c, d] = [c0 , d0 ] impliziert [a + c, b + d] = [a0 + c0 , b0 + d0 ]. Der Nachweis wird
dem Hörer (Leser) überlassen.
2.2 Satz. (Z, +) ist eine abelsche Gruppe mit Null= [0, 0] und −[a, b] = [b, a].
Beweis. Assoziativ- und Kommutativgesetz übertragen sich sofort von N auf
Z.
[a, b] + [0, 0] = [a + 0, b + 0] = [a, b]
[a, b] + [b, a] = [a + b, a + b] = [0, 0] denn (a + b) + 0 = 0 + (a + b)
Einbettung von N in Z. Betrachte die Abbildung
l : N −→ Z,
a 7−→ [a, 0]
2.3 Satz. Die Abbildung l ist injektiv und mit der Addtion verträglich“,
”
d.h.: l(a + b) = l(a) + l(b). Man kann daher (N, +) als Teilmenge von (Z, +)
auffassen indem man a für [a, 0] schreibt.
Beweis. [a, 0] = [b, 0] =⇒ a + 0 = b + 0 =⇒ a = b, also ist l injektitiv.
[a, 0] + [b, 0] = [a + b, 0 + 0] = [a + b, 0].
2.4 Satz. Fasse N gemäß 2.3 als Teilmenge von Z auf. (Schreibe a für [a, 0].)
Für a, b ∈ N ist dann [a, b] = a − b und es gilt
˙
˙ >0
Z = (−N>0 )∪{0}
∪N
(N>0 := {n ∈ N | n 6= 0}, −M = {−m | m ∈ M } für M ⊆ N.)
Beweis. a − b = [a, 0] + (−[b, 0]) = [a, 0] + [0, b] = [a, b]. Nach 1.8 gilt in N
genau eine der Relationen a = b, a < b, b < a
a = b : [a, b] = [a, a] = [0, 0] = 0
94
a < b : b = a + x, x ∈ N>0 =⇒ [a, b] = [0, x] = −[x, 0] = −x ∈ −N>0
b < a : a = b + y, y ∈ N>0 =⇒ [a, b] = [y, 0] = y ∈ N0
Multiplikation ganzer Zahlen. Nach 2.4 ist [a, b] = a − b.
Heuristische Vorüberlegung. (Z, +, ·) soll ein Ring werden, also muß
nach dem Distributiv- und Assoziativgesetz gelten:
[a, b] · [c, d] = (a − b)(c − d) = (ac + bd) − (ad + bc) = [ac + bd, ad + bc]
. Wir definieren daher:
Definition. [a, b] · [c, d] := [ac + bd, ad + bc]
(Dies ist die Fortsetzung der Multiplikation auf N ⊆ Z :
[a, 0] · [c, 0] = [ac + 0, a · 0 + 0·] = [ac, 0] = ac.)
Wie man leicht nachrechnet, gilt
2.5 Regel. Seien a, b ∈ N. Dann gilt:
a) (−1)a = −a
b) (−a)b = −ab
c) a(−b) = −ab
d) (−a)(−b) = ab
2.6 Satz (Z, +, ·) ist ein nullteilerfreier Ring mit Einselement 1 und Nullelement 0.
Beweis. Die Nullteilerfreiheit gilt nach 1.13 und 2.5. Zeige noch als Beispiel
das Distributivgesetz in Z:
[a, b]([c, d] + [e, f ]) = [a, b][c + e, d + f ] =
[(ac + ae) + (bd + bf ), (ad + af ) + (bc + be)] =
[ac + bd, ad + bc] + [ae + bf, af + be] = [a, b][c, d]+
+[a, b][e, f ]
Die Anordnung der ganzen Zahlen.
Definition. Für x, y ∈ Z sei
x ≤ y :⇐⇒ y − x ∈ N
x < y :⇐⇒ x ≤ y und x 6= y( d.h. y − x ∈ N>0 )
2.7 Satz. ≤ “ ist eine lineare Ordnung auf Z, welche die Ordnung auf N
”
fortsetzt. Sie ist monoton, d.h.
95
(1) Aus x ≤ y folgt x + z ≤ y + z (für alle x, y, z ∈ Z).
(2) Ist z ∈ N so folgt aus x ≤ y schon x · z ≤ y · z.
Beweis. Seien a, b ∈ N. Gilt a ≤ b in N, so ist b = a + x, x ∈ N, also
b−a = x ∈ N, d.h. a ≤ b in Z (und umgekehrt). Also ist ≤“eine Fortsetzung
”
der Ordnung von N auf Z.
Reflexivität. Aus x − x = 0 ∈ N folgt x ≤ x.
Antisymmetrie. x ≤ y und y ≤ x =⇒ y − x ∈ N und x − y ∈ N
2.4
=⇒ y − x ∈ N und y − x = −(x − y) ∈ N =⇒ y − x ∈ N ∩ (−N) = {0}
=⇒ y = x.
Transitivität. Sei x ≤ y und y ≤ z. Dann ist y − x ∈ N und z − y ∈ N
z − x = (z − y) + (y − x) ∈ N =⇒ x ≤ z.
Lineararität. Sei x 6≤ y. Zu zeigen y ≤ x.
2.4
Aus x 6≤ y folgt y − x 6∈ N =⇒ y − x = −a, a ∈ N =⇒
x − y = −(y − x) = −(−a) = a ∈ N =⇒ y ≤ x.
Monotomie.
(1) x ≤ y =⇒ y − x ∈ N =⇒ (y + z) − (x + z) = y − x ∈ N =⇒ x + z ≤ y + z
(2) x ≤ y und z ≥ 0 =⇒ y − x ∈ N und z ∈ N =⇒ (y − x) · z ∈ N, d.h.
yz − xz ∈ N =⇒ xz ≤ yz.
Übungsaufgabe. Für a ∈ Z nennt man
a falls a ≥ 0
|a| :=
−a falls a < 0
den Betrag von a. Zeigen Sie, daß für alle a, b ∈ Z
| − a| = |a|, |a| ≥ 0, |a| = 0 =⇒ a = 0, |ab| = |a||b| und |a + b| ≤ |a| + |b|
Damit ist Kapitel I, §1 neubegründet worden. Wir werden daher hinfort alles
verwenden, was wir in den Kapiteln I und II über die ganzen Zahlen gelernt
haben.
96
§3 Die g–adische Darstellung natürlicher
Zahlen
Wir sind gewöhnt, natürliche Zahlen im Dezimalsystem darzustellen und
mit diesen Darstellungen zu rechnen. Dazu führt man zehn Zeichen (Ziffern)
ein, üblicherweise
0, 1 := 00 , 2 := 10 , 3 := 20 , 4 := 30 , 5 := 40 , 6 := 50 , 7 := 60 , 8 := 70 , 9 := 80 .
Sind a0 , a1 , . . . , an ∈ {0, 1, 2, . . . , 9} solche Ziffern, n ≥ 1 (und ist an 6= 0), so
bedeutet die Ziffernfolge
(∗)
an an−1 · . . . · a1 a0
die Zahl
0
0
an · 9 n + an−1 9 n−1 + · . . . · +a1 · 90 + a0 .
Insbesondere bedeutet die Ziffernfolge 10 die Zahl 1 · 90 + 0 = 90 . Wir wollen
zeigen, daß man mit dieser Ziffernnotation unmißverständlich Zahlen darstellen kann, d.h.:
Jede Zahl z ∈ N hat eine und nur eine Darstellung (∗) mit Ziffern aus
{0, 1, 2, . . . , 9}.
Man spricht von der Dezimaldarstellung der Zahl z.
Hier ist die Grundzahl zehn, man benutzt zehn Zeichen. Es gibt auch Systeme mit mehr oder weniger als zehn Zeichen. Die Babyloner rechneten im
Zwölfersystem.
Die Computer begnügen sich mit zwei Zeichen, 0 und 1; 1101001“ bedeutet
”
im Zweiersystem die Zahl
1 · 26 + 1 · 25 + 0 · 24 + 1 · 23 + 0 · 22 + 0 · 2 + 1 = 64 + 32 + 8 + 1 = 105,
wenn man Sie im Zehnersystem darstellt.
Wir wollen hier jede natürliche Zahl g ≥ 2 als Grundzahl zulassen.
3.1 Satz. Sei g ≥ 2 eine natürliche Zahl. Dann läßt sich jede natürliche Zahl
a eindeutig in der Form
a = an g n + an−1 g n−1 + . . . + a2 g 2 + a1 g + a0
97
schreiben, wobei a0 , . . . , an ∈ N, n ≥ 0, 0 ≤ ni < g und an 6= 0 falls a 6= 0.
Hat man für die ganzen Zahlen z mit 0 ≤ z < g Zeichen (Ziffern, Chriffren)
vereinbart, so schreibt man für a auch die Aneinanderreihung der betreffenden Zeichen für a0 , . . . , an :
an an−1 . . . a1 a0 bedeutet die Zahl
an g n + an−1 g n−1 + . . . + a2 g 2 + a1 g + a0 = a.
Beide Schreibweisen nennt man die g–adische Darstellung von a.
Beweis. Existenz der g–adischen Darstellung. Induktion nach a. Für a = 0
und a = 1 ist dies klar. Sei also a ≥ 2.
Schluß von a − 1 auf a. Die Behauptung sei bereits bewiesen für natürliche
Zahlen b mit 1 ≤ b < a. Zu zeigen: Dann gilt die Behauptung auch für a.
Dividiere dazu a durch g mit Rest:
a = q · g + r,
0 ≤ r < g,
q≥0
Setze a0 := r. Im Falle q = 0 ist die gewünschte Darstellung
a = a0
(n = 0)
Ist q > 0, so ist q < qg ≤ a, da g ≥ 2.
Nach Induktionsannahme besitzt daher q eine g–adische Darstellung
q = q0 + q1 g + · . . . · +qm g m , 0 ≤ qi < g für i = 0, m, qm 6= 0.
Es folgt a = a0 + qg = a0 + q0 g + q1 · g 2 + . . . + qm g m+1 .
Setze a1 = q0 , . . . , am+1 = qm und erhalte die Darstellung
a = a0 + a1 g + . . . + am+1 g m+1 , 0 ≤ aj < g, j = 0, . . . , m + 1, am+1 6= 0.
Eindeutigkeit der Darstellung. a ≥ 1 habe zwei Darstellungen
a0 + a1 g + . . . + an g n = a00 + a01 g + . . . + a0m g m = a, 0 ≤ ak , a0k < g
wobei o.E. m ≥ n ≥ 0, an 6= 0 und a0m 6= 0.
Zeige zunächst, daß m = n ist.
98
Es ist a ≥ g m , a ≥ g n und aν ≤ g − 1. Es folgt
a ≤ (g−1)g n +. . .+(g−1)g+g−1 = (g−1)(g n +g n−1 +. . .+g+1) = g n+1 −1 < g n+1
Angenommen m ≥ n + 1. Dann wäre g n+1 ≤ g m ≤ a < g n+1 , Widerspruch.
Also ist m = n.
Zeige nun, daß aν = a0ν für ν = 0, . . . , n: Andernfalls ist die Menge
M = {ν | 0 ≤ ν ≤ n, aν 6= a0ν } nicht leer und besitzt daher ein Maximum k.
Es folgt
a0 + a1 g + . . . + ak g k = a00 + a01 g + . . . + a0k g k ; 0 ≤ k ≤ n, ak 6= a0k
Es muß dann k > 0 sein, da sonst a0 = a00 und a0 6= a00 gelten würde.
Es folgt
(ak − a0k )g k = (a00 − a0 ) + (a01 − a1 )g + . . . + (a0k−1 − ak−1 )g k−1 mit
|a0ν − aν | ≤ g − 1 für ν = 0, . . . , k. Es folgt
|ak − a0k |g k ≤ |a0k−1 − ak−1 |g k−1 + . . . + |a01 − a1 |g + |a00 − a0 |
≤ (g − 1)(g k−1 + g k−2 + . . . g + 1) = g k − 1 < g k ,
im Widerspruch zu |ak − a0k | ≥ 1, da ak 6= a0k .
Beispiel. Sei a die natürliche Zahl mit der Dezimaldarstellung
a = 7 + 3 · 10 + 5 · 100 + 1 · 1000
Berechnung der 9–adischen Ziffernfolge von a.
Division von a durch 9 mit Rest ergibt
1537 : 9 = 170 Rest 7, d.h.
63
7
1537 = 170 · 9 + 7
170 = 18 · 9 + 8
18 = 2 · 9 + 0
Dividiere 170 mit Rest, usw.
Es folgt
1537 = 170 · 9 + 7 = (18 · 9 + 8)9 + 7 = (2 · 9 · 9 + 8)9 + 7
= 2 · 93 + 0 · 92 + 8 · 91 + 7 · 90 =2087
ˆ
im Neunersystem
Zur Unterscheidung der verschiedenen System kann man einen Index g für
die g–adische Ziffernfolge angeben, also
(1537)10 = (2087)9
99
Neunerprobe und Elferprobe im Dezimalsystem
Sei g > 1 und (an an−1 . . . a1 a0 )g die g–adische Darstellung der Zahl a ≥ 1.
Definition.
a) Die Quersumme von a (bzgl. g) ist die Zahl
Q(a) := a0 + a1 + . . . + an .
b) Die alternierende Quersumme von a (bzgl. g) ist die Zahl
Q0 (a) := a0 − a1 + a2 − + . . . + (−1)n an .
3.2 Satz. a ≡ Q(a) mod (g − 1) und a ≡ Q0 (a) mod (g + 1).
Beweis. Es ist g ≡ 1 mod (g − 1) und g ≡ −1 mod (g + 1).
Nach den Regeln der Kongruenzrechnung gilt also
g ν ≡ 1 mod (g − 1) und g ν ≡ (−1)ν mod (g + 1)
für alle 0 ≤ ν ≤ n.
n
P
aν g ν ergibt sich daraus
Für a =
ν=0
a≡
a≡
n
P
ν=0
n
P
ν=0
aν 1ν = Q(a) mod (g − 1) und
aν (−1)ν = Q0 (a) mod (g + 1)
Sind x ≡ y mod m, so ist m | x ⇐⇒
x ≡ 0 mod m ⇐⇒ y ≡ 0 mod m ⇐⇒ m | y. Aus 3.2 folgt also
3.3 Korollar.
(g − 1) | a ⇐⇒ (g − 1) | Q(a) und
(g + 1) | a ⇐⇒ (g + 1) | Q0 (a)
Speziell gilt für g = 10
3.4 Korollar.
9 | a ⇐⇒
9 | Q(a)
11 | a ⇐⇒ 11 | Q0 (a)
und
Induktiv erhält man 9 | a ⇐⇒ 9 | Qn (a), 11 | a ⇐⇒ 11 | Q0n (a). Für große n
wird Qn (a) bzw. Q0n (a) einstellig und es ist offensichtlich ob 9 | Qn (a) bzw.
11 | Q0n (a).
100
3.5 Korollar.
+
+
Q(a · b) ≡ Q(a) · Q(b) mod (g − 1) und
+
+
Q0 (a · b) ≡ Q0 (a) · Q0 (b) mod (g + 1)
+
Beweis. Nach 3.2 gilt Q(x) ≡ x mod (g − 1) für alle x ∈ N =⇒ Q(a · b) ≡
+
+
a · b ≡ Q(a) · Q(b) mod (g − 1). Entsprechend schließt man für Q0 und
g + 1.
3.6 Korollar. 3 | a ⇐⇒ 3 | Q(a)
Beweis. Offenbar gilt: 3 | x =⇒ 9 | x2 .
Da 3 eine Primzahl ist gilt: 9 | x2 =⇒ 3 | x2 =⇒ 3 | x.
Also gilt: 9 | x2 ⇐⇒ 3 | x für alle x ∈ N.
Nach 3.2 und 3.5 ist a2 ≡ Q(a2 ) ≡ Q(a)2 mod 9, somit
3 | Q(a) ⇐⇒ 9 | Q(a)2 ⇐⇒ 9 | Q(a)2 ⇐⇒ 9 | a2 ⇐⇒ 3 | a
Neunerprobe. Als Resultat einer Multiplikation erhält man a · b = c. Man
möchte überprüfen, ob die Rechnung stimmen kann:
Ist a · b = c“ richtig gerechnet, so gilt nach 3.5 auch Q(a) · Q(b) ≡ Q(c)
”
mod 9. Mit anderen Worten: Aus Q(a)Q(b) 6≡ Q(c) mod 9 folgt ab 6= c, die
Rechnung ist falsch.
Beispiel. Man erhält beim Multiplizieren
1312 · 911 = 1195232
Q(a) = 7, Q(b) = 11, Q(c) = 23
Q(a) · Q(b) = 77 ≡ 5 mod 9, Q(c) ≡ 5 mod 9
Die Rechnung kann also stimmen. (Sie stimmt auch!)
Elferprobe. Wie oben hat man a · b = c“ gerechnet. Ist dies richtig, so gilt
”
auch Q0 (a) · Q0 (b) ≡ Q0 (c) mod 11.
Beispiel. Man erhält (etwa durch einen Schreibfehler)
1312 · 911 = 1105232. Es gilt
Q0 (a) = 3, Q0 (b) = 9, Q0 (c) = −4 ≡ 7 mod 11.
Q0 (a)Q0 (b) = 27 ≡ 5 6≡ 7 ≡ Q0 (c) mod 11.
Die Rechnung muß also falsch sein. (Mit der Neunerprobe hätte man’s nicht
gemerkt.)
101
§4 Die rationalen Zahlen
Der Ring der ganzen Zahlen hat den Mangel, daß nicht jede Gleichung a =
bX, b 6= 0 innerhalb Z lösbar ist. (Z.B. ist 1 = 2 · X unlösbar in Z). Zu seiner Beseitigung erweitert man den Zahlbereich zum Körper der rationalen
Zahlen (Brüche).
A. Die rationalen Zahlen.
Definition. Die Menge der rationalen Zahlen besteht
(1) aus den ganzen Zahlen, und
(2) aus den Paaren (a, b) mit a, b ∈ Z, b ≥ 2, ggT (a, b) = 1
˙
Q = Z∪{(a,
b) | a, b ∈ Z, b ≥ 2, ggT (a, b) = 1}
4.1 Einführung der Bruchschreibweise. Seien a, b ∈ Z, b 6= 0:
a)
a
1
:= a ∈ Z
b) Ist ggT (a, b) = 1 und b ≥ 2, so setzen wir
a
b
:= (a, b).
c) Nach Kap. I, 2.8 gibt es durch (a, b) eindeutig bestimmte Zahlen
d = ±ggT (a, b), ã und b̃ mit a = dã, b = db̃, b̃ ≥ 1, ggT (ã, b̃) = 1, d 6= 0.
(
!
ã
falls b̃ = 1
a
ã
Setze b := b̃ =
. Damit ist
(ã, b̃) falls b̃ ≥ 2
a
Q = { | a, b ∈ Z, b 6= 0}.
b
Für a, b ∈ Z mit b 6= 0, nennt man den Ausdruck ab einen Bruch, a seinen
Zähler und b seinen Nenner. Nach obiger Definition schreibt sich jedes
r ∈ Q in eindeutiger Weise als r = ãb̃ mit b̃ ≥ 1 und ggT (ã, b̃) = 1. Dieser
Ausdruck heißt gekürzte Bruchdarstellung von r ∈ Q. Sind also ãb̃ und dc̃˜
gekürzte Brüche, so gilt
(∗)
c̃
ã
= ⇐⇒ ã = c̃ und b̃ = d˜
b̃
d˜
102
4.2 Gleichheit von Brüchen. Seien a, b ∈ Z, b 6= 0.
b
b
a)
0
b
= 1 und
=0
b) Für alle t ∈ Z\{0} gilt die Kürzungsregel
at
a
=
bt
b
c) Sind c, d ∈ Z, d 6= 0, so gilt
a
c
= ⇐⇒ ad = bc
b
d
Beweis. a) und b) sind Spezialfälle von c).
Zu c) Seien ãb̃ = ab und dc̃˜ = dc die zugehörigen reduzierten Bruchdarstellungen. Dann gilt
˜ ggT (ã, b̃) = ggT (c̃, d)
˜ = 1,
a = tã, b = tb̃, c = t0 c̃, d = t0 d,
b̃ > 0, d˜ > 0 und tt0 6= 0.
(∗)
a
= c =⇒ ã = c̃ =⇒ ã = c̃, b̃ = d˜ =⇒ ad = tt0 ãd˜ = tt0 b̃c̃ = bc.
b
d
b̃
d˜
ad = bc =⇒ tt0 ãd˜ = tt0 b̃c̃ =⇒ tt0 (ãd˜ − b̃c̃) = 0, tt0 6= 0 =⇒ ãd˜ − b̃c̃ nach 2.6,
d.h. ãd˜ = b̃c̃.
˜ c̃) = 1, d˜ > 0, b̃ > 0. Es folgt ã = c̃ und b̃ = d,
˜
Ferner gilt ggT (ã, b̃) = ggT (d,
da man in Z eine eindeutige Primfaktorzerlegung hat. Also ist
ã
c̃
c
a
= = =
˜
b
d
b̃
d
B. Addition und Multiplikation von Brüchen.
Seien ab , dc ∈ Q. Wir definieren
a
b
+
a
b
·
c
d
c
d
:=
ad+bc
bd
(Addition) und
:=
ac
bd
(Multiplikation)
4.3 Bemerkung. Addition und Multiplikation von rationalen Zahlen sind
unabhängig von der Darstellung definiert, d.h.
Aus
(i)
a
b
ac
bd
=
=
a0
b0
und
a0 c0
;
b0 d0
c
d
=
(ii)
c0
d0
folgt
ad+bc
bd
=
a0 d0 +b0 c0
b0 d0
103
Beweis. Nach 4.2 c) gilt
(iii) ab0 = a0 b und cd0 = c0 d, und somit
ac 4.2b) acb0 d0
= bdb0 d0
bd
=
ab0 cd0
bdb0 d0
a0 bc0 d 4.2b) a0 c0
= b0 d0
bdb0 d0
=
0
0
0 bb0
0
0
0 dbb0
ad+bc 4.2b) (ad+bc)b0 d0
=
= ab ddbdb+cd
= a bddbdb+c
0 d0
0 d0
bd
bdb0 d0
4.4 Regel. Sei ab ∈ Q. Dann gilt
=
bd(a0 d0 +c0 b0 ) 4.2b) a0 d0 +c0 b0
=
bd(b0 d0 )
b0 d0
a) + und · setzen die Addition und Multiplikation von Z auf Q fort.
b)
a b
·
b a
= 1, falls a, b ∈ Z\{0}. Insbesondere ist a· a1 = 1 für alle a ∈ N\{0}.
c)
a
b
löst die Gleichung bX = a.
d)
a
b
·1=
e)
a
b
+
c
b
a
b
=
a
b
und
a+c
;
b
+ 0 = ab .
insbesondere ist
a
b
+
−a
b
=
0
b
=0
Beweis.
a)
a
1
a
1
+ 1b = a·1+b·1
= a+b
=a+b
1·1
1
ab
· 1b = 1·1
= ab
=
ab
1
b)
a
b
·
b
a
=
ab
ba
=
1
1
c) b ·
a
b
=
b
1
a
b
=
d)
e)
·
= 1 nach 4.2
ab
1·b
=
a
1
= a nach 4.2
a
b
a
b
· 1 = ab · 11 = a·1
= ab
b·1
+ 0 = ab + 01 = a·1+0·b
=
b·1
a
b
+
c
b
=
ab+cb
bb
=
(a+c)b
bb
=
a
b
a+c
b
nach 4.2
4.5 Satz. (Q, +, ·) ist ein Körper.
Beweis. Nach 4.4 sind nur noch das Distributiv-, Assoziativ- und Kommutativgesetz zu zeigen. Zeige exemplarisch:
+de
+ade
a c
( + fe ) = ab · cfdf
= acfbdf
und
b d
b(acf +dae)
acbf +bdae
a e
a c
· + b · f = bdbf = b(dbf )
b d
also gilt das Distributivgesetz.
=
acf +ade
,
bdf
104
C. Anordnung der rationalen Zahlen.
Definition. Eine rationale Zahl heißt positiv, wenn sie eine Darstellung
r = ab hat mit ab > 0.
4.6 Bemerkung. Ist r positiv und r = dc , so ist auch cd > 0.
Beweis. Sei r = ab mit ab > 0. Es folgt ad = bc, also (ab)(cd) = b2 c2 =
(bc)2 > 0; wegen ab > 0 folgt auch cd > 0.
Setze P = { ab |
a
b
∈ Q positiv } = Menge der positiven rationalen Zahlen.
4.7 Satz. P ist abgeschlossen bzgl. der Addition und Multiplikation und es
˙
˙
gilt: Q = P ∪{0}
∪(−P
).
Beweis. N>0 ist abgeschlossen bezüglich + und ·. Also ist nach Definition
der Positivität rationaler Zahlen auch P abgeschlossen bezüglich + und ·:
Seien r = ab , s = dc aus P =⇒ rs = ac
, r + s = ad+bc
=⇒ acbd = (ab)(cd) > 0
bd
bd
und (ad + bc)bd = (ab)d2 + (cd)b2 > 0.
Sei ab 6∈ P ∪ {0} =⇒ ab < 0 =⇒ −(a)b = −ab > 0 =⇒
− −a
∈ −P.
b
Sei r = − ab ∈ −P, d.h. ab > 0 und r =
ist daher r 6∈ P ∪ {0}.
−a
b
−a
b
∈ P =⇒
a
b
=
mit (−a)b = −ab < 0. Nach 4.6
Definition. r ≤ s := s − r ∈ P0 := P ∪ {0}.
P0 ist nach 4.7 abgeschlossen bzgl. + und ·.
4.8 Satz. ≤“ ist eine lineare Ordnung auf Q, welche die Ordnung auf Z
”
fortsetzt. Sie ist monoton, d.h.
r≤s
r≤s
=⇒
und
r+t≤s+t
t ≥ 0 =⇒ rt ≤ st
Beweis. a ≤ b in Z ⇐⇒ b − a ∈ N ⇐⇒
Def.
b
1
−
a
1
=
b−a
1
a
Def. 1
∈ P0 ⇐⇒
≤
b
1
in Q.
Damit setze ≤“ die Ordnung auf Z fort.
”
Zeige beispielsweise noch die Transitivität von ≤“ :
”4.7
r ≤ s und s ≤ t ⇐⇒ s − r ∈ P0 und t − s ∈ P0 =⇒ t − r = (s − r) + (t − s) ∈
P0 =⇒ t ≤ r
105
und die Monotomie
bzgl. + “ : r ≤ s =⇒ s − r ∈ P0 =⇒
”
(s + t) − (r + t) = s − r ∈ P0 =⇒ r + t ≤ s + t
4.7
und bzgl. · “ : r ≤ s, t ≥ 0, =⇒ s − r, t ∈ P0 =⇒
”
st − rt = (s − r)t ∈ P0 =⇒ rt ≤ st.
4.9 Das Prinzip des Archimedes. Für alle r, s ∈ P gibt es eine natürliche
Zahl n mit n · r > s.
Beweis. Schreibe r = hp , s = hq mit dem gleichen Nenner h > 0 und p, q > 0.
(Dies erreicht man durch Erweitern.)
Es gilt dann nach 4.8:
nr > s ⇐⇒ (nr)h > sh ⇐⇒ np > q, denn rh = p, sh = q.
Es genügt also, zu zeigen: Zu gegebenen natürlichen Zahlen p 6= 0 und q 6= 0
gibt es eine natürliche Zahl n mit np > q.
Beweis: Wegen p ≥ 1 ist (q + 1)p ≥ (q + 1) · 1 = q + 1 > q.
4.10 Satz. Die Elemente von Q liegen dicht gedrängt“:
”
Für r, s ∈ Q mit r < s liegt immer noch ein t ∈ Q dazwischen, d.h. r < t < s.
Beweis. Setze t := 21 (r + s) : s − t = s − 12 r − 12 s =
= 21 s − 12 r = 21 · (s − r) > 0, da s > r. Also ist s > t
t − r = 12 (r + s) − r = 12 s − 12 r = 12 (s − r) > 0, da s > r und somit s − r > 0.
Also ist auch t > r.
Im nächsten Abschnitt werden wir sehen, daß trotz der Aussage 4.10 die
Menge der rationalen Zahlen in gewissem Sinne lückenhaft“ sind. Um die”
se Lücken zu stopfen, haben die Mathematiker die reellen Zahlen erfunden.
Wir werden im nächsten Paragraphen den Weg nachzeichnen, den der große
Mathematiker Richard Dedekind gegangen ist, um die reellen Zahlen zu begründen.
106
§5 Die reellen Zahlen
Heuristische Vorbetrachtung. Wir zeichnen einen Strahl und veranschaulichen die nicht negativen rationalen Zahlen als Punkte auf diesem Strahl.
0
1
3
1
2
2
3
1
4
3
3
2
5
3
2
3
4
Der Ausgangspunkt wird mit 0 gleichgesetzt. Dann wird ein von 0 verschiedener Punkt markiert und gleich 1 gesetzt. Trage nun die Strecke [0, 1] sukzessive nach rechts ab und erhalte weitere Punkte; bezeichne diese nacheinander
mit 2, 3, 4, . . . .
Für jedes n ∈ N+ teile man die Strecke [0, 1] in n gleiche Teile. Nenne
den ersten Teilpunkt n1 . Marken für die Zahlen m
, m ≥ 2 erhält man durch
n
wiederholtes abtragen der Strecke [0, n1 ]. Das Endergebnis ist der rationale Zahlenstrahl. Durch Spiegelung am Nullpunkt erhält man schließlich die
rationale Zahlengerade. Jede rationale Zahl wird somit durch einen Punkt
der Zahlengerade repräsentiert.
Messen von Strecken. Der Abstand zwischen zwei Punkten der Ebene wird
gemessen, indem man sie von 0 aus auf dem Zahlenstrahl abträgt. Man sagt,
die Strecke habe eine rationale Länge, wenn dabei das Ende auf eine Marke
r ∈ Q fällt.
Unvollständigkeit der rationalen Zahlen. Es gibt Strecken, die keine
rationale Länge haben. Dazu zeichen wir ein Quadrat mit Kantenlänge 1
und tragen die Diagonale von 0 aus ab. Behauptung: Das Ende r markiert
keine rationale Zahl.
107
1
r
r
0
r
1
2
Beweis. Durch Vergleich der Flächen des großen und des kleinen Quadrats
sieht man, dass r2 = 2. Angenommen r ist eine rationale Zahl, r = ab , a > 0
und b > 0 ganz, ggT (a, b) = 1.
Es folgt a2 = 2b2 , d.h. 2 | a2 =⇒ 2 | a =⇒ a = 2c =⇒
4c2 = a2 = 2b2 =⇒ b2 = 2c2 =⇒ 2 | b2 =⇒ 2 | b, im Widerspruch zu
ggT (a, b) = 1.
Fazit. Mit dem rationalen Strahlenstrahl kann man nicht jede Strecke exakt
messen.
Zur Behebung dieses Mangels hatte Dedekind folgende Idee:
Man zerschneide die Zahlengerade auf alle erdenkliche Weisen“ in zwei
”
Stücke s und s, auch an allen Stellen, die nicht durch rationale Zahlen markiert sind. Die reellen Zahlen sind dann gerade die Gesamtheit aller so entstandenen Paare (s, s). Nun hat man unendlich viele Maßstäbe, mit denen
alles meßbar ist.
Definition. Sei β ⊆ Q eine Menge rationaler Zahlen. Das Paar (Q \β, β)
[bzw. die Menge β] heißt ein Dedekindscher Schnitt, wenn gilt
108
(D1) β 6= ∅ und Q \β 6= ∅
(D2) Aus r ∈ β und s > r folgt s ∈ β
(D3) β hat kein Minimum.
Q\β
β
Die Menge aller Dedekindschen Schnitte bezeichnen wir mit R.
Beispiele von Dedekindschen Schnitten.
a) Rationale Schnitte. Sei s ∈ Q. Dann ist nach 4.10
β = s := {r ∈ Q | r > s} ein Dedekindscher Schnitt.
Behauptung. Aus s 6= t, s, t ∈ Q folgt s 6= t.
Beweis. Ohne Einschränkung sei t > s. Dann ist t ∈ s und t 6∈ t, also
s 6= t.
Damit hat man eine injektive Abbildung
Q ,→ R s 7→ s = {r ∈ Q | r > s}
Fasse vermöge der Zuordnung s 7→ s die Menge Q als Teilmenge von
R auf; lasse den unteren Strich künftig auch weg.
√
b) Der Dedekindsche Schnitt 2: Betrachte die Menge
β := {r | r ∈ Q, r > 0 und r2 > 2}
Wir werden später sehen, daß β 2 = 2“ (6.7).
”
Behauptung 1. β ist ein Dedekindscher Schnitt.
Beweis. (D1) ist klar.
(D2) Sei r ∈ β und s ∈ Q mit s > r =⇒ s2 > r2 > 2, also s ∈ β.
109
(D3) Zu zeigen: Für jedes r ∈ β gibt es ein s ∈ β mit s < r.
2 −2
r>0
Setze s := 2r+2
=⇒ s > 0 und r − s = rr+2
> 0 =⇒ s < r
r+2
s2 − 2 =
2(r2 −2)
(r+2)2
> 0 =⇒ s2 > 2, also ist s ∈ β, s < r.
Behauptung 2. β ist kein rationaler Schnitt s, s ∈ Q. Insbesondere ist
Q ( R.
Beweis. Angenommen β = s, s ∈ Q. Dann besitzt die Menge α := Q\s das
Maximum s.
Zeige aber, daß α kein Maximum haben kann, Widerspruch.
Beweis. Wegen 1 ∈ α = Q \β genügt es zu zeigen: Ist r ∈ α und r ≥ 1,
so gibt es ein s ∈ α mit s > r. Setze s := 2r+2
. Dann ist s > 0 und r2 ≤ 2
r+2
wegen r 6∈ β. Wie oben gesehen ist r2 6= 2, also r2 < 2. Es folgt
s2 − 2 =
2(r2 − 2)
r2 − 2
2
<
0,
also
s
<
2
und
r
−
s
=
<0
(r + 2)2
r+2
Also ist s > r, s ∈ α.
Die Elemente von R nennen wir reelle Zahlen. Alle rationalen Zahlen sind
also reelle Zahlen, aber nicht alle reellen Zahlen sind rational.
Die Anordnung der reellen Zahlen. Seien β und β 0
Dedekind’sche Schnitte.
Definition. β ≤ β 0 genau dann, wenn β 0 ⊆ β
β < β 0 := β ≤ β 0 und β 6= β 0 , d.h. β 0 ( β)
5.1 Bemerkung. ≤“; ist eine lineare Ordnung auf R, welche die Ordnung
”
auf Q fortsetzt.
Beweis. Fortsetzung der Ordnung auf Q. Seien s, t ∈ Q, t ≤ s.
Zu zeigen: t ≤ s, d.h. s ⊆ t:
r ∈ s =⇒ r > s ≥ t =⇒ r > t =⇒ r ∈ t; also ist s ⊆ t.
Reflexivität. β ⊆ β
Transitivität. β 0 ⊆ β und β 00 ⊆ β 0 =⇒ β 00 ⊆ β =⇒ β ≤ β 00
Antisymmetrie. β 0 ⊆ β und β ⊆ β 0 =⇒ β = β 0
Linearität. Zu zeigen: Aus β1 ( β2 folgt β2 ⊆ β1 . Wegen β1 ( β2 gibt es
ein r ∈ β1 mit r ∈ β1 mit r 6∈ β2 . Dann gilt nach (D2): r < s für alle s ∈ β2 .
110
Also ist r ∈ β1 und r < s für alle s ∈ β2 . Nach (D2) ist dann s ∈ β1 für alle
s ∈ β2 , d.h. β2 ⊆ β1 .
Untere Schranken und Infimum. Sei M ⊆ R.
Definition.
a) x ∈ R heißt eine untere Schranke von M , wenn x ≤ m für alle
m ∈ M . Die Menge M heißt nach unten beschränkt; wenn M eine
untere Schranke besitzt.
b) Sei M nach unten beschränkt. Eine untere Schranke s von M heißt Infimum (größte untere Schranke) von M , wenn alle unteren Schranken
von M kleiner oder gleich s sind. Schreibe dann dafür Inf M . Also
Inf M = Max{s | s ist untere Schranke von M }, falls die rechte Seite
existiert!
Beispiel. M = {r ∈ R | r > 0} : Inf M = 0, 0 6∈ M .
5.2 Vollständigkeit der reellen Zahlen. Jede nicht leere nach unten beschränkte Teilmenge von R hat ein Infimum.
Beweis. Sei B ⊆ R nicht leer und nach unten beschränkt. α ∈ R sei eine
untere Schranke für B, d.h.
α ≤ β für alle β ∈ B(d.h. β ⊆ α für alle β ∈ B)
S
Behauptung. β0 :=
β ∈ R und β0 = Inf B
β∈B
Beweis: Für β0 ∈ R müssen die Axiome (D1), (D2), (D3) nachgewiesen
werden.
(D1) B 6= ∅ und β 6= ∅ für β ∈ B =⇒ β0 6= ∅. α ≥ β0 , denn α ≥ β für alle
β ∈ B =⇒ Q\β0 ⊇ Q\α 6= ∅.
(D2) Sei r ∈ β0 ; s ∈ Q mit s > r =⇒ Es gibt ein β ∈ B mit r ∈ β und s ∈ Q
(D2)
mit r < s =⇒ s ∈ β =⇒ s ∈ β0 .
(D3)
(D3) r ∈ β0 =⇒ Es gibt ein β ∈ B mit r ∈ β =⇒ Es gibt ein s ∈ β mit
s < r =⇒ s ∈ β0 und s < r.
β0 = Inf B, denn: (i) β ⊆ β0 für alle β ∈ B =⇒ β0 ≤ β für alle β ∈ B, d.h.
β0 ist untere Schranke von B.
111
(ii) Sei β̃ untere Schranke von B =⇒Sβ̃ ≤ β für alle β ∈ B
β ⊆ β̃ =⇒ β̃ ≤ β0 .
=⇒ β ⊆ β̃ für alle β ∈ B =⇒ β0 =
β∈B
Addition in R. Seien β1 , β2 ∈ R.
β1 + β2 := {r + s | r ∈ β1 und s ∈ β2 }
5.3 Satz. (R, +) ist eine abelsche Gruppe. Die Addition auf R setzt die
Addition auf Q fort. Es gelten die folgenden Regeln:
(i) 0 = {r | r ∈ Q, r > 0} ist ein neutrales Element von R.
(ii) −β = {−r | r ∈ Q\β, r 6= Max Q\β, falls dieses existiert }
(iii) Aus α < β folgt α + γ < α + β für alle α, β, γ ∈ R.
Beweis. H.D.Ebbinghaus et al: Zahlen, Springer, Berlin (1988)
Multiplikation in R. Seien α, β ∈ R
Definition. Ist α ≥ 0 und β ≥ 0, so setzt man
α · β := {rs | r ∈ α, s ∈ β}
Ist α < 0 und β < 0, so setzt man αβ := (−α)(−β)
Ist α < 0 und β > 0, so setzt man αβ := −(−α)β
Ist α > 0 und β < 0, so setzt man αβ := −α(−β)
5.4 Satz. (R, +, ·) ist ein Körper; er heißt Körper der reellen Zahlen. Die
Multiplikation setzt die Multiplikation auf Q fort. Ferner gelten die folgenden
Regeln
(1) 1 ist das neutrale Element der Multiplikation
(2) Für α > 0 ist α−1 = {r−1 | r ∈ Q\α, r > 0, r 6= Max Q\α}
Für α < 0 ist α−1 = −(−α)−1
(3) Ist α ≤ β und γ ≥ 0, so ist auch αγ ≤ βγ.
Beweis. E. Landau: Grundlagen der Analysis, Leipzig (1930)
5.6 Satz von Archimedes. Seien a, b ∈ R mit b > 0. Dann gibt es ein
n ∈ N mit nb > a.
112
Beweis. Angenommen für alle n ∈ N wäre nb ≤ a, also −a ≤ −nb für
alle n ∈ N. Dann wäre die reelle Zahl −a untere Schranke für die Menge
M = {−nb | n ∈ N}.
Nach 5.2 existiert das Infimum I von M in R. Ferner gilt wegen b > 0
auch I + b > I. Also ist nach Definition des Infimums I + b keine untere
Schranke von M . Es gibt daher eine Zahl n0 ∈ N mit −n0 b < I + b. Es folgt
I > −(n0 + 1)b. Wegen −(n0 + 1)b ∈ M widerspricht dies der Tatsache, daß
I untere Schranke von M ist.
Also ist die Annahme falsch und es existiert ein n ∈ N mit nb > a.
113
§6 Konvergente Folgen
Gemäß §5 gelten folgende
Grundlegende Eigenschaften der reellen Zahlen ( Axiome“)
”
(R, +, ·, ≤) ist eine Menge mit zwei Verknüpfungen +“ und ·“ und einer
”
”
Relation ≤“, so daß gilt:
”
(R1) (R, +, ·) ist ein Körper.
(R2) ≤“ ist eine lineare Anordnung auf R, welche folgende Monotonieei”
genschaften besitzt: Seien a, b, c ∈ R.
(i) Aus a ≤ b folgt a + c ≤ b + c.
(ii) Aus a ≤ b und c ≥ 0 folgt ac ≤ bc.
(R3) (Vollständigkeitsaxiom). Jede nicht leere, nach unten beschränkte
Teilmenge hat in R ein Infimum.
Man kann zeigen, daß R durch diese Axiome“ im folgenden Sinne eindeutig
”
festgelegt ist:
6.1 Bemerkung. Sei (K, +, ·, ≤) eine Menge mit Verknüpfungen +“ und
”
·“ sowie einer Relation ≤“, so daß (R1) bis (R3) erfüllt sind. Dann gibt es
”
”
eine bijektive Abbildung R −→ K , r 7→ r̃ mit den Eigenschaften
(1)
(2)
(3)
˜ s = r̃ + s̃
r+
r ˜· s = r̃ · s̃
r ≤ s ⇐⇒ r̃ ≤ s̃
Weiter ergab sich in §5 aus (R1) bis (R3) der
Satz des Archimedes. Zu jedem Paar (a, b) ∈ R × R mit b > 0 gibt es ein
n ∈ N mit nb > a.
A. Konvergente Folgen. Für a ∈ R setzen wir
(
|a| =
a,
falls a ≥ 0
−a, falls a < 0
|a| heißt Betrag von a. Offenbar ist |a| = | − a|. Wie man leicht nachrechnet,
gilt
114
6.2 Regel.
a) |a| ≥ 0; |a| = 0 ⇐⇒ a = 0
b) |a||b| = |ab|
c) |a + b| ≤ |a| + |b|
Erinnerung. Eine Folge reeller Zahlen ist eine Abbildung
a : N =⇒ R, n 7−→ an . Andere Schreibweisen sind
(an | n ∈ N); (an )n∈N ; (an ); a0 , a1 , a2 , . . .“
”
Auch Abbildungen {n ∈ N | n ≥ k} −→ R, n 7−→ an werden als Folgen
bezeichnet: (an | n ≥ k) oder ak , ak+1 , . . .“
”
Beispiele.
a) Die konstanten Folgen (c): c, c, c, . . . “(c ∈ R)
”
1
1 1 1
b) ( n )n∈N>0 : 1, 2 , 3 , 4 , . . .
c) (xn )n∈N : 1, x, x2 , x3 , . . .
d) Rekursive Folgen, etwa: a1 = 1, an =
8 13 21
Folge 1, 21 , 32 , 35 , 85 , 13
, 21 , 34 , . . .
1
1+an−1
für n ≥ 2 definiert die
Definition. Eine Folge (an ) konvergiert gegen a ∈ R, wenn gilt:
Zu jeder reellen Zahl ε > 0 existiert eine Zahl N ∈ N, so daß |an − a| < ε für
alle n > N .
Anders ausgedrückt. Für jedes vorgegebene ε > 0 - so klein es auch
gewählt sein mag - sind nur endlich viele Folgenglieder außerhalb des
Bereichs I = (a − ε, a + ε) := {x | a − ε < x < a + ε}.
a−ε
a
a+ε
(Den schraffierten Bereich I nennt man das Intervall zwischen a − ε und
a + ε. Es hat die Länge 2ε.) Die Folge häuft “sich also nur in der Nähe“von
”
”
a. Eine Folge, die nicht konvergiert, heißt divergent.
115
Für die Aussage (an ) konvergiert gegen a“ schreibt man kurz
”
(an ) −→ a oder a = lim an
n→∞
Wir werden gleich sehen, daß die Zahl a dann durch die Folge (an ) eindeutig
bestimmt ist und nennen a den Limes oder Grenzwert der Folge (an ).
Beispiele.
a) (c) konvergiert gegen c.
(
1
falls n gerade
−1 falls n ungerade
ist divergent, denn sie häuft sich bei 1 und bei −1.
b) Die Folge an = (−1)n =
c) ( n1 )n≥1 konvergiert gegen Null.
Beweis von c). Sei ε > 0 vorgegeben. Nach Archimedes gibt es eine Zahl
N , so daß N > 1ε . Für alle n > N ist dann n1 < N1 < ε. Daher ist
|
1
1
− 0 |= < ε für alle n > N
n
n
6.3 Satz. (Eindeutigkeit des Grenzwertes.) Konvergiert (an ) gegen a und
(bn ) gegen b, so gilt:
a) Aus an ≤ bn für alle n ∈ N folgt a ≤ b
b) Jede konvergente Folge hat nur einen Grenzwert.
Beweis.
a) Angenommen a > b. Wähle ε = a−b
> 0. Dann existiert ein N ∈ N, so
2
daß für alle n > N gilt:
|an − a| < a−b
und |bn − b| < a−b
. Es folgt
2
2
a − b ≤ (a − b) + (bn − an ) = |(a − an ) + (bn − b)| ≤
≤ |a − an | + |bn − b| = |an − a| + |bn − b| <
Widerspruch.
116
a−b a−b
+
= a − b,
2
2
b) Im Fall (an ) = (bn ) ist an ≤ bn und bn ≤ an . Nach a) folgt daraus a ≤ b
und b ≤ a, also a = b.
Folgen kann man addieren und multiplizieren. Man setzt
λ(an ) := (λan )
für λ ∈ R
(an ) + (bn ) := (an + bn )
(an ) · (bn ) := (an bn )
6.4 Satz. (an ) konvergiere gegen a und (bn ) gegen b. Dann gilt
a) λ(an ) konvergiert gegen λa (λ ∈ R)
b) (an ) + (bn ) konvergiert gegen a + b
c) (an )(bn ) konvergiert gegen ab
d) Ist cn = an+1 , d.h. (cn ) : a1 , a2 , a3 , . . . , so konvergiert (cn ) gegen a.
Beweis. Für a) und b) zeigen wir, daß λ(an )+µ(bn ) −→ λa+µb für λ, µ ∈ R.
Sei ε > 0 vorgegeben. Es ist zu zeigen: Für fast alle“ n (d.h. für alle n bis
”
auf endlich viele Ausnahmen) ist An =| λan + µbn − (λa + µb) |< ε.
An = | λ(an − a) + µ(bn − b) |≤ |λ| |an − a| + |µ| |bn − b|
Setze B = |λ| + |µ| + 1 und η =
ε
B
> 0.
Dann gilt für fast alle n ∈ N
|an − a| < η und |bn − b| < η, also
An ≤ |λ| |an − a| + |µ| |bn − b| ≤ |λ| η + |µ| η < Bη = ε
c) Sei ε > 0 vorgegeben. Es ist zu zeigen, daß
(∗)
| an bn − ab |< ε
für fast alle n ∈ N. Dazu zeigen wir zunächst
6.5 Lemma. Jede konvergente Folge (bn ) ist beschränkt, d.h. es gibt ein
B ∈ R, so daß |bn | < B für alle n ∈ N.
117
Beweis. Sei (bn ) −→ b. Für ε = 1 gibt dann ein n ∈ N, so daß |bn − a| < 1
für alle n > N . Es folgt
|bn | − |b| ≤ |bn − b| < 1, also |bn | < |b| + 1 für alle n > N .
=⇒ |bn | < Max {|b0 |, . . . , |bN |, |b| + 1} + 1 := B.
Zum Beweis von (∗). Nach 6.2 gilt
(∗∗)
=
|an bn − ab| = |an bn − abn + abn − ab| =
|bn (an − a) + a(bn − b)| ≤ |bn | |an − a| + |a| |bn − b|
Nach 6.5 existiert ein B > 0, so daß |bn | < B für alle n und |a| < B.
Daher ist nach (∗∗)
|an · bn − ab| < B(|an − a) + |bn − b|) für alle n
ε
Für fast alle n ist |an − a| < 2B
= ε0 und |bn − b| < ε0 , es folgt
ε
ε
) = ε für fast alle n.
B(|an − a| + |bn − b|) < B( 2B + 2B
c) ist klar.
B. Monotone Konvergenz.
Definition. Eine Folge (an ) heißt
monoton wachsend, wenn an ≤ an+1 für alle n ∈ N
monoton fallend, wenn an+1 ≤ an für alle n ∈ N
nach unten beschränkt, wenn es ein B gibt mit an ≥ B für alle n ∈ N
nach oben beschränkt, wenn es ein B gibt mit an ≤ B für alle n ∈ N.
Wie gesehen sind konvergente Folgen (nach unten und oben) beschränkt. Von
dieser Aussage gibt es eine partielle Umkehrung.
6.6 Satz von der monotonen Konvergenz.
a) Eine monoton fallende nach unten beschränkte Folge konvergiert gegen
das Inifum der Menge seiner Folgenglieder.
b) Eine monoton wachsende nach oben beschränkte Folge konvergiert.
c) Jede reelle Zahl erhält man als Grenzwert einer Folge rationaler Zahlen.
Beweis.
118
a) Nach Voraussetzung gibt es ein B ∈ R mit B ≤ an+1 ≤ an für alle
n ∈ N. Daher besitzt die Menge {an | n ∈ N} nach 5.3 ein Infimum
a = Inf M ∈ R. Es ist dann insbesondere an ≥ a für alle n ∈ N. Sei
ε > 0 vorgegeben.
Dann ist a + ε keine untere Schranke von M , da a die größte untere
Schranke von M ist. Also gibt es ein N ∈ N mit aN < a + ε. Da (an )
monoton fallend ist gilt dann auch an ≤ aN < a + ε für alle n > N .
Also ist a ≤ an < a + ε und somit
|an − a| = an − a < a + ε − a = ε für alle n > N
b) Ist (an ) monoton wachsend und nach oben beschränkt, so ist (bn ) =
(−an ) monoton fallend und nach unten beschränkt, konvergiert also
nach a) gegen ein b ∈ R. Dann konvergiert nach 6.4 auch (an ) = −(bn ),
und zwar gegen −b.
c) Sei α ∈ R beliebig. Setze βn := α + n1 , n ≥ 1. Dann ist (βn ) monoton
fallend und βn > α; daher ist (βn ) konvergent nach a). Definitionsgemäß
bedeutet α < βn , daß βn ( α. Wähle ein rn ∈ α mit rn 6∈ βn . Dann ist
α ≤ rn ≤ βn für alle n ∈ N.
1
n→∞ n
Nach 6.4 ist lim βn = α + lim
n→∞
= α.
Sei ε > 0 vorgegeben. Dann gibt es ein n ∈ N, so daß
βn − α = |α − βn | < ε für alle n > N . Es folgt
|α − rn | = rn − α ≤ βn − α < ε für alle n > N, d.h. α = lim rn .
n→∞
C. Quadratwurzeln. Als Anwendung wollen wir zeigen, wie man zu
jeder positiven reellen Zahl eine Quadratwurzel berechnet. Sei a > 0 aus R.
Wir konstruieren ein w ∈ R mit w2 = a. Dazu betrachten wir die rekursive
Folge (an ) mit
(∗)
a0 = a + 1 und an+1 =
an
a
+
für n ≥ 0.
2
2an
6.7 Satz. Die Folge (an ) konvergiert gegen eine positive reelle Zahl w mit
2
w
√ = a. Man nennt w die Quadratwurzel aus 2a und schreibt dafür auch
a. Ist a ∈ Q, so ist w = {r | r ∈ Q, r > 0 und r > a} =: α.
119
6.8 Lemma. Setze noch a−1 = a + 2. Dann gilt für alle n ≥ 0
an > 0, an < an−1 und a2n > a.
Insbesondere ist (an )n∈N monoton fallend und nach unten beschränkt, also
nach 6.6 konvergent.
Zeige zunächst, wie 6.7 aus dem Lemma folgt. Sei w = lim an . Wegen 6.6
ist w = Inf {an | n ∈ N} ≥ 0. Nach (∗) gilt ferner
n→∞
2an+1 an = a2n + a.
Mit 6.4 folgt daraus 2(an+1 )(an ) −→ 2ww = 2w2 und
2(an+1 )(an ) = (a2n ) + (a) −→ w2 + a. Also ist 2w2 = w2 + a und w2 = a mit
w > 0.
Beweis von 6.8. (Vollständige Induktion nach n)
n = 0 : a0 = a + 1 > 0; a0 = a + 1 < a + 2 = a−1 , a20 = (a + 1)2 ≥ 2a + 1 > a
Sei n ≥ 0 und es gelte an > 0, an < an−1 und a2n > a.
Schluß von n auf n + 1:
Setze x =
a−a2n
; a2n+1
2a2n
an+1 = an +
an+1 =
an
2
+
= an (1 + x)2 > a2n (1 + 2x) = a wegen x 6= 0
a−a2n
< an , denn a − a2n < 0 und an
2an
a
< a2n > 0, da an > 0 und a > 0
2an
>0
Sei M = {β | β ∈ R, β > 0 und β 2 ≥ a}. Wie man leicht sieht (Übungsaufgabe) ist dann α ∈ M und β ⊆ α für alle β ∈ M , d.h. α = InfM = w.
Nachtrag. Sei M eine nach oben beschränkte Menge dann ist
−M = {−x | x ∈ M } nach unten beschränkt. Ist b = Inf (−M ) so ist
offenbar −b die kleinste obere Schranke von M . Wir haben also gesehen
5.3’ Satz. Jede nach oben beschränkte Menge besitzt ein Supremum.
Aus dem Beweis von 6.6 ergibt sich
6.6’ Satz. Jede monoton wachsende und nach oben beschränkte Folge konvergiert gegen das Supremum der Menge ihrer Folgenglieder.
120
§7 Dezimalbrüche
Wir wollen die bekannte Tatsache herleiten, daß die reellen Zahlen nichts
anderes sind als die sogenannten Dezimalbrüche.
A. Dezimalbrüche. Sei (an )n≥1 eine Folge reeller Zahlen. Betrachte
dazu die Folgen (sn )n≥1 mit
s1 = a1 , s2 = a1 + a2 , . . . , sn = a1 + . . . + an , . . .
(sn )n≥1 wird dann auch als unendliche Reihe mit den Summanden a1 , a2 , a3 , . . .
bezeichnet.
Konvergiert (sn ) gegen die Zahl s, so schreibt man für s auch den Ausdruck
∞
P
an .
n=1
s=
”
∞
X
an“ bedeutet also: s = lim sn
n→∞
n=1
Definition. Eine Folge (an )n≥1 heißt Ziffernfolge, wenn gilt:
(i) an ∈ N und 0 ≤ an ≤ 9 für alle n ∈ N.
(d.h.: an ist eine Ziffer im Zehnersystem.)
(ii) Für jedes n ∈ N gibt es ein m > n mit am 6= 9.
(M.a.W.: Unendlich viele Ziffern an sind von 9 verschieden.)
7.1 Satz. Sei (an )n≥1 eine Ziffernfolge. Dann konvergiert die unendliche Reihe
mit den Teilsummen
a2
an
sn = a101 + 100
+ . . . + 10
n und es gilt
0≤
∞
X
an
<1
n
10
n=1
Beweis. Wegen (i) gilt für die Teilsummen sn =
sn +
1
10n
≤
9
10
+
9
102
+ ... +
9
10n
+
1
10n
=
9
10
+
= ... = 1
Also gilt für alle n ≥ 2
121
9
102
a1
10
+
a2
100
+ ... +
+ ... +
9
10n−1
+
an
10n
1
10n−1
=
(1)
0 ≤ sn−1 ≤ sn < 1
Nach 6.6’ konvergiert daher (sn ) gegen s := sup{sn | n ∈ N}.
Ferner gilt nach (1): 0 ≤ s ≤ 1. Also ist
∞
X
an
≤1
0≤
10n
n=1
(2)
für alle Ziffernfolgen (an )n≥1 .
Wegen (ii) gibt es ein k ≥ 1, so daß ak < 9. Dann ist auch die Folge
(
an
für k 6= n
ãn =
eine Ziffernfolge
an + 1 für k = n
und s̃n :=
n
P
ν=1
ãν
10ν
= sn +
1
10k
für alle n ≥ k.
Wende (2) auf die Folge (s̃n ) an und erhalte
∞
X ãν
1
1
s+ k =
≤ 1, also 0 ≤ s < s + k ≤ 1.
ν
10
10
10
ν=1
7.2 Satz. Verschiedene Ziffernfolgen (an ) und (bn ) induzieren auch verschie∞
∞
P
P
an
bn
dene Grenzwerte
und
.
n
10
10n
n=1
n=1
Beweis. Sei k = Min{n | n ≥ 1 und an 6= bn }. Wir können ak ≥ bk + 1
annehmen. Setze
tn =
b1
bn
+ ... + n
10
10
;
sn =
a1
an
+ ... + n.
10
10
Dann gilt für alle n ≥ 1
10k sk+n = (10k−1 a1 + . . . + ak )
=
α
k
k−1
10 tk+n = (10 b1 + . . . + bk )
=
β
k+n
+ ( ak+1
+ . . . + a10
n )
10
+
s̃n
, α ∈ N,
bk+1
ak+n
+ ( 10 + . . . + 10n )
+
t̃n
, β ∈ N.
122
Nach Voraussetzung ist α ≥ β + 1.
Ferner sind ãn := ak+n , b̃n := bk+n , n ≥ 1 ebenfalls Ziffernfolgen. Nach 7.1
gilt daher
0 ≤ lim s̃n < 1 und 0 ≤ lim t̃n < 1. Es folgt
n→∞
n→∞
!
∞
X an
10k
= 10k lim sk+n = α + lim s̃n ≥
n
n→∞
n→∞
10
n=1
≥ α ≥ β + 1 > β + lim t̃n = 10k lim tk+n = 10k
n→∞
Kürzen ergibt:
∞
P
n=1
an
10n
>
∞
P
n=1
n→∞
∞
X
bn
10n
n=1
!
.
bn
.
10n
Definition. Eine reelle Zahl der Form
r = a0 +
∞
X
an
10n
n=1
wobei a0 ∈ N und (an )n≥1 eine Ziffernfolge ist heißt Dezimalbruch.
Nach 3.1 hat a0 eine eindeutige Zifferndarstellung
a0 = bn bn−1 . . . bn = bn 10n + bn−1 10n−1 + . . . + b1 10 + b0 , wobei
n ≥ 0, 0 ≤ bν ≤ 9 für ν = 0, . . . , n und bn 6= 0, falls n ≥ 1.
Aus 3.1 und 7.2 ergibt sich also:
Die Dezimalbrüche entsprechen eineindeutig den Paaren von Folgen natürlicher Zahlen bestehend aus
(1) einer endlichen Folge
bn , bn−1 , . . . , b1 , b0 mit 0 ≤ bν ≤ 9 für alle 0 ≤ ν ≤ n
und bn 6= 0, falls n 6= 0, und
(ii) einer Ziffernfolge
a1 , a2 , a3 , . . .
Schreibe beide Folgen nebeneinander, getrennt durch ein Komma:
(∗)
bn bn−1 . . . b0 , a1 a2 a3 . . .
123
Der Ausdruck (∗) heißt dann die Dezimaldarstellung der Zahl
n
n−1
r = bn 10 + bn−1 10
∞
X
an
+ . . . + b1 · 10 + b0 +
10n
n=1
7.3 Satz. Jede nicht negative reelle Zahl ist ein Dezimalbruch.
Zum Beweis sind einige Vorbereitungen zu treffen.
Der ganze Anteil einer nicht negativen reellen Zahl.
Sei a ∈ R, a ≥ 0. Nach Archimedes (5.6 mit b = 1) gilt:
Es gibt ein n ∈ N mit n > a
Nach dem Prinzip vom kleinsten Element (§1) besitzt daher
M = {n | n ∈ N, n > a} ein kleinstes Element m. Dann ist m − 1 ≥ 0 und
m − 1 ist die größte natürliche Zahl ≤ a. Sie wird mit [a] bezeichnet.
Offenbar ist
0 ≤ a − [a] < 1
Man nennt daher [a] auch den ganzen Anteil von a und s = a − [a] den
gebrochenen Anteil von a. Es gilt
a = [a] + s,
[a] ∈ N und 0 ≤ s < 1
Beweis von 7.3. Nach 3.1 hat a0 = [a] eine Dezimaldarstellung a0 =
∞
P
an
bn bn−1 . . . b0 . Es genügt daher, s in einen Dezimalbruch s =
mit einer
10n
n=1
Ziffernfolge (an )n≥1 zu entwickeln.
Definiere dazu rekursiv
r0 = s
a1 = [10r0 ],
..
.
r1 = 10r0 − a1
an = [10rn−1 ], rn = 10rn−1 − an , n ≥ 0
Induktiv folgt: 0 ≤ rn < 1, an ∈ N und 0 ≤ an ≤ 9 für alle n ≥ 1.
Setze: sn =
a1
10
+ ... +
an
.
10n
Es ist zu zeigen
(1) (sn ) konvergiert gegen s
124
(2) an 6= 9 für unendlich viele n.
Beweis. Zu (1). Durch vollständige Induktion ergibt sich
rn
s − sn = n für alle n ≥ 1 :
10
r1
a1
n = 1 s − s1 = r0 − 10 = 10 .
an
10n
rn−1
10n−1
rn
= 10
n.
n
Nach der binomischen Formel ist 10n > 2n = (1 + 1)n ≥ 1 +
> n und
1
rn
1
1
daher 0 ≤ 10
n < 10n < n .
Schluß von n − 1 auf n : s − sn = s − sn−1 −
=
−
an
10n
Da ( n1 ) eine Nullfolge ist gilt (ε > 0 beliebig)
rn
1
< < ε für fast alle n.
n
10
n
Zu (2). Angenommen es sei am = 9 für alle m ≥ n.
|s − sn | =
Setze tk = (sn+k − sn )10n , d.h.
9
9
9
+ 2 + . . . + k für alle k ≥ 1
10 10
10
n
n
Es folgt (tk ) = 10 (sn+k −sn ) −→ 10 (s−sn ) = rn für k → ∞ und 0 ≤ rn < 1.
tk =
Andererseits ist aber tk +
1
10k
= 1, daher gilt nach 6.4
(1) = (tk ) + ( 101k ) −→ rn + 0 = rn , denn wegen
0 < 101k < k1 ist mit ( k1 ) auch ( 101k ) eine Nullfolge.
Es folgt rn = 1, im Gegensatz zu 0 ≤ rn < 1.
Fazit. Die nicht negativen reellen Zahlen besitzen eine eindeutig bestimmte
Dezimaldarstellung.
bn bn−1 . . . b0 , a1 a2 a3 . . . , wobei n ≥ 0 und bν und aµ 10–adische Ziffern sind
mit bn 6= 0, falls n ≥ 1 und aµ 6= 9 für unendlich viele µ.
Offenbar bedeutet die Multiplikation mit 10 eine Verschiebung des Kommas
um eine Stelle nach rechts.
Frage. Wie sieht man es der Dezimaldarstellung von r ∈ R an, ob r eine
rationale Zahl ist?
7.4 Satz. Sei r ≥ 0 eine reelle Zahl und 0, a1 a2 a3 . . . die Dezimaldarstellung
des gebrochenen Anteils von r.
Dann sind folgende Aussagen äquivalent:
125
(i) r ist rational
(ii) Die Folge (an )n≥1 ist periodisch, d.h. ab einer gewissen Stelle s+1 kehrt
immer ein und derselbe Ziffernkomplex c1 , . . . , ct wieder: Es gibt s ≥ 0
und t ≥ 1 mit
a1 , a2 , a3 , . . . = a1 . . . as
=: a1 . . . as
c1 . . . c t
c1 . . . c t
c1 . . . ct
c1 . . . c t
......
Beweis. (ii) =⇒ (i) “. Sei r ∈ R, r > 0 mit periodischer Dezimalbruchent”
wicklung. Dann gibt es ein µ ∈ N, so daß
10µ · r = g + 0, c1 . . . cν , g ∈ N
s+g
∈ Q.
10µ
mit h = c1 . . . cν ∈ N. Es folgt s = 10νh−1 ∈ Q.
Sei r > 0 eine rationale Zahl. Schreibe r = pq mit
Zeige, daß s = 0, c1 . . . cν ∈ Q. Dann ist auch r =
10ν · s = h + s
(i) =⇒ (ii) “.
”
ganz und ggT (p, q) = 1.
p ≥ 1, q ≥ 1
Nach Abzug des ganzen Anteils können wir annehmen, daß 0 < r < 1.
Unterscheide drei Fälle:
1) q = 2α 5β mit α, β ∈ N, α + β ≥ 1. Wähle ein n ∈ N mit n ≥ α und
n
n ≥ β. Dann ist r̃ := 10n · r = 210
α 5β · p ∈ N.
Aus 0 < r < 1 folgt: 0 < r̃ < 10n . Daher hat r̃ eine Dezimaldarstellung
der Form
r̃ = a0 + a1 · 10 + . . . + aµ 10µ mit 0 ≤ µ ≤ n − 1,
Es folgt r =
r̃
10n
=
a0
10
+
a1
10n−1
+ ... +
aµ
10n−µ
0 ≤ ak ≤ 9,
aµ 6= 0
und
r = 0, 0 · · · 0aµ aµ−1 . . . a1 a0 0 . . . (ν = 1, c1 = 0)
2) ggT (q, 10) = 1. Dann ist g := 10 + qZ ein Element der Einheitengruppe
G = (Z/qZ)× des Rings Z/qZ.
126
Es folgt ν = ord g | ord G = ϕ(g) und g ν = 1 in G, d.h. 10ν ≡ 1 mod q.
Definitionsgemäß ist ν = ord g das kleinste n ≥ 1 mit 10n ≡ 1 mod q,
10ν · r =
(mq + 1)p
p
10ν p
=
= mp + = mp + r, m ∈ N
q
q
q
Also ist r der gebrochene Anteil von 10ν · r, d.h.: Ist
r = 0, r1 r2 . . . rν
rν+1 . . . , so ist
10ν · r = r1 . . . rν , rν+1 . . . = r1 . . . rν , r1 r2 . . .
Es folgt r1 = rν+1 , r2 = rν+2 , . . . rν = r2ν , . . . .rk = rk+ν für alle k ≥ 0, d.h.
r1 r2 r3 . . . ist rein periodisch mit Periode r1 . . . rν .
3) q = 2α 5β · Q mit ggT (Q, 10) = 1.
Sei n wie in 1) so, daß n ≥ α und n ≥ β. Setze ν = ord (10 + Q · Z) in
0
G = (Z/QZ)× . Dann gilt 10n · r = PQ und P 0 ∈ N mit ggT (Q, P 0 ) = 1
wegen ggT (p, Q) = (10, Q) = 1.
Wegen 0 < r < 1 ist 0 < 10n r < 10n .
Dividiere P 0 durch Q mit Rest. Erhalte
P0
P
P
= R+ mit ggT (P, Q) = 1, ggT (10, Q) = 1, 0 ≤
< 1, 0 ≤ R < 10n
Q
Q
Q
P, R ∈ N.
wobei P, R ∈ N.
R hat dann eine Dezimaldarstellung R = An−1 An−2 . . . A0 . Die Zahl
P
P
∈ Q mit 0 ≤ Q
< 1 erfüllt die Voraussetzungen von 2). Also gilt
Q
P
= 0, r1 . . . rν und
Q
r = 0, An−1 An−2 . . . A0 r1 . . . rν
127
§8 Die komplexen Zahlen
Wir betrachten eine Polynomgleichung
X n + an−1 X n−1 + . . . + a1 X + a0 = 0
mit reellen Koeffizienten a0 , a1 , . . . , an−1 , n ≥ 2. Sie hat im Allgemeinen keine reelle Lösung, d.h es gibt kein y ∈ R mit
y n + an−1 y n−1 + . . . + a1 y + a0 = 0
Um diesen Mangel zu beheben, haben Euler und Andere die komplexen Zahlen erfunden. Sie bilden eine Erweiterung von R, in der alle Gleichungen
obigen Typs Lösungen haben. Die komplexen Zahlen werden mit C bezeichnet.
Zum Beispiel ist die Gleichung X 2 + 1 = 0 in R unlösbar und in C lösbar.
Die Lösungen bezeichnet man gewöhnlich mit i und −i. Der Bereich C wird
ein Körper sein, der R umfaßt, möglichst klein“ ist und i enthält. Genauer
”
wird gelten: C = {a + ib | a, b ∈ R} mit einer imaginären“ Größe i, welche
”
die Bedingungen i2 = −1 erfüllt.
Konstruktion von C. Die Menge R2 = {(x, y) | x, y ∈ R} der Paare reeller
Zahlen bilden bekanntlich einen Vektorraum vermöge der Operationen
(x1 , y1 ) + (x2 , y2 ) = (x1 + x2 , y1 + y2 ) (Vektoraddition)
λ(x, y) = (λx, λy) für alle λ ∈ R
(Skalarmultiplikation)
(i) Die dem Körper C zugrunde liegende Menge ist R2 ( C = R2“).
”
2
(ii) Die Addition auf C ist die Vektoraddition auf R .
(iii) Die Multiplikation von Elementen aus C ist erklärt durch
(x1 , y1 ) · (x2 , y2 ) := (x1 x2 − y1 y2 , x1 y2 + y1 x2 )
8.1 Bemerkung.
(1) (C, +) ist eine abelsche Gruppe (denn (R2 , +, ·) ist sogar ein Vektorraum).
(2) Die Multiplikation ist kommutativ und (1, 0) · (x, y) = (x, y), d.h. 1C :=
(1, 0) ist neutrales Element der Multiplikation.
128
(3) Für die Multiplikation gilt das Assoziativgesetz.
(4) Jedes (x, y) 6= (0, 0) hat ein Inverses bezüglich der Multiplikation:
(
x
y
,
−
) · (x, y) = (1, 0) = 1C
x2 + y 2 x2 + y 2
(5) Es gilt das Distributivgesetz in C.
Mit anderen Worten bedeuten (1) bis (5): C ist ein Körper mit Eins =
(1, 0) und Null = (0, 0).
Die Gültigkeit von (2), (3) und (5) prüft man leicht nach. Wir wollen
noch R als einen Unterring von C auffassen:
Einbettung“ von R in C. Die Abbildung
”
ϕ : R −→ C = R2 x 7−→ (x, 0)
ist injektiv. Sie ist mit Addition und Multiplikation verträglich, d.h.
ϕ(x + y) = ϕ(x) + ϕ(y) und ϕ(x · y) = ϕ(x)ϕ(y)
Wir identifizieren x ∈ R mit (x, 0) ∈ C. Damit wird R zu einem Unterring von C. Insbesondere ist dann 1 = (1, 0) und 0 = (0, 0):
R(C ,
x=(x,
ˆ
0) für x ∈ R
(6) (0, 1)2 = (0, 1) · (0, 1) = (−1 · 1, 0) = −(1, 0) = −1
Setze nun i = (0, 1). Dann gilt:
(α) i2 + 1 = 0, d.h. i löst die Gleichung X 2 = −1.
(β) Jedes (x, y) ∈ C schreibt sich eindeutig in der Form
(x, y) = (x, 0) + (0, y) = (x, 0) + (0, 1)(y, 0) = x + iy; x, y ∈ R.
Damit ist C = {x + iy | x, y ∈ R}.
Ist z = (x, y) = x + iy ∈ C mit x ∈ R, y ∈ R, so heißt x der
Realteil und y der Imaginärteil von z. Schreibe dafür x = Rez
und y = Imz.
z heißt rein imaginär, wenn Re z = 0, also z = iy, y ∈ R.
129
Die Gaußsche Zahlenebene. Zeichnet man in der Zeichenebene E ein
kartesisches Koordinatensystem aus, so kann man die Elemente von C = R2
mit den Punkten von E identifizieren. Man spricht deshalb von C auch als
von der Gaußschen Zahlenebene.
iR = y-Achse
.
. z = x + iy = (x, y)
iy .
y |z|
.
x
.x.
R = x-Achse
Bei dieser Darstellung entsprechen die reellen Zahlen den Punkten auf der
x–Achse. Die Punkte auf der y–Achse sind die rein imaginären Zahlen.
Konjugation. Für x, y ∈ R heißt
z := x − iy die zu z = x + iy konjugierte Zahl.
Die Länge des Vektors (x, y) heißt Betrag der komplexen Zahl z = x + iy,
d.h. nach Pythagoras
p
|z| := x2 + y 2 , wenn z = x + iy mit x, y ∈ R.
8.2 Regel. Für komplexe Zahlen z und w gilt
130
a) z = z;
b) z = z ⇐⇒ z ∈ R;
c) z + w = z + w, zw = zw;
d) |z|2 = zz;
e) Für z 6= 0 ist z −1 =
1
z;
|z|2
f) |zw| = |z||w|
Beweis. a) und b) sind klar nach Definition.
c) Seien z = (x, y) und w = (u, v).
z + w = (x + u) + i(y + v) = x + u − iy − iv = x − iy + u − iv = z + w
zw = (xu − yv) + i(xv + yu) = (xu − yv) − i(xv + yu) =⇒
z w = (xu − (−y)(−v)) + i(x(−v) + (−y)u) = zw
d) zz = (x + iy)(x − iy) = x2 − i2 y 2 = x2 + y 2 = |z|2
e) folgt aus d)
f) |zw|2 = (zw)zw = (zw)(z w) = (zz) · ww = |z|2 |w|2 nach den vorangegangenen Regeln.
Als Nächstes wollen wir Addition, Multiplikation und Konjugation eine geometrische Deutung geben. Für die Multiplikation ist es zweckmäßig, vorher
zu erklären, was die Polarkoordinaten eines Punktes der Ebene bzw. einer
komplexen Zahl z = x + iy = (x, y) sind.
131
Polarkoordinaten in der Gaußschen Zahlenebene.
iR
ϕ
y = r sin ϕ
.
z = x + iy
r = |z|
ϕ
..
x = r cos ϕ
R
Aus der Zeichnung ist ersichtlich: Ist r = |z| und ϕ der Winkel (im Bogenmaß)
zwischen der positiven reellen Achse und dem Ortsvektor von z 6= 0, so
schreibt sich z in der Form
z = x + iy = r(cos ϕ + i sin ϕ)
Die Zahl z ist durch Angabe von r und ϕ eindeutig festgelegt. Man nennt r
und ϕ die Polarkoordinaten von z. Ihr Betrag r = |z| ist durch z eindeutig
festgelegt. Dagegen ist ϕ nur bis auf ein Vielfaches von 2π bestimmt:
r(cos ϕ + i sin ϕ) = r(cos(ϕ + n · 2π) + i sin(ϕ + n2π)) für n ∈ Z.
ϕ heißt Argument von z. Mit ϕ soll auch ϕ + n2π, n ∈ Z Argument von z
sein. Schreibe Arg z“ für das Argument von z.
”
Geometrische Deutung von Addition, Multiplikation und Konjugation.
132
(i) Addition.
z = x + iy und w = u + iv
x+u
z + w = (x+u) + i (y+v)
v
x
z
y+v
y
.w
v
u
x
0
Vektoren addieren sich wie Kräfte, d.h. die Punkte 0, w, z + w, z bilden
ein ( Kräfte“ -) Parallelogramm.
”
(ii) Multiplikation. Stelle z 6= 0 und w 6= 0 in Polarkoordinaten dar, d.h.
z = r(cos ϕ + i sin ϕ) und w = s(cos ψ + i sin ψ) mit
r = |z|, ϕ = Arg z; s = |w|, ψ = Arg w
Nach 8.2 ist |zw| = rs.
Aus den Additionstheoremen für Sinus und Cosinus folgt
zw = rs(cos ϕ cos ψ − sin ϕ sin ψ) + i(cos ϕ sin ψ + sin ϕ cos ψ)
= |zw|(cos(ϕ + ψ) + i sin(ϕ + ψ)
Also gilt: Arg zw = Arg z+ Arg w und |zw| = |z||w|
133
Merkregel. Komplexe Zahlen (dargestellt durch ihre Polarkoordinaten) werden multipliziert, indem man ihre Beträge multipliziert und
ihre Argumente addiert. Induktiv erhält man noch für alle n ≥ 1:
z n = rn (cos nϕ + i sin nϕ), d.h. : |z n | = |z|n und Arg (z n ) = n · Arg z
Anschauliche Darstellung.
iR
.zw
rs
.w
s
ϕ
ψ
r
ϕ
.z
R
134
(iii) Konjugation.
iR
. z = x + iy
y
y
y
.
R
y
. z = x − iy
Die Konjugationsabbildung C −→ C, z 7−→ z ist also die Spiegelung an der reellen Achse.
135
Anhang A: Die Gleichung X 3 + Y 3 = Z 3
§1 Teilbarkeit in Integritätsbereichen
Sei R ein Integritätsbereich. Wie in Z erklärt man auch in R den Begriff der
Teilbarkeit.
Definition. d ∈ R ist ein Teiler von a ∈ R, wenn es ein q ∈ R gibt mit
a = d · q. Schreibe d | a“. Wie in Z gilt auch hier für a, b, c, d ∈ R:
”
1.1 Regel.
a) a | a und 1 | a.
b) a | b und b | c =⇒ a | c.
c) a | b und c | d =⇒ ac | bd.
d) a | b und a | c =⇒ a | (bx + cy) für alle x, y ∈ R.
e) ac | bc und c 6= 0 =⇒ a | b.
Der Beweis geht wörtlich wie in I.2.1.
1.2 Bemerkung. Ein Element e ∈ R ist genau dann eine Einheit von R,
wenn e | 1. (Damit teilt nach 1.1 eine Einheit sogar jedes Element von R.)
R× bezeichne wie früher die Einheitengruppe von R.
Definition. a und b aus R heißen assoziiert zueinander, wenn
a | b und b | a
(Schreibe dann a ∼ b“)
”
1.3 Bemerkung.
a) ∼ ist eine Äquivalenzrelation.
b) Aus a ∼ b folgt für alle c ∈ R:
(i) a | c ⇐⇒ b | c;
(ii) c | a ⇐⇒ c | b
c) Genau dann ist a ∼ b, wenn es ein e ∈ R× gibt mit b = ea.
136
Beweis.
a) Reflexivität und Symmetrie sind klar.
a ∼ b und b ∼ c =⇒ a | b , b | a , b | c und c | b.
a | b und b | c =⇒ a | c ; c | b und b | a =⇒ c | a.
Also ist a ∼ c.
b) Nach Voraussetzung ist a | b und b | a.
(i) a | c und b | a =⇒ b | c ; b | c und a | b =⇒ a | c.
(ii) c | a und a | b =⇒ c | b ; c | b und b | a =⇒ c | b.
c) a ∼ b =⇒ b = aq und a = bq 0 =⇒ a = (aq)q 0 = a(qq 0 ) =⇒
b = 1 · a = a = 0 oder a 6= 0 und qq 0 = 1 =⇒ q Einheit, b = qa.
e Einheit mit b = ea =⇒ a = e−1 b =⇒ a | b und b | a.
Definition. b heißt ein trivialer Teiler von a, wenn b eine Einheit ist oder
wenn b ∼ a (d.h. b = ea, e ∈ R× ). Die nicht trivialen Teiler b von a nennt
man auch echte Teiler von a. Schreibe dafür b k a“.
”
1.4 Korollar. Sei a 6= 0 und a = bc. Dann gilt:
b k a genau dann, wenn c k a
1.3
Beweis. b k a =⇒ b keine Einheit und b a =⇒ b keine Einheit und c keine
1.3
Einheit =⇒ c a und c keine Einheit =⇒ c k a, und umgekehrt.
Definition. Ein Element u von R heißt irreduzibel (unzerlegbar), wenn
gilt:
(i) u 6= 0 und u ist keine Einheit
(ii) u hat keine echten Teiler.
p heißt Primelement in R, wenn gilt:
(i) p 6= 0 und p ist keine Einheit
(ii) Aus p | ab folgt p | a oder p | b.
137
Beispiele.
a) Die irreduziblen Elemente in Z sind ±p, p eine Primzahl.
b) Nach I.3.3 gilt in Z: a irreduzibel ⇐⇒ a Primelement. Wir werden
später Ringe kennenlernen, in denen die Begriffe irreduzibel“und Prim”
”
element“ verschieden sind.
1.5 Bemerkung. Sei u ∈ R irreduzibel. Dann gilt:
a) Ist a ∈ R keine Einheit, so ist
a | u genau dann, wenn a ∼ u
b) Aus a ∼ u folgt: a ist irreduzibel.
Beweis.
a) a | u =⇒ a Einheit oder a ∼ u =⇒ a ∼ u
a ∼ u =⇒ a | u (gilt immer).
1.1
b) a ∼ u =⇒ u = εa, ε Einheit; u 6= 0, u 6∈ R× =⇒ a 6= 0 und a ist keine
Einheit.
d | a =⇒ d | u = εa =⇒ d Einheit oder d ∼ u =⇒ d Einheit oder
d ∼ a. Als ist d kein echter Teiler von a.
1.6 Bemerkung. Sei p ∈ R ein Primelement. Dann gilt:
a) Aus p | a1 . . . an folgt p | a1 oder p | a2 oder . . . oder p | an .
b) p ist irreduzibel.
c) Jedes p0 ∼ p ist ebenfalls ein Primelement.
Beweis.
a) ergibt sich induktiv aus der Definition.
b) d | p =⇒ p = dq und q | p =⇒ p | d oder p | q.
(i) p | d und d | p =⇒ p ∼ d;
(ii) p | q und q | p =⇒ p ∼ q
=⇒ d Einheit. Also hat p keine echten Teiler.
138
1.3
1.3
c) p0 | ab =⇒ p | ab =⇒ p | a oder p | b =⇒ p0 | a oder p0 | b.
1.7 Bemerkung. Seien p1 , . . . , pr und q1 · . . . · qs Primelemente und
a = p1 · . . . · pr ∼ q1 · . . . · qs = b
Dann ist r = s und bei geeigneter Numerierung ist
pj ∼ qj für j = 1, . . . , r
1.6.b)
Beweis. (Induktion nach r.) r = 1 : p1 prim =⇒ p1 hat keine echten Teiler
=⇒ s = 1. Analog:s = 1 =⇒ r = 1.
Schluß von r − 1 auf r. r ≥ 2 =⇒ s ≥ 2 (wie gerade gesehen).
1.6a)
1.5
p1 | b =⇒ Es gibt ein j mit p1 | qj , o.E. j = 1 =⇒ p1 ∼ q1 . Kürzen
ergibt p2 · . . . · pr ∼ q2 · . . . · qs . Nach Induktionsannahme ist r = s und
pj ∼ qj , j = 2, . . . , s (Numerierung geeignet gewählt).
Faktorielle Ringe. Ein nullteilerfreier Ring heißt faktoriell, wenn sich jede
Nichteinheit a ∈ R\{0} als Produkt von Primelementen schreiben läßt:
a = p1 · . . . · ps , s ≥ 1, p1 , . . . , ps Primelemente.
Beispiel. Z ist (nach I.3.1) ein faktorieller Ring.
1.8 Bemerkung. In einem faktoriellen Ring ist jedes irreduzible Element
schon ein Primelement.
Beweis. Sei a = p1 · . . . · ps , s ≥ 1 wie oben und a irreduzibel. Dann muß
s = 1 sein, da sonst a echte Teiler hätte.
Sei bis auf weiteres R ein faktorieller Ring. Wähle ein Repräsentatensystem
P für die Klassen assoziierter Primelemente, d.h.:
(1) P ist eine Menge von Primelementen von R.
(2) Für p 6= q aus P gilt p 6∼ q.
(3) Zu jedem Primelement π ∈ R gibt es ein p ∈ P mit π ∼ p.
Beispiel. R = Z : P = Menge der Primzahlen.
1.9 Satz. Sei P wie oben. Dann gilt:
139
a) Jedes a ∈ R\{0} hat eine Darstellung
(∗)
a = εpν11 · . . . · pνrr , wobei r ≥ 0, ε ∈ R×
mit paarweise verschiedenen p1 , . . . , pr ∈ P und ν1 , . . . , vr ∈ N>0 .
b) Die Darstellung (∗) ist bis auf die Reihenfolge der Faktoren p1 , . . . , pr
eindeutig durch a festgelegt.
Beweis.
a) Ist a = ε Einheit, so wähle man r = 0. Ist a 6= 0 und keine Einheit, so
schreibt sich a in der Form a = π1 . . . πs mit Primelementen π1 , . . . πs .
Jedes π ∈ {π1 , . . . , πs } ist von der Form εp; p ∈ P. Setze in a = π1 ·. . .·πs
ein. Durch Zusammenfassen der Einheiten und der gleichen Faktoren
aus P zu Potenzen erhält man die gewünschte Darstellung.
b) folgt sofort aus 1.7.
Wir nennen (∗) die kanonische Zerlegung von a.
1.10 Korollar. Sei a = εpν11 · . . . · pνrr 6= 0 in kanonischer Zerlegung gegeben.
Die Teiler von a sind dann die Elemente
(∗∗)
d = δpν11 · . . . · pνrr mit δ ∈ R× und 0 ≤ µj ≤ νj , j = 1, . . . , r.
Beweis. Die Elemente der Form (∗∗) sind offenbar Teiler von a. Sei umgekehrt a = dq. Ist p ∈ P und p | d, so gilt auch p | a, also p ∈ {p1 , . . . , pr }.
Entsprechendes gilt für q. Also gibt es δ, δ 0 ∈ R× mit
d = δpµ1 1 · . . . · pµr r , q = δ 0 pλ1 1 . . . pλr r und folglich
1
a = δδ 0 pµ+λ
· . . . · pµr r +λ1 . Aus 1.9b) folgt
1
ε = δδ 0 , µj + λj = νj , j = 1, . . . , r =⇒ 0 ≤ µj ≤ νj .
Definition. x1 , . . . , xn , u ≥ 1 heißen teilerfremd, wenn sie keinen gemeinsamen Primteiler haben. Sie heißen paarweise teilerfremd, wenn je zwei
von ihnen teilerfremd sind.
1.11 Korollar. Seien x, y, z ∈ R\{0}, n ∈ N, u ≥ 1. Es gilt
a) Ist x · y = z n und sind x, y teilerfremd, so ist x von der Form ε · v n , ε
Einheit, v ∈ R.
140
b) Ist xn y = z n , so ist y = v n mit v ∈ R.
Beweis. In jedem Fall ist jeder Primteiler von x bzw. y ein Primteiler von
z. Man hat also p1 , . . . , pr ∈ P (paarweise verschieden) und Darstellungen
x=ε
r
Y
µ
pj j , y
=δ
j=1
r
Y
ν
pj j
und z = δ
0
r
Y
pλj r
j=1
j=1
mit Einheiten ε, δ, δ 0 und µj , νj , λj ∈ N.
a) Da x und y teilerfremd sind gilt o.E. µ1 = . . . = µs = 0 und
νs+1 = . . . = νr = 0, 0 ≤ s ≤ r. Es folgt
n
z = xy = εδ
r
Y
µ
pj j
·
j=s+1
s
Y
ν
pj j
=δ
0n
j=1
r
Y
nλj
pj
j=1
Nach 1.9 folgt µj = nλj , j = s + 1, . . . , r und x = ε
r
Q
j=s+1
b)
δ
0n
r
Y
nλ
pj j
n
n
n
=z =x y=ε δ
j=1
r
Y
!n
λ
pj j
.
nµj +νj
pj
j=1
0
Nach 1.6 folgt: δ n = εn δ ∈ R× und nλj = nµj + νj . Es folgt n(λj − µj ) = νj ,
r
Q
ν0
insbesondere λj − µj = νj0 ∈ N mit v := δε−1
pj j ∈ R.
Es folgt v · x = δ
r
Q
j=1
j=1
λ
pj j = z und v n xn = z n = y · xn , also y = v n .
Definition. Sei p ∈ p. Für a ∈ R\{0} sei
(
0, falls p - a
vp (a) :=
der Exponent von p in der kanonischen Zerlegung von a, falls p | a
vp (a) heißt Ordnung von a an der Primstelle p.
1.12 Regel. Für a, b ∈ R\{0} gilt:
a) a ∈ R× =⇒ vp (a) = 0 für alle p ∈ p
141
b) vp (ab) = vp (a) + vp (b)
c) Ist a + b 6= 0, so gilt vp (a + b) ≥ Min{vp (a), vp (b)}.
Ist zudem vp (a) < vp (b), so gilt vp (a + b) = vp (a).
Der einfache Beweis wird dem Leser überlassen.
142
§2 Euklidische Ringe
Sei R ein Integritätsbereich.
Ideale. Eine nicht leere Teilmenge a von R heißt Ideal, wenn gilt:
(i) Aus a, b ∈ a folgt a + b ∈ a.
(ii) Aus a ∈ a folgt xa ∈ a für alle x ∈ R.
2.1 Bemerkung.
a) Die Null gehört zu jedem Ideal.
b) Seien a1 , . . . , an ∈ R, n ≥ 1. Dann ist die Menge
{x1 a1 + . . . + xn an | x1 , . . . , xn ∈ R} ein Ideal, das von {a1 , . . . , an }
”
erzeugte Ideal“.
Beispiel. Ist m ∈ Z, so ist mZ = {mx | x ∈ Z} das von m erzeugte Ideal
von Z.
Definition. Ein Ideal a ⊆ R heißt Hauptideal, wenn es von einem Element
a erzeugt wird, d.h. wenn a = {ax | x ∈ R}. Schreibe dann für a auch aR“
”
oder Ra“ oder (a)“ .
”
”
Beispiele. {0} = (0) ist ein Hauptideal, das Nullideal. R = (1) ist ebenfalls
ein Hauptideal.
Hauptidealringe. R heißt Hauptidealring, wenn jedes Ideal von R ein
Hauptideal ist.
2.2 Satz. Z ist ein Hauptidealring.
Beweis. Sei a ⊆ Z ein Ideal, a 6= (0). Dann enthält a ein kleinstes positives
Element a, denn x ∈ a =⇒ −x ∈ a.
Behauptung: a = (a)
Beweis. Sei b ∈ a beliebig. Dividiere b durch a mit Rest.
b = qa + r
,
0≤r<a
Da a ein Ideal ist und a ∈ a, ist auch (−q)a ∈ a, somit auch
r = b + (−q)a ∈ a, 0 ≤ r < a. Nach Wahl von a muß r = 0 sein, d.h.
b = qa ∈ a. Damit ist a ⊆ (a). Umgekehrt ist (a) ∈ a, denn xa ∈ a für alle
x ∈ R, da a ∈ a.
143
2.3 Satz. Jeder Hauptidealring ist faktoriell.
Beweis. Wir zeigen zunächst
2.4 Lemma. Sei R ein Hauptidealring und sei (an )n∈N eine Folge von Elementen aus R, so daß
(a0 ) ⊆ (a1 ) ⊆ (a2 ) ⊆ . . . ⊆ (an ) ⊆ (an+1 ) ⊆ . . .
Dann gibt es ein n ∈ N mit (an+m ) = an für alle m ∈ N. ( In einem Haupt”
idealring wird jede aufsteigende Kette von Idealen stationär.“)
S
Beweis von 2.4. Sei a :=
(an ) = {x | Es gibt ein n ∈ N mit x ∈ (an )}.
N
n∈
Behauptung. a ist ein Ideal.
Beweis. 0 ∈ (a0 ) ⊆ a, also ist a 6= ∅.
Sei x ∈ a, r ∈ R beliebig. Dann gibt es ein n ∈ N mit
x ∈ (an ) =⇒ rx ∈ (an ) ⊆ a. Sind x, y ∈ a, so gibt es n, m ∈ N mit
x ∈ (an ), y ∈ (am ), o.E. n ≤ m =⇒ x, y ∈ (am ) =⇒ x + y ∈ (am ) ⊆ a.
Da R ein Hauptidealring
ist, gibt es ein a ∈ R mit a = (a).
S
(an ) =⇒ Es gibt ein n ∈ N mit a ∈ (an ) =⇒ (a) ⊆ (an )
=⇒ a ∈ a =
N
n∈
=⇒ a = (a) ⊆ (an ) ⊆ a und a = (an ) =⇒ (an ) ⊆ (an+m ) ⊆ a ⊆ (an ) und
(an ) = (an+m ), für alle m ∈ N.
Beweis von 2.3. Sei a ∈ R\{0}. Es ist zu zeigen:
(∗) Es gibt u ∈ R× und Primelemente p1 , . . . , pr , so daß
a = up1 · . . . · pr
(r ≥ 0)
Jede Einheit hat eine solche Darstellung mit r = 0. Jedes Primelement hat
eine solche Darstellung mit u = 1, r = 1.
Beweis von (∗). Sei M := {(a) | a hat keine Zerlegung (∗)}.
Angenommen 2.3 wäre falsch, d.h. M 6= ∅.
Behauptung. In M gibt es ein maximales Element (a0 ).
D.h.: (a0 ) ∈ M und aus (a0 ) ( (b) folgt (b) 6∈ M.)
144
Beweis. Wäre die Behauptung falsch, so könnte man in M sukzessive eine
echt aufsteigende Kette von Idealen
(a0 ) ( (a1 ) ( (a2 ) ( . . .
bilden, im Widerspruch zu 2.4.
Sei also (a0 ) in M ein maximales Element. Dann ist wegen (a0 ) ∈ M das
Element a0 keine Einheit. Nach dem folgenden Lemma ist a0 auch nicht
irreduzibel.
2.5 Lemma. In einem Hauptidealring sind irreduzible Elemente bereits
Primelemente.
Beweis. Sei u irreduzibel und u | ab. Es ist zu zeigen, daß u | a oder u | b.
Setze a = {r | u teilt ar}. Nach 1.1 ist a ein Ideal. Ferner gilt b ∈ a und
u ∈ a. Da R ein Hauptidealring ist, gibt es ein d ∈ R mit a = (d). Wegen
1.5
u ∈ a folgt u = rd, d.h. d | u =⇒ d ∼ u oder d ist Einheit
(i) d Einheit: u | ad wegen d ∈ a und ad ∼ a =⇒ u | a.
(ii) d ∼ u : b ∈ a = (d) =⇒ d | b. Wegen d ∼ u folgt u | b.
Da nun a0 weder Einheit noch irreduzibel ist, gilt
a0 = bc mit b 6∈ R× und c 6∈ R×
Es folgt (a0 ) ( (b) und (a0 ) ( (c), denn b 6∈ (a0 ) und c 6∈ (a0 ). Nach Wahl
von (a0 ) gilt daher (b) 6∈ M und (c) 6∈ M . Also haben b und c Zerlegungen
der Form (∗), somit auch a0 = bc, Widerspruch zu (a0 ) ∈ M .
Also ist die Ausnahme M 6= ∅ falsch und 2.3 ist bewiesen.
Euklidische Ringe. Der Ring R heißt euklidisch, wenn gilt:
Es gibt eine Funktion ν : R\{0} −→ N, so daß es zu je zwei Elementen
a, b ∈ R, b 6= 0 Elemente q, r ∈ R gibt mit
(i) a = bq + r
(ii) r = 0 oder ν(r) < ν(b)
ν nennt man auch eine euklidische Norm für R.
Beispiele.
145
a) ν : Z −→ N, n 7−→ |n| ist nach dem Satz über die Division mit Rest in
Z eine euklidische Norm für Z. Also ist Z ein euklidischer Ring.
b) Sei K ein Körper und R = K[X] der Polynomring über K. Dann ist
die Abbildung Grad : R\{0} −→ N eine euklidische Norm für R (nach
II.4.2) und R = K[X] ist ein euklidischer Ring.
2.6 Satz. Jeder euklidische Ring ist ein Hauptidealring.
Beweis. (fast wörtlich der Beweis von 2.2). Sei ν die Norm von R und a ⊆ R
ein von (0) verschiedenes Ideal. Wähle ein a ∈ a mit minimaler Norm. Es
ist (a) ⊆ a.
Sei umgekehrt b ∈ a, b 6= 0. Dividiere b durch a mit Rest:
b = qa + r,
r = 0 oder ν(r) < ν(a).
Wäre r 6= 0, so wäre wegen a, b ∈ a auch r = b + (−q)a ∈ a, r 6= 0 und
ν(r) < ν(a), im Widerspruch zur Wahl von a. Also ist r = 0, d.h.
b = qa ∈ (a). Damit ist auch a ⊆ (a) und insgesamt (a) = a.
2.7 Korollar. Jeder euklidische Ring ist faktoriell. Insbesondere läßt sich
jedes Polynom f ∈ K[X] über einem Körper K in ein Produkt von Primpolynomen zerlegen. Die Einheiten von K[X] sind nach II.2.5 die Einheiten von
K. Offenbar sind die Polynome X −λ, λ ∈ K irreduzibel, also Primpolynome
nach 2.6.
Beispiel. In der Analysis lernt man den sogenannten Fundamentalsatz
der Algebra. Jedes nicht konstante Polynom
f (X) = a0 + a1 X + . . . + an X n , n ≥ 0, an 6= 0
mit komplexen Koeffizienten a0 , a1 , . . . , an besitzt eine komplexe Nullstelle y,
d.h.
f (y) = a0 + a1 y + . . . + an y n = 0. Dividiere f (X) durch X − y mit Rest und
erhalte
f (X) = q(X) · (X − y) + r
wobei q(X) ∈ C[X] und r = 0 oder r 6= 0 und Grad r = 0, d.h. r ∈ C. Setze
y für die Unbestimmte X ein, erhalte
0 = f (y) = q(y)(y − y) + r = r, also f (X) = q(X) · (X − y)
146
Daher sind die einzigen Primpolynome p(X) in C[X] die Polynome vom Grad
1, p(X) = aX + b, a 6= 0. Es ist p(X) ∼ X − λ, λ = − ab . Nach 2.7 hat f (X)
daher eine Zerlegung
f (X) = an
r
Y
(X − λm )vm , ν1 + . . . + νr = n, λi 6= λj für i 6= j
m=1
Offenbar sind λ1 , . . . , λr die Nullstellen, ihre Anzahl ist ≤ n = Grad f .
147
§3 Imaginär – quadratische Zahlbereiche
In diesem Abschnitt werden wir Integritätsbereiche kennenlernen, die nicht
faktoriell sind.
Sei m > 1 eine quadratfreie natürliche Zahl, d.h. m = 1 oder
√
m = p1 · . . . · pr mit paarweise verschiedenen Primzahlen. m ∈ R sei die
(positive) Quadratwurzel aus m. Setze
√
√
Z[i m] := {x + yi m | x, y ∈ Z} ⊆ C
√
Die Menge Z[i m] bildet die Knoten eines Gitternetzes in der Gaußschen
Zahlenebene, dessen
√ Lage in der folgenden Skizze angedeutet werden soll. Die
Elemente von Z[i m] werden dabei durch einen Kringel ◦“ angedeutet.
”
iR
√
2i m
√
4 + 2i m
√
i m
0
1
148
2
R
√
Die
eines Netzes mit Maschenhöhe
√ Punkte von Z[i m] sind also die Knoten
√
m und Maschenbreite 1 und Z = Z[i m] ∩ R.
3.1 Bemerkung.
√
a) R = Z[i m] ist ein Unterring von C.
b) Ist z ∈ R, so ist auch z ∈ R. (Abgeschlossenheit gegenüber der Konjugation.)
√
√
c) K := Q[i m] = {z | z = x + yi m, x, y ∈ Q} ist ein Körper. Genauer
ist K = {α · β −1 | α, β ∈ R, β 6= 0}.
Beweis.
b) ist klar.
a) Die Abgschlossenheit bzgl. der Addition ist klar.
√
√
√
(x + yi m)(x0 + y 0 i m) = (xx0 − yy 0 m) + (xy 0 + x0 y)i m
Also ist R abgeschlossen bzgl. der Multiplikation, weil dies für Z gilt.
c) Wie in a) und b) folgt, daß K ein Ring ist, der gegen Konjugation
z
abgeschlossen ist. Ferner ist zz = x2 + y 2 m ∈ Q,√somit z zz
= 1 und
z
∈ K, d.h., K ist sogar ein Körper. z = x + yi m ∈ K ist von der
zz
Form αn mit n ∈ N, α ∈ R, wenn n 6= 0 ein Hauptnenner von x, y ∈ Q
ist. Umgekehrt ist für β ∈ R\{0} auch β −1 ∈ K, somit αβ −1 ∈ K.
√
Die Ringe der Form Z[i m] wie oben nennen wir imaginär –quadratische
Zahlbereiche.
Die Norm auf C. Für z ∈ C setzen wir
N (z) := zz( Norm“ von z)
”
M.a.W.: N (z) = |z|2 .
√
√
Ist α = x + yi m ∈ Z[i m], so ist
√
√
N (α) = (x + iy m)(x − iy m) = x2 + y 2 m ∈ N
Daher induziert N eine Abbildung
√
ν : Z[i m]\{0} −→ N, r 7−→ N (r)
√
3.2 Eigenschaften der Norm: Sei R = Z[i m], m wie oben.
149
a) N (αβ) = N (α)N (β) für α, β ∈ C.
b) N (α) ≥ 0; N (α) = 0 ⇐⇒ α = 0 gilt für alle α ∈ C.
c) α ∈ R =⇒ N (α) ∈ N
d) Für α, β ∈ R gilt: Aus α | β folgt N (α) | N (β)
e) α ∈ R ist genau dann eine Einheit, wenn N (α) = 1.
Genauer ist R× = {±1, ±i}, falls m = 1 und R× = {±1} sonst.
f) Ist N (α) eine Primzahl für ein α ∈ R, so ist α irreduzibel in R.
Beweis.
a) Gilt, weil αβ = αβ.
b) folgt aus der entsprechenden Aussage über |α|.
c) wurde bereits gezeigt.
d) folgt aus a).
e) αβ = 1 =⇒ N (α)N (β) = N (αβ) = N (1) = 1; wegen√N (α), N (β) ∈ N
folgt N (α) = N (β) = 1. Sei umgekehrt α = x + yi m, x, y ∈ Z mit
N (α) = x2 + my 2 = 1.
1) m ≥ 2 :=⇒ y = 0 =⇒ x = ±1 =⇒ α = ±1 Einheit
2) m = 1 :=⇒ (y = 0 und x = ±1) oder (y = ±1 und x = 0), also
α = ±1 oder α = ±i. Dies sind Einheiten in Z[i].
f) Sei N (α) eine Primzahl und α = βγ mit β, γ ∈ R.
e)
p = N (α) = N (β)N (γ) =⇒ N (β) = 1 oder N (γ) = 1 =⇒ β Einheit
oder γ Einheit. Also ist α irreduzibel.
3.3 Korollar.
√
a) Für jedes m ist 2 kein Primelement in Z[i m].
√
b) Für m ≥ 3 ist 2 irreduzibel in Z[i m].
√
Insbesondere ist nach 1.8 der Ring Z[i m] für m ≥ 3 nicht faktoriell.
Beweis.
150
a) m(m+1) ist gerade, d.h. 2 | m(m+1).
Wäre
so
√
√2 in R ein Primelement,
√
(m+i m) gelten:
müßte wegen
√ m(m+1) = (m+i
√ m)(m−i m) = N √
√
2 | m + i m oder 2 | m − i m in R. Aus 2(x + iy m) = m ± i m
folgt 2y = ±1, y ∈ Z, Widerspruch.
3.2d)e)
b) α k 2 =⇒ N (α) k√
N (2) = 4 =⇒ N (α) = 2.
Es ist α = x + iy m, x, y ∈ Z; m ≥ 3 =⇒ N (α) = x2 + my 2 6= 2,
Widerspruch.
3.4 Satz. Der Ring der ganzen Gaußschen Zahlen Z[i] ist faktoriell.
(In der Gaußschenzahlenebene entsprechen die Elemente von Z[i] gerade den
ganzzahligen Gitterpunkten.)
Beweis. Sei R = Z[i] und
ν : R\{0} =⇒ N
x + iy 7−→ N (x + iy) = x2 + y 2 (x, y ∈ Z)
Zu zeigen: ν in eine euklidische Normfunktion.
Seien a, b ∈ R, b 6= 0. Dann ist ab ∈ C. Schreibe ab ist der Form ab = x + iy
mit x, y ∈ R. Seien m, n die ganzen Zahlen die x bzw. y am nächsten liegen,
d.h. : | x − m |≤ 21 und |y − n| ≤ 12 .
Setze q := m + in und z := ab = x + iy. Dann gilt a = bz und
N (z − q) = N ((x − m) + i(y − n)) = (x − m)2 + (y − n)2 ≤ 21 .
Setze r := a − bq ∈ R. Ist r 6= 0, so gilt
ν(r) = N (a − bq) = N (bz − bq) = N (b)N (z − q) ≤ 12 N (b) < ν(b),
da N (b) > 0 wegen b 6= 0.
√
3.5 Satz. Der Ring Z[i 2] ist faktoriell.
Beweis. Analog zu 3.4 zeigt man, daß
√
√
ν : Z[i 2] =⇒ N, α = x + iy 2 7−→ x2 + 2y 2
eine euklidische Normfunktion ist. (Übungsaufgabe.)
Die Primelemente von Z[i] nennt man Gaußsche Primzahlen.
Frage: Welche Primzahlen p ∈ N sind Gaußsche Primzahlen?
Beispiel: N (1 + i) = N (1 − i) = (1 + i)(1 − i) = 2 ist eine Primzahl in
N. Wegen 3.2 und 3.4 sind daher 1 + i und 1 − i Gaußsche Primzahlen.
151
Ferner hat 2 in Z[i] echte Teiler, ist also keine Gaußsche Primzahl. Es gilt
(−i)(1 + i) = 1 − i, also ist (1 + i) ∼ (1 + i) und 2 = (1 + i)(1 + i).
3.6 Satz. Sei p eine ungerade Primzahl. Dann gibt es zwei Möglichkeiten.
Entweder
a) p ist eine Gaußsche Primzahl, oder
b) es gibt eine Gaußsche Primzahl q, so daß p = qq = N (q). Die Zahl q ∈
Z[i] ist dann ebenfalls eine Gaußsche Primzahl, welche nicht assoziiert
zu q ist.
Beweis. p hat in R = Z[i] eine Primfaktorzerlegung p = q1 · . . . · qr ; qi ∈ R
Primelement. Es ist r ≥ 1 da N (p) = p2 > 1 und
p2 = N (p) = N (q1 ) · . . . · N (qr ). Wegen N (qi ) 6= 1 folgt r = 1 oder r = 2.
1. Fall: r = 1 =⇒ p = q1 ist eine Gaußsche Primzahl
2. Fall: r = 2 =⇒ p = q1 q2 ; p2 = N (q1 )N (q2 ) =⇒ N (q1 ) = N (q2 ) =
p; N (q1 ) = q1 q 1 = p = q1 q2 , also ist q 1 = q2 ebenfalls eine Gaußsche Primzahl
und p = q1 q 1 .
Angenommen q1 = x + iy ∼ q 1 = x − iy, d.h. q = uq, u ∈ R× = {±1, ±i}.
1. Fall: u = ±1 =⇒ ±x ± iy = x − iy =⇒ y = 0 oder x = 0
=⇒ p = q1 q 1 = x2 oder p = q1 q 1 = y 2 , Widerspruch.
2. Fall: u = ±i : ±ix ∓ y = x − iy =⇒ x = −y oder x = y
=⇒ p = q1 q 1 = x2 + y 2 = 2x2 , Widerspruch, denn p war ungerade.
3.7 Satz. Eine ungerade Primzahl p ∈ N ist genau dann eine Gaußsche
Primzahl, wenn p ≡ 3 mod 4.
I.8.1
Beweis. p ≡ 3 mod 4 =⇒ p ist nicht Summe von zwei Quadraten
=⇒ p 6= N (α) für alle α = x + iy ∈ Z[i]. Nach 3.6 ist daher p eine Gaußsche
Primzahl.
I.7.13
p ≡ 1 mod 4 =⇒ Es gibt ein x ∈ Z mit x2 ≡ −1 mod p =⇒ p | x2 + 1 =
(x + i)(x − i). Wäre p eine Gaußsche Primzahl so wäre p | x + i oder p | x − 1
in Z[i]. Beides ist nicht möglich:
p(a + bi) = x + i =⇒ pb = 1, Widerspruch
p(a + bi) = x − i =⇒ pb = −1, Widerspruch.
152
§4 Der Ring
Z[− 12
+i
√
3
2 ]
√
Wir haben gesehen, daß der imaginär–quadratische Zahlbereich S = Z[i 3]
kein faktorieller Ring ist. Für spätere Anwendungen werden wir S in einen
möglichst kleinen faktoriellen Unterring R ⊆ C einbetten. Sei
√
1
3
ζ := − + i
und R := Z[ζ] = {a + bζ | a, b ∈ Z} ⊆ C
2
2
Lage der Punkte ζ, ζ 2 und ζ 3 in der Zahlenebene.
= − 12 und sin 2π
=
Es ist cos 2π
3
3
√
3
,
2
d.h.
ζ = cos 2π
+ i sin 2π
, | ζ |= 1 und Arg ζ =
3
3
2π
3
= 120◦ . Es folgt | ζ 2 |=| ζ 3 |= 1
und Arg ζ 2 = 120◦ , Arg ζ 3 = 360◦ .
Somit liegen ζ, ζ 2 und ζ 3 auf dem Einheitskreis E : x2 + y 2 = 1, ihre Argumente sind 120◦ , 240◦ , 360◦ . Insbesondere ist ζ 3 = 1.
ζ
.
.
ζ3
.
ζ2
Die Punkte ζ, ζ 3 , ζ 3 = 1 bilden somit ein gleichseitiges Dreieck. Es folgt weiter
153
ζ 2 = ζ = − 12 − i
(1)
√
3
, ζ3
2
= 1 und ζ + ζ 2 = ζ + ζ = −1, d.h.
1 + ζ + ζ 2 = 0, ζ 3 = 0, ζ 2 = ζ −1 = ζ
Skizze der Punktegitter S und R in der Gaußschen Zahlenebene C.
Kennzeichne die Punkte von S mit ד die von R mit ◦“ und die Einheiten
”
”
von R mit ∗“.
”
√
i 3
ζ
*
*
*
*
0
*
1
*
R ist ein schiefwinkliges Gitter und S ein rechtwinkliges mit doppelt so
großem Inhalt. Die Einheiten von R bilden ein regelmäßiges Sechseck (folgt
aus 4.2).
4.1 Bemerkung. R ist ein Ring, S ⊆ R und aus z ∈ R folgt z ∈ R.
Beweis. Die Abgeschlossenheit bzgl. der Addition ist klar. Nach (1) ist
ζ = ζ 2 = −1 − ζ, somit ist R abgeschlossen bzgl. der Konjugation.
(a
√+ bζ)(c + dζ) = ac + (ad + bc)ζ + bd(−1 − ζ) ∈ R.
i 3 = 1 + 2ζ ∈ R, somit ist S ⊆ R.
154
Als nächstes soll gezeigt werden, daß R ein faktorieller Ring ist. Sei wie zuvor
N (α) = αα für α ∈ C.
4.2 Satz.
a) N (αβ) = N (α)N (β) für alle α, β ∈ C.
b) N (α) ≥ 0; N (α) = 0 ⇐⇒ α = 0 (α ∈ C)
c) α ∈ R =⇒ N (α) ∈ N
d) α ∈ R× ⇐⇒ N (α) = 1, falls α ∈ R
e) R× = {±1, ±ζ, ±ζ 2 }
Beweis. a) und b) wurden in 3.2 gezeigt.
c) N (a + bζ) = (a + bζ)(a + bζ) = a2 + ab(ζ + ζ) + b2 ζζ 2 =
= a2 − ab + b2 ∈ N, falls a, b ∈ Z
d) Die Richtung =⇒“ ergibt sich wie in 3.2. Umgekehrt folgt aus αα =
”
N (α) = 1, daß α ∈ R× .
e) Wegen ζζ 2 = 1 sind die sechs angegebenen
Zahlen Einheiten. Sei um√
gekehrt α ∈ R× . Schreibe α = 12 (c + di 3) mit c, d ∈ Z. Dann ist nach
d) 1 = αα = 14 (c2 + 3d2 ). Dies ist nur möglich, wenn d = 0 und c = ±2
oder d = ±1 und c = ±1. Also hat R auch höchstens sechs Einheiten.
Wegen ζ 3 = 1 gilt ε6 = 1 für alle ε ∈ R× .(R× besteht also aus den
sogenannten sechsten Einheitswurzeln“).
”
4.3 Satz. R ist ein euklidischer Ring mit euklidischer Norm N , d.h.: Sind
α, β ∈ R, β 6= 0 vorgegeben, so gibt es q, ρ ∈ R mit
α = βq + ρ und N (ρ) < N (β)
Genauer gilt sogar: N (ρ) ≤ 34 N (β).
Insbesondere ist R ein faktorieller Ring.
Zum Beweis benötigen wir ein
4.4 Lemma. Jedes z ∈ C schreibt sich in der Form
z = x + y · ζ mit x, y ∈ R
155
Beweis. Es ist z = u + iv mit u, v ∈ R
1 1√
1
2
ζ=− +
3i =⇒ i = √ + √ ζ =⇒ z =
2 2
3
3
1
2
u+ √ v + v· √ ζ
3
3
Beweis von 4.3 Sei z := αβ ∈ C. Schreibe z in der Form z = x + yζ mit
x, y ∈ R. Wähle wie im Beweis von 3.4 ganze Zahlen m, n mit |x − m| ≤ 12
und |y − n| ≤ 12 und setze q = m + nζ und ρ = α − qβ. Dann gilt
α = βq + ρ und z − q = (x − m) + (y − n)ζ. Es folgt
N (ρ) =
=
=
=
N (α − qβ) = N (zβ − qβ) = N (z − q)N (β) =
((x − m) + (y − n)ζ)(x − m) + (y − n)ζ 2 ) · N (β) =
((x − m)2 + (x − m)(y − n)(ζ + ζ 2 ) + (y − n)2 )N (β) =
((x − m)2 − (x − m)(y − n) + (y − n)2 )N (β) ≤
1 1 1
≤ ((x − m)2 + |x − m||y − n| + (y − n)2 )N (β) ≤ ( + + )N (β)
4 4 4
3
≤
N (β) < N (β), da β 6= 0.
4
Zum Beweis des großen Fermatischen Satzes im Fall n = 3 benögigen
wir die
√
1
1
folgenden Regeln für das Rechnen am Ring RR = Z[− 2 + i 2 3].
Wir nennen Zahlen r, s ∈ R kongruent modulo 3R wenn r − s ∈ 3R.
4.5 Regel.
a) Ist N (α) eine Primzahl, so ist α in R ein Primelement.
b) 1 − ζ ist ein Primelement. Es hat folgende Eigenschaften:
1 − ζ = 1 − ζ = 1 − ζ 2 = (1 − ζ)(1 + ζ) = −ζ 2 (1 − ζ) ∼ 1 − ζ; also ist
N (1 − ζ) = −ζ 2 (1 − ζ)2 = 3 = (1 − ζ)(1 − ζ).
Insbesondere ist 3 ∼ (1 − ζ)2 , da −ζ 2 ∈ R× .
c) Ist a + bζ ∈ R mit a, b ∈ Z so sind a bzw. b kongruent zu +1, −1 oder 0
modulo 3Z. M.a.W.: a + bζ ist modulo 3R kongruent zu einer der neun
Zahlen aus N = {±1, ±ζ, ±(1 + ζ), ±(1 − ζ), 0}.
Damit ist jedes α ∈ R entweder zu einer Einheit, oder zu ±(1 − ζ) oder
zu 0 kongruent modulo 3R.
156
d) Jeder Kubus in R (das ist eine Zahl x3 mit x ∈ R) ist modulo 3R
kongruent zu 1, −1 oder 0, d.h.:
Für jedes x ∈ R schreibt x3 in der Form
x3 = ν + 3c mit ν ∈ {±1, 0} und c ∈ R
e) Ist x ∈ R und u ∈ R× und gilt ux3 = ±1 modulo 3R, so ist
u = ±1 und
ux3 = (ux)3 ist ein Kubus.
Beweis.
a) Wie in 3.2 zeigt man, daß d irreduzibel ist, und somit nach 4.3 ein
Primelement.
b) Die angegebenen Identitäten rechne man nach. Wegen N (1 − ζ) = 3 ist
1 − ζ ein Primelement nach a).
c) ist klar.
d) Nach c) gilt: x ≡ ν mod 3R mit ν ∈ N .
Wie in Z folgt daraus x3 ≡ ν 3 mod 3R (nachrechnen), wobei (±1)3 =
±1, (±ζ)3 = ±1, (±ζ 2 )3 = ±1
b)
(1 − ζ)2 ∼ (1 − ζ)(1 − ζ) = N (1 − ζ) = 3 =⇒ ±(1 − ζ)2 ∈ 3R, d.h.
±(1 − ζ)3 ≡ 0 mod 3R.
e) Nach d) ist x3 ≡ ±1 mod 3R. Wie in Z folgt ±1 ≡ ux3 ≡ ±u mod
3R, d.h. u ≡ ±1 mod 3R. Ferner ist u eine Einheit. Nach 4.2 ist noch
auszuschließen, daß u = ±ζ oder u = ±ζ 2 .
Im Fall u ≡ 1 mod 3R schließt man wie folgt:
u = ζ =⇒ 1 − ζ ≡ 0 mod 3R =⇒ 1 − ζ = 3α =⇒
3 = N (1 − ζ) = N (3)N (α) = 9N (α), Widerspruch.
u = −ζ =⇒ 1 + ζ ≡ 0 mod 3R =⇒ 9 | N (1 + ζ) = N (−ζ 2 ) = 1,
Widerspruch
u = ζ 2 =⇒ 9 | N (1 − ζ 2 ) = N (1 − ζ)N (1 + ζ) = N (1 − ζ) = 3
Widerspruch.
u = −ζ 2 =⇒ 9 | N (1 + ζ 2 ) = N (−ζ) = 1 Widerspruch.
Analog schließt man für u ≡ −1 mod 3R.
157
Bemerkung. Im Beweis haben wir wiederholgt von den folgenden Regeln
Gebrauch gemacht, deren Beweis dem Leser überlassen bleibt:
Sei a ⊆ R ein Ideal (etwa a = 3R) und a, b ∈ R.
Definition. a ≡ b mod a := a − b ∈ a.
Regel. ≡ mod a ist eine Äquivalenzrelation und es gilt
a) a ≡ b mod a =⇒ a + c ≡ b + c mod a
b) a ≡ b mod a =⇒ ac ≡ bc mod a
c) a ≡ b mod a =⇒ an ≡ bn mod a
158
§5 Die Gleichung X3 + Y3 = Z3
Wie man sofort nachrechnet, gilt in jedem Ring R
(1) x3 + y 3 + z 3 = (x + y + z)3 − 3(x + y)(y + z)(z + x)
(2) x3 + y 3 = (x + y)(x2 − xy + y 2 ), insbesondere x + y | x3 + y 3 für alle
x, y, z ∈ R.
Sei nun wieder ζ = cos 2π
+ i sin 2π
wie in §4 und R = Z[ζ] ⊆ C. Dann
3
3
gilt für x, y ∈ C
(3) x3 + y 3 = (x + y)(x + yζ)(x + yζ 2 )
denn (x + yζ)(x + yζ 2 ) = x2 + xy(ζ + ζ 2 ) + y 2 ζ 3 = x2 − xy + y 2
5.1 Satz. (Fermat, Euler) Es gibt keine a, b, c ∈ R\{0} mit
(∗)
a3 + b3 + c3 = 0
5.2 Korollar. Die Gleichung X 3 + Y 3 = Z 3 hat keine Lösung mit positiven
ganzen Zahlen x, y, z.
Sonst wäre nämlich a = x, b = y, c = −z ∈ R\{0} eine Lösung von (∗).
Beweis von 5.1.
Beweis idee“: Wäre a3 + b3 = c3 , wobei a, b, c ∈ R\{0} (o.E.) teilerfremd
”
sind, so folgt aus (1) und (3)
(a + b + c)3 = 3(a + b)(b + c)(c + a), (−c)3 = (a + b)(a + bζ)(a + bζ 2 ).
Aus der Tatsache, daß R faktoriell ist, schließt man, daß dies nicht sein kann.
Bevor wir mit dem eigentlichen Beweis von 5.1 beginnen, zeigen wir
5.3 Lemma. Ist a3 + b3 + c3 = 0 für teilerfremde Zahlen a, b, c ∈ R(\{0}, so
wird genau eine der drei Zahlen vom Primelement 1 − ζ geteilt.
Beweis. Nach (1) gilt in R wegen a3 + b3 + c3 = 0
(a + b + c)3 = 3(a + b)(b + c)(c + a) ∼ (1 − ζ)2 (a + b)(b + c)(c + a)
Da 1 − ζ ein Primelement ist folgt (1 − ζ) | a + b + c und daher
(1 − ζ)3 | (a + b + c)3 ∼ (1 − ζ)2 (a + b)(b + c)(c + a).
159
Kürzen ergibt (1 − ζ) | (a + b)(b + c)(c + a) und da 1 − ζ ein Primelement von
R ist, folgt, daß 1 − ζ einen der drei Faktoren teilt, o.E. (1 − ζ) | a + b. Nach
(2) ist a + b | a3 + b3 = −c3 . Insgesamt ergibt sich 1 − ζ | c3 , somit 1 − ζ | c.
Da aber a, b, c teilerfremd sind und a3 + b3 = −c3 , folgt:
a, b, c sind paarweise teilerfremd, daher 1 − ζ - a und 1 − ζ - b.
Beweis von 5.1 durch Widerspruch.
Angenommen es gäbe a, b, c ∈ R\{0} mit a3 + b3 + c3 = 0. Durch kürzen
erreicht man, daß a, b, c sogar teilerfremd sind. Wegen 5.3 ist dann die Menge
M = {(a, b, c) | a, b, c ∈ R\{0} teilerfremd, 1 − ζ | c, a3 + b3 + c3 = 0}
nicht leer. Für x ∈ R\{0} bezeichne v1−ζ (x) die Ordnung von x an der
Primstelle 1 − ζ.
Wir wählen ein Tripel (a, b, c) ∈ M , so daß die Ordnung v1−ζ (c) minimal ist.
Der Widerspruch wird sich ergeben, indem aus (a, b, c) ein weiteres Tripel
(a∗ , b∗ , c∗ ) ∈ M konstruiert wird mit v1−ζ (c∗ ) < v1−ζ (c).
Wegen (1 − ζ) - ab und (1 − ζ) | 3 gilt für a (entsprechend für b)
a 6≡ 0 mod 3R und a 6≡ ±(1 − ζ) mod 3R.
Aus 4.5 c ergibt sich: a ≡ ε mod 3R, wobei ε ∈ R eine Einheit ist, d.h.
ε ∈ {±1, ±ζ, ±ζ 2 }.
(i) Aus a ≡ ±ζ mod 3R folgt aζ 2 ≡ ±1 mod 3R.
(ii) Aus a ≡ ±ζ 2 mod 3R folgt aζ ≡ ±1 mod 3R.
Ferner ist a3 = (aζ 2 )3 = (aζ)3 .
Man kann daher a in (∗) durch aζ 2 oder aζ ersetzen und somit o.E. annehmen,
daß a ≡ ±1 mod 3R. Entsprechend verfährt man mit b, und kann b ≡ ±1
mod 3R annehmen. Ferner ist wegen 1 − ζ | c auch 3 ∼ (1 − ζ)2 | c2 | c3 , d.h.
c3 ≡ 0 mod 3R.
Wäre nun a ≡ b mod 3R, so auch a3 ≡ b3 mod 3R und 0 ≡ a3 +b3 +c3 ≡ 2a3 ≡
±2 ≡ ∓1 mod 3R. Insbesondere wäre 3 eine Einheit in R, im Widerspruch
zu 3 ∼ (1 − ζ)2 .
Also ist a 6≡ b mod 3R und wir können annehmen, daß a ≡ 1 mod 3R und
b ≡ −1 mod 3R.
Man kann also a und b in der Form
a = 1 + 3α, b = −1 + 3β mit α, β ∈ R
160
schreiben.
a + bζ
aζ + b
ζ 2 (a + b)
; B 0 :=
; C 0 :=
1−ζ
1−ζ
1−ζ
Setzt man a = 1 + 3α und b = −1 + 3β ein, so erhält man:
Wir setzen nun A0 :=
A0 =
(1 − ζ) + 3(α + βζ)
= 1 − ζ 2 (1 − ζ)(α + βζ),
1−ζ
B0 =
−(1 − ζ) + 3(αζ + β)
= −1 − ζ 2 (1 − ζ)(αζ + β) und
1−ζ
3ζ 2 (α + β)
C =
= −ζ(1 − ζ)(α + β), jeweils, da 3 = −ζ 2 (1 − ζ)2
1−ζ
0
Insbesondere sind A0 , B 0 , C 0 aus R. Ferner gilt
(5) A0 + B 0 + C 0 = −ζ(1 − ζ)(α + β)(1 + ζ + ζ 2 ) = 0.
3
(a + b)(a + bζ)(a + bζ 2 ) (3) a3 + b3
−c
0 0 0
(6) A B C =
=
=
= D03 .
(1 − ζ)3
(1 − ζ)3
1−ζ
Insbesondere gilt A0 6= 0, B 0 6= 0, C 0 6= 0.
Behauptung. A0 , B 0 und C 0 sind paarweise teilerfremd.
Beweis. Wegen A0 + B 0 + C 0 = 0 genügt es zu zeigen, daß A0 und B 0 teilerfremd sind. Nach Voraussetzung sind aber a und b teilerfremd. Ferner gilt
a = −ζA0 + ζ 2 B 0 , b = ζ 2 A0 − ζB 0 . Also müssen auch A0 und B 0 teilerfremd
sein.
Wegen (6) schreiben sich nach 1.11 die Elemente A0 , B 0 und C 0 in R in der
Form Einheit mal dritte Potenz“.
”
Wegen 1 − ζ | C 0 folgt daher (1 − ζ)3 | C 0 . Also gilt wegen 3 ∼ (1 − ζ)2
A0 + B 0 ≡ −C 0 ≡ 0 mod 3R, d.h. A0 ≡ −B 0 mod 3R
Ferner ist A0 ≡ 1 mod (1 − ζ)R, insbes. 1 − ζ - A0 .
Es folgt A0 ≡ u mod 3R mit u ∈ R× , somit B 0 ≡ −A0 ≡ −u mod 3R.
Wir setzen nun
A := u−1 A0 ,
B := u−1 B 0
und C := u0 C 0
Aus den entsprechenden Eigenschaften des Tripels (A0 , B 0 , C 0 ) folgt
161
A, B, C sind teilerfremd
A+B+C =0
−3
0
0
0
0
0
0
ABC = u A B C = ±A B C =
±c
1−ζ
3
= D3
Wie für A0 und B 0 schließt man auch hier:
A und B sind von der Form Einheit mal dritte Potenz“.
”
Ferner gilt
A ≡ u−1 A0 ≡ u−1 u ≡ 1 mod 3R
B ≡ u−1 B 0 ≡ u−1 (−u) ≡ −1 mod 3R
Nach 4.5 e) sind dann A und B sogar dritte Potenzen in R. Wegen ABC = D3
ist dann gemäß 1.11 b) auch C in R eine dritte Potenz:
A = a3∗ , B = b3∗ , C = c3∗ mit a∗ , b∗ , c∗ ∈ R\{0}.
Dabei ist
a∗ , b∗ und c∗ teilerfremd und a3∗ + b3∗ + c3∗ = 0.
Ferner ist 1 − ζ | C 0 ∼ C = c3∗ , und somit 1 − ζ | c∗ .
Also ist auch (a∗ , b∗ , c∗ ) ∈ M . Es gilt jedoch
3v1−ζ (c∗ ) = v1−ζ (c3∗ ) = v1−ζ (c) = v1−ζ (ABC) = v1−ζ
= 3v1−ζ
c
1−ζ
c
1−ζ
3 !
= 3(v1−ζ (c) − 1)
Es folgt v1−ζ (c∗ ) < v1−ζ (c), im Widerspruch zur Minimalität von v1−ζ (c).
162
Anhang B: Quadratische Irrationalzahlen
§1 Reel-quadratische Zahlkörper
Eine reelle Zahl x 6∈ Q heißt quadratische Irrationalzahl, wenn sie Lösung
einer quadratischen Gleichung
(1)
aX 2 − bX − c = 0, a 6= 0
mit rationalen Koeffizienten a, b und c ist.
Nach Multiplikation von (1) mit einer geeigneten rationalen Zahl kann man
erreichen, daß
(∗)
{a, b, c} ⊆ Z, a > 0 und ggT(a, b, c) = 1
1.1 Bemerkung. Die Gleichung (1) mit den Eigenschaften (∗) ist durch
die quadratische Irrationalzahl x eindeutig bestimmt. Man nennt sie die
Gleichung von x.
Beweis. Sei a0 x2 − b0 x − c0 = 0, wobei {a0 , b0 , c0 } ebenfalls die Bedingung (∗)
erfüllt. Dann ist
aa0 x2 −ab0 x−ac0 = 0 = a0 ax2 −a0 bx−ac0 und daher (a0 b−ab0 )x+(a0 c−ac0 ) = 0.
Wegen x 6∈ Q folgt a0 b = ab0 und a0 c = ac0 , also a = ggT(aa0 , ab0 , ac0 ) =
ggT (a0 a, a0 b, a0 c) = a0 . Somit ist a = a0 , b = b0 und c = c0 .
Die Lösungen von (1) sind bekanntlich die Zahlen
√
b
D
±
, wobei D := b2 + 4ac ∈ N
2a
a
kein Quadrat
√ ist. Man nennt D die Diskriminante von x. Es ist also √
x = u + v D mit u, v ∈ Q; die zweite Lösung von (1) ist dann x0 = u − v D.
Man nennt sie die zu x konjugierte quadratische Irrationalzahl. Sie hat
offenbar die gleiche Diskriminante wie x.
√
Sei umgekehrt y = u + √
v d mit rationalen Zahlen u, v und d ∈ Q kein
Quadrat. Setze y 0 = u − v d. Dann ist yy 0 = u2 − v 2 d ∈ Q, y + y 0 = 2u ∈ Q,
also ist
(X − y)(X − y 0 ) = X 2 − (y + y 0 )X + yy 0 = 0
163
eine quadratische Gleichung mit rationalen Koeffizienten und den Lösungen
y und y 0 . Es gilt also:
√
Die quadratischen Irrationalzahlen sind die Zahlen der Form x = u + v d,
wobei u, v ∈ Q, v 6∈ Q und d ∈ N √
kein Quadrat ist. Die zu x konjugierte
quadratische Irrationalzahl ist u − v d.
√
Sei nun d ∈ N kein Quadrat. Dann ist auch d 6∈ Q.
Wir definieren
√
√
K = Q[ d] = {u + v d | u, v ∈ Q} und
√
√
R = Z[ d] = {u + v d | u, v ∈ Z}
1.2 Satz.
√
a) K ist ein Q–Vektorraum mit Basis {1, d}.
b) K ist ein Körper und R ist ein Ring.
c) Die Konjugationsabbildung
√
√
σ : K −→ K, u + v d 7−→ u − v d
hat folgende Eigenschaften:
σ(a) = a für alle a ∈ Q.
σ(σ(x)) = x für alle x ∈ K und σ(r) ∈ R für alle r ∈ R.
σ(x + y) = σ(x) + σ(y) und σ(x · y) = σ(x) · σ(y) für alle x, y ∈ K.
σ(xn ) = σ(x)n für alle x ∈ K und n ∈ N.
d) Für u,√
v, n ∈ N>0 ist√mit geeigneten
√ũ, ṽ ∈ N>0 √
(u + v d)n = ũ + ṽ d und (u − v d)n = ũ − ṽ d
Beweis.
√
a) Offenbar
ist
K
ein
Q–Vektorraum,
der
von
1
und
√ d erzeugt wird.
√
= 0 mit
1 + d sind über Q linear unabhängig: Sei a · 1 + b d √
a, b ∈ Q. Aus b = 0 folgt a = 0. Wäre b 6= 0, so wäre d = − ab ∈ Q,
Widerspruch.
164
b) Die Abgeschlossenheit gegenüber Addition und Negation ist für K und
R klar. Abgeschlossenheit gegenüber der Multiplikation:
Seien (a, b), (a0 b0 ) ∈ Q2 . Dann gilt
√
√
√
(a + b d)(a0 + b0 d) = (aa0 + bb0 d) + (ab0 + a0 b) d ∈ K
und die rechte Seite liegt in R, falls a, b, a, b0 ∈ Z. Also sind K und R
Ringe. Für den Beweis von d) halten wir noch fest:
(∗) Aus (a, b), (a0 , b0 ) ∈ N2>0 folgt aa0 + bb0 d, ab0 + a0 b ∈ N>0 .
Für die Körpereigenschaft von K muß noch jedes x ∈ K\{0} invertier√
√
a)
bar sein: x = a + b d 6= 0 =⇒ a 6= 0 oder b 6= 0, also auch a − b d 6= 0.
√
√
Es folgt a2 − b2 d = (a + b d)(a − b d) 6= 0 und
√
√
a
b
−
d =1
(a + b d)
a2 − b2 d a2 − b2 d
√
c) Zur Basis B = {1, d} gehört der Basisisomorphismus
√
a
Φ : Q2 −→ K,
7−→ a + b d
b
1 0
ist invertierbar mit A · A = E2 . Daher ist
A=
0 −1
Φ−1
A
Φ
σ : K −→ Q2 −→ Q2 −→ K
√
u
u
=u−v d
ein Isomorphismus, σ(u + v d) = Φ A
=Φ
−v
v
√
√
√
und σ(σ(u + v d)) = σ(u − v d) = u + v d
√
√
σ(u) = σ(u + 0 d) = u − 0 · d = u für alle u ∈ K.
√
√
Falls r = u + v d ∈ R ist σ(r) = u + (−v) d ∈ R. Da σ ein Vektorraumisomorphismus ist, gilt σ(x + y) = σ(x) + σ(y)
√
√
√
σ((u + v d)(u0 + v 0 d)) = (uu0 + vv 0 d) − (av 0 + u0 v) d
√
√
√
(u − v d)(u0 − v 0 d) = (uu0 + vv 0 d) − (uv 0 + u0 v) d.
√
Induktiv ergibt sich daraus σ(xn ) = σ(x)n .
165
d) Der erste√Teil folgt durch√Induktion aus (∗). Wegen
σ((a + b d)n ) = σ(a + b d)n gilt auch der Rest.
√
Die Körper der Gestalt K = Q[ d], wobei d ∈ N kein Quadrat ist, nennt
man quadratische Zahlkörper. Halte d fest.
Definition. Die Funktion
N : K −→ Q, x 7−→ xσ(x)
heißt Norm von K.
√
Für x = a + b d ist also N (x) = a2 − b2 d(a, b ∈ Q).
In Analogie zu Anhang A.3.2 zeigt man leicht
1.3 Eigenschaften der Norm. Für x, y ∈ K gilt:
a) N (xy) = N (x)N (y); N (a) = a2 für a ∈ Q,
(insbesondere ist N (−1) = N (1) = 1).
b) Ist x 6= 0, so ist N (x) 6= 0 und x−1 =
σ(x)
.
N (x)
c) Ist x ∈ R, so ist N (x) ∈ Z.
d) Sei R× die Einheitengruppe von R und x ∈ R.
Genau dann ist x ∈ R× , wenn N (x) ∈ Z× = {±1}.
e) G = {x | N (x) = 1} ist eine Untergruppe von R× mit
(i) G = −G
(ii) r−1 = σ(r) für alle r ∈ G
Beweis.
a) ergibt sich sofort aus 1.2.
b) σ : K −→ K ist ein Isomorphismus von Q–Vektorräumen, also ist
σ(x) 6= 0 und somit N (x) = xσ(x) 6= 0 für x =
6 0.
σ(x)
σ(x)
Ferner ist N (x) ∈ K mit x N (x) = 1.
c) ist klar nach dem Beweis von 1.2.c).
166
d) Ist x ∈ R× , so ist x−1 ∈ R× mit x−1 x = 1 und 1 = N (x−1 )N (x), wobei
N (x), N (x−1 ) ∈ Z. Es folgt N (x) = ±1. Ist umgekehrt N (x) = ±1 und
x ∈ R, so ist Nσ(x)
= ±σ(x) ∈ R und x Nσ(x)
= 1, also x ∈ R× .
(x)
(x)
e) Nach a) ist G abgeschlossen unter der Multiplikation und Inversion und
1 ∈ G. Also ist G eine Untergruppe von R× .
N (−x) = N (−1)N (x) = N (x) nach a), also ist G = −G.
Ferner ist rσ(r) = N (r) = 1 für r ∈ G, d.h. r−1 = σ(r).
Im Fall d = 2 gilt noch
(1.4) Satz.
√
a) {ε ∈ R× | ε > 1} = {(1 + 2)n | n ∈ N>0 }
√
b) R× = {±(1 + 2)n | n ∈ Z}
√
c) G = {±(3 + 2 2)n | n ∈ Z}
Beweis.
a) Zeige zunächst, daß
√
1 + 2 = Min{ε | ε ∈ R× und ε > 1}
√
√
Beweis. N (1 + 2) = −1 =⇒ 1 + 2 ∈ R×
√
Sei ε ∈ R× mit ε > 1. Zeige, daß ε ≥ 1 + 2.
√
Schreibe ε = y + x 2 mit x, y ∈ Z : N (ε) = 2σ(ε) = ±1 nach 1.3.
(∗)
Aus |N (ε)| = 1 folgt ε · √
|σ(ε)| = 1 und |σ(ε)| = ε−1 < 1. Es folgt
2y = ε + σ(ε) >
√0 und 2x 2 = ε − σ(ε) > 0, also x > 0 und y > 0 aus
Z =⇒ ε ≥ 1 + 2.
Sei nun ε ∈ R× , ε > 1.
√
√
√
Wegen (1 + 2)n ≥ 1 + n 2 ist lim (1 + 2)n = ∞. Also gibt es genau
n→∞
√
√
ein n ∈ N>0 mit (1 + 2)n−1 < ε < (1 + 2)n . Es folgt η := (1+√ε2)n−1 ∈
√
R× , denn R×√ist eine Gruppe, die ε und 1 + 2 enthält. Außerdem gilt
1 < η ≤ 1 + 2 nach Wahl von n.
167
Nach (∗) gilt aber auch η ≥ 1 +
√
2 und daher ε = (1 +
√
2)n .
Umgekehrt ist für jedes n ≥ 1
√
√
(1 + 2)n > 1 und (1 + 2)n ∈ R× .
b) Offenbar gilt für ε ∈ R× \{±1}.
0 < ε < 1 ⇐⇒ ε−1 > 1
−1 < ε < 0 ⇐⇒ 0 < −ε < 1 ⇐⇒ −ε−1 > 1
ε < −1 ⇐⇒ −ε > 1
Ist also H := {ε ∈ R× | ε > 1}, so ist√R× = {±1} ∪ H ∪ H −1 ∪ (−H) ∪
(−H −1 ) und nach a) gilt H = {(1 + 2)n | n ∈ N>0 }. Es folgt
√
R× = {±(1 + 2)n | n ∈ Z}
√
2)n , n ∈ Z. Dann ist
√
N (ε) = N (1 + 2)n = (−1)n = 1 genau dann, wenn n = 2m, m ∈ N.
√
√
Ferner ist (1 + 2)2 = 3 + 2 2. Es folgt
√
G = {ε ∈ R× | N (ε) = 1} = {±(3 + 2 2)m | m ∈ Z}
c) Sei ε = ±(1 +
168
§2 Die Pell’sche Gleichung
Sei d ∈ N>0 kein Quadrat. Dann nennt man
Y 2 − dX 2 = 1
(∗)
die zu d gehörige Pell’sche Gleichung.
Die Lösungsmenge Hd := {(x, y) ∈ R2 | y 2 − dx2 = 1} der obigen Gleichung
stellt eine Hyperbel in der Ebene R2 dar.
Frage: Welche Gitterpunkte (x, y) ∈ Z2 liegen auf der Hyperbel Hd ?
2.1 Satz. Auf Hd liegen unendlich viele Gitterpunkte.
Genauer gilt: Ist (x0 , y0 ) ∈ N2>0 der Gitterpunkte auf Hd mit der kleinsten
positiven x–Koordinate, so ist
√
√
Hd ∩ Z2 = {(x, y) ∈ Z2 | Es gibt ein n ∈ Z mit y + x d = ±(y0 + x0 d)n }.
√
√
und
Bemerkung. Sei N die Norm von K = Q[ d], R = Z[ d] √
G = {r ∈ R | N (r) = 1} (vgl. §1). Dann ist wegen N (y + x d) = y 2 − dx2
√
(x, y) ∈ Hd ∩ Z2 genau dann, wenn y + x d ∈ G.
Es ist also zu zeigen, daß
(1)
√
G = {±(y0 + x0 d)n | n ∈ Z}
wenn (x0 , y0 ) ∈ Hd ∩ N2>0 mit kleinster x–Koordinate ist.
Im Fall d = 2 wurde dies im Satz 1.4 bereits bewiesen, wobei (x0 , y0 ) = (2, 3)
war.
Zum Beweis von (1) sind
Zunächst werden wir
√ einige Vorbereitungen nötig.
2
zeigen, daß es ein y + x d ∈ G gibt mit (x, y) ∈ N>0 .
2.2 Lemma. Die Ungleichung
(2)
√
1
|Y − X d| <
X
hat unendlich viele Lösungen (x, y) ∈ N2>0 .
169
Für eine reelle Zahl r ≥ 0 bezeichnen wir mit brc den ganzen Anteil von r.
Die ist die ganze Zahl, die bei der Dezimalbruchentwicklung von r vor dem
Komma steht.
√
Beweis von 2.2. Für x = 1 und y = b dc ist
√
√
√
√
1
|y − x d| = |b dc − d| < 1, d.h. |y − x d| < .
x
Damit hat (2) überhaupt eine Lösung in N2>0 .
Behauptung. Zu jeder Lösung (x, y) ∈ N2>0 von (2) gibt es eine weitere
Lösung (x0 , y 0 ) ∈ N2>0 von (2) mit
√
√
|y 0 − x0 d| < |y − x d|.
Damit erhält man wunschgemäß eine unendliche Folge von paarweise verschiedenen Lösungen von (2).
√
√
Beweis der Behauptung. Wegen d 6∈ Q ist |y − x d| 6= 0 für alle
(x, y) ∈ N2>0 . Also gibt es ein m ∈ N>0 mit
√
1
|y − x d| > .
m
(3)
Konstruiere nun ein (x0 , y 0 ) ∈ N2>0 mit
1
m
√
> |y 0 − x0 d|.
Betrachte dazu die Menge
√
√
√
M := {1, b dc, b2 d + 1c, . . . , bm dc + 1}
√
von m + 1 positiven ganzen Zahlen. Für jede u = bλ dc + 1 ∈ M gilt
√
0 < u − λ d ≤ 1.
Damit erhält man eine Abbildung
√
√
√
ϕ : M −→ (0, 1], bλ dc + 1 7−→ bλ dc + 1 − λ d
Zerlege (0, 1] in m halboffene Intervalle der Länge
(0, 1] = (0,
1
.
m
1 2
m−1
1
] ∪ ( , ] ∪ ... ∪ (
, 1]
m
m m
m
170
Weil M aus m + 1 Elementen besteht gibt es u1 < u2 in M , so daß ϕ(u1 )
und ϕ(u2 ) in das gleiche Intervall der Länge m1 fallen. Insbesondere ist dann
|ϕ(u2 ) − ϕ(u1 )| <
1
m
√
√
Es ist u1 = bλ1 dc + 1, u2 = bλ2 dc + 1 mit 0 ≤ λ1 < λ2 ≤ m.
Es folgt
√
√
√
1
|(u2 −u1 )−(λ2 −λ1 ) d| = |(u2 −λ2 d)−(u1 −λ1 d)| = |ϕ(u2 )−ϕ(u1 )| < .
m
Setzt man y 0 = u2 − u1 und x0 = λ2 − λ1 , so erhält man
√
1
|y 0 − x0 d| <
und 0 < x0 ≤ m
m
√
und (x0 , y 0 ) erfüllt die Bedingung |y 0 − x0 d| < m1 ≤ x10 .
Ferner gilt nach Wahl von m wegen (3)
√
√
1
|y 0 − x0 d| <
< |y − x d|, was zu beweisen war.
m
√
2.3 Lemma. Es gibt ein k ∈ Z, 0 <| k| < 1 + 2 d, so daß die Gleichung
(4)
Y 2 − dX 2 = k
in N2>0 unendlich viele Lösungen hat.
√
Beweis. Nach 2.2 gibt es ein (x, y) ∈ N2>0 mit |y − x d| < x1 .
Daraus ergibt sich die Beziehung
√
√
√
√
|y + x d| = |y − x d + 2x d| < x1 + 2x d, also
√
√
√
√
0 < |y 2 − dx2 | = |y − x d||y + x d| < x12 + 2 d ≤ 1 + 2 d.
Nach 2.2 gibt es dann sogar unendlich viele (x, y) ∈ N2>0 mit
√
0 < |y 2 − dx2 | < 1 + 2 d
Bei der Abbildung
ϕ : N2>0 −→ Z\{0}(x, y) 7−→ y 2 − dx2
171
√
√
werden also unendlich viele Punkte in das Intervall I = (−1 − 2 d, 1 + 2 d)
abgebildet, welches aber nur endlich viele ganze Zahlen enthält. Also wird
bei der Abbildung ϕ wenigstens ein Wert k ∈ I ∩ Z\{0} an unendlich vielen
Stellen (x, y) ∈ N2>0 angenommen.
Wir zeigen nun, daß die Gleichung Y 2 − dX 2 = 1 eine Lösung (x, y) ∈ N2>0
besitzt.
Nach 2.3 gibt es ein k ∈ Z\{0}, so daß die Gleichung (4) in N2>0 unendlich
viele Lösungen hat. Es gibt also insbesondere zwei Lösungen von (4) in N2>0 ,
so daß
(x1 , y1 ) 6= (x2 , y2 ),
x1 ≡ x2 mod k und y1 ≡ y2 mod k.
Wir setzen
y1 y2 − dx1 y2
y1 x2 − y2 x1
und x =
.
k
k
Dann gilt nach Wahl von (x1 , y1 ) und (x2 , y2 )
y=
y1 y2 − dx1 x2 ≡ y12 − dx21 ≡ k ≡ 0 mod k und
y1 x2 − y2 x1 ≡ y1 x1 − y1 x1 ≡ 0 mod k
und somit (x, y) ∈ Z2 .
Wir zeigen nun, daß x 6= 0, y 2 − dx2 = 1 und y 6= 0.
Angenommen x = 0. Es folgt y1 x2 = y2 x1 , also y2 =
Ferner ist x2 =
x2
x.
x1 1
Einsetzen in (4) ergibt
k=
y22
−
x2
y.
x1 1
dx22
=
x2
x1
2 2
x2
2
2
y1 − dx1 =
k, d.h. x2 = x1
x1
und damit auch y2 = y1 , Widerspruch.
Aus k 2 (y 2 − dx2 ) = (y1 y2 − dx1 x2 )2 − d(y1 x2 − y2 x1 )2 =
= (y12 − dx21 )(y22 − dx22 ) = k 2 folgt y 2 − dx2 = 1.
Ferner ist y 2 = 1 + dx2 ≥ 1 und daher y 6= 0.
Damit ist (|x|, |y|) ∈ N2>0 eine Lösung der Pell’schen Gleichung.
Sei nun (x0 , y0 ) ∈ N2>0 die Lösungt der Pell’schen Gleichung mit kleinster
x–Koordinate. Wie schon ausgeführt wurde, ist zu zeigen, daß
√
G = Γ := {±(y0 + x0 d)n | n ∈ Z}
172
√
Wegen N (y0 + x0 d) = y02 − x20 d = 1 ist Γ ⊆ G ⊆ R× .
Offenbar ist Γ wie G eine Untergruppe von R× , also Γ = Γ−1 , und es gilt
Γ = −Γ und G = −G.
Zeige zunächst, daß
(5)
G+ := {r ∈ G | r > 1} ⊆ Γ
−1
Dann ist auch G = {1} ∪ G + ∪ − G+√
∪ G−1
+ ∪ −G+ ⊆ Γ. Zum Beweis von
(5) zeigen wir zuerst, daß ε := y0 + x0 d = Min G+ .
√
Sei dazu r = y + x d ∈ G+ , r 6= ε. Angenommen r < ε.
Aus rσ(r) = 1 folgt 1 > σ(r) > 0, d.h.
√
√
1 > y − x d > 0 und 2y = r + σ(r) > 1, 2x d = r − σ(r) > 0.
Es folgt y > 0 und x > 0. Nach Wahl von x0 ist daher x ≥ x0 .
Aus r < ε und x ≥ x0 folgt y < y0 und daher
p
p
y2 − 1
y02 − 1
√
√
x=
<
= x0 , Widerspruch.
d
d
√
√
√
Sei nun y + x d ∈ G+ , y + x d 6= ε. Wie gesehen y + x d > ε. Wegen ε > 1
ist lim εn = ∞. Es gibt also genau ein n ∈ N>0 mit
n→∞
√
εn−1 < y + d ≤ εn .
√
Da G eine Gruppe ist und {ε, y + x d} ⊆ G}, ist auch η =
(6)
√
y+x d
n−1
ε
∈ G.
Aus (6) folgt 1 < η ≤ ε und η ∈ G+ .
Daher ist auch η ≥ ε = Min G+ , d.h. η = ε und
√
y + x d = εn .
Wir wollen noch sehen, wie man rekursiv von (x0 , y0 ) ausgehend alle Gitterpunkte auf Hd berechnet. Dabei kann man sich aus Symmetriegründen auf
den ersten Quadranten beschränken.
Wähle yn , xn ∈ N>0 so, daß
√
√
yn + xn d = (y0 + x0 d)n+1
173
(n ∈ N).
Nach 2.1 ist dann
Hd ∩ N2>0 = {(xn , yn ) | n ∈ N}.
√
√
Wegen yn − xn d = (y0 − x0 d)n+1 ergibt sich
xn =
yn =
√
√
(y0 +x0 d)n+1 −(y0 −x0 d)n+1
√
2 d
√
√
(y0 +x0 d)n+1 +(y0 −x0 d)n+1
2
und
für alle n ∈ N
2.4 Korollar. Die positiven Gitterpunkte (xn , yn ), n ∈ N auf Hd berechen
sich rekursiv aus (x0 , y0 ) mit den Formeln
xn = yn−1 x0 + xn−1 y0
yn = yn−1 y0 + dxn−1 x0
für alle n ≥ 1
Beweis. yn−1 x0 + xn−1 y0 =
√
√
√
√
√
√
(y0 + x0 d)n x0 d + (y0 − x0 d)n x0 d + (y0 + x0 d)n y0 − (y0 − x0 d)n y0
√
=
2 d
√
√
√
√
(y0 + x0 d)n (y0 + x0 d) − (y0 − x0 d)n (y0 − x0 d)
√
=
= xn
2 d
Der Beweis der zweiten Formel verläuft analog.
Zum Schluß wollen wir noch zeigen, wie man im Fall einer Primzahl der
Form d = m2 + 1, m ∈ N die kleinste positive ganze Lösung der Pell’schen
Gleichung berechnet.
2.5 Satz. Ist d = m2 + 1 eine Primzahl, m ∈ N, so ist (2m, 2m2 + 1) die
kleinste positve Lösung der Pell’schen Gleichung Y 2 − dX 2 = 1.
Beweis. (2m, 2m2 + 1) ist offenbar eine Lösung.
Sei (x, y) ∈ N2>0 eine beliebige Lösung. Dann ist
x2 (m2 + 1) = y 2 − 1 = (y − 1)(y + 1).
Da nun m2 + 1 eine Primzahl ist, folgt
m2 + 1 | y + 1 oder m2 + 1 | y − 1.
1. Fall. Ist m2 + 1 = y + 1 oder m2 + 1 = y − 1, so ist
174
x2 = m2 − 1 oder x2 = m2 + 3. Es folgt
(m − x)(m + x) = 1 oder (x − m)(x + m) = 3.
Dies ist nur möglich, wenn m = 1, x = 2 und y = 3 gilt.
2. Fall. m2 + 1 k y − 1 oder m2 + 1 k y + 1. Es folgt
y + 1 ≥ 2(m2 + 1) und daher x2 (m2 + 1) = (y + 1)(y − 1) ≥
≥ 2(m2 + 1)(2(m2 + 1) − 2) = 4(m2 + 1)m2 .
Kürzen ergibt
x2 ≥ 4m2 und x ≥ 2m.
175
§3 Anwendungen der Pell’schen Gleichung
A. Quadrat- und Dreieckszahlen.
Quadratzahlen:
•
1
••
••
4
•••
•••
•••
9
• • ••
• • ••
• • ••
• • ••
16
•••••
•••••
•••••
•••••
•••••
25
•
••
•••
• • ••
10
•
••
•••
• • ••
•••••
15
• • • • ••
• • • • ••
• • • • ••
• • • • ••
• • • • ••
• • • • ••
36
Dreieckszahlen:
•
1
•
••
3
•
••
•••
• • ••
•••••
• • • • ••
21
•
••
•••
6
•
••
•••
• • ••
•••••
• • • • ••
•••••••
28
•
••
•••
• • ••
•••••
• • • • ••
•••••••
• • • • • • ••
36
Dabei sind 1 und 36 sowohl Quadrat- als auch Dreieckszahlen.
Frage: Wie kann man alle natürlichen Zahlen bestimmen, die sowohl Quadratals auch Dreieckszahlen sind? Gibt es davon unendlich viele?
176
Ist w ∈ N>0 eine Quadrat- und Dreieckszahl, so gibt es natürliche Zahlen m
und n mit
w = m2 =
(1)
n(n + 1)
(= 1 + 2 + . . . + n)
2
Setze x := 2m und y = 2n + 1. Aus x und y berechnen sich n und m als
n = y−1
und m = x2 .
2
Aus (1) folgt
x2
4
=
y−1 y+1 1
· 2 · 2,
2
d.h. x und y genügen der Pell’schen Gleichung.
Y 2 − 2x2 = 1
(2)
Ist also w = m2 n(n+1)
mit m, n ∈ N>0 , so ist (x, y) = (2m, 2n + 1) ein
2
Gitterpunkt auf der Hyperbel H2 : Y 2 − 2X 2 = 1 im positiven Quadranten.
Sei umgekehrt (x, y) ∈ H2 ∩ N2>0 . Dann gilt 2x2 = y 2 − 1 = (y + 1)(y − 1).
Insbesondere ist y ungerade und x gerade und y > 1. Setze
m :=
x
2
und
n :=
y−1
.
2
Dann sind m, n ∈ N>0 und
m2 =
x2
y2 − 1
y+1y−1 1
1
=
=
· = (n + 1)n · .
4
8
2
2
2
2
2
Damit ist x4 eine Quadrat- und Dreieckszahl, wobei x2 die Kantenlänge des
zugehörigen Quadrats und y−1
die Kantenlänge des zugehörigen Dreiecks ist.
2
Wir fassen das bisher gesehene zusammen.
3.1 Satz. Die Quadrat- und Dreieckszahlen aus N>0 entsprechen eineindeutig
den positiven Gitterpunkten auf der Hyperbel H2 : Y 2 − 2X 2 = 1. Genauer
gilt:
eine Quadrat- und Dreieckszahl, so ist
a) Ist w = m2 = n(n+1)
2
(2m, 2n + 1) ∈ H2 .
b) Ist (x, y) ∈ H2 ∩ N2>0 , so sind die Zahlen m = x2 und n = y−1
ganz und
2
n(n+1)
2
w = m = 2 ist die zugehörige Quadrat- und Dreieckszahl.
177
Wir wenden Korollar 2.4 im Fall d = 2 an und erhalten
3.2 Korollar. Die Gitterpunkte H2 ∩ N2>0 ergeben sich rekursiv aus
x0 = 2, y0 = 3 mit den Formeln
xn = 2yn−1 + 3xn−1 ; yn = 3yn−1 + 4xn−1 für alle n ≥ 1.
Aus 3.1 und 3.2 ergeben sich schließlich Rekursionsformeln für die Quadratund Dreieckszahlen.
3.3 Korollar. Ordnet man die Quadrat- und Dreieckszahlen
zn = qn2 = dn (d2n+1 ) nach ihrer Größe, so gilt für die Kantenlängen qn bzw. dn
der zugehörigen Quadrate bzw. Dreiecke
q0 = d0 = z0 = 1
qn = 2dn−1 + 3qn−1 + 1 ; dn = 3dn−1 + 4qn−1 + 1 für alle n ≥ 1
Man erhält also
q1 = 2 + 3 + 1 = 6
; d1 = 3 + 4 + 1 = 8,
z1 = 36
q2 = 16 + 18 + 1 = 35
; d2 = 24 + 24 + 1 = 49,
z2 = 1225
q3 = 98 + 105 + 1 = 204 ; d3 = 147 + 140 + 1 = 288, z3 = 41616
usw.
Beweis. Nach 3.1 sind die Zahlen zn =
Dreieckszahlen und es gilt
qn =
dn =
xn
2
yn −1
2
=
=
x2n
,n
4
= 0, 1, 2, . . . die Quadrat- und
Kantenlänge des Quadrats und
Kantenlänge des Dreiecks.
Setzt man die Formeln aus 3.2 ein, so ergibt sich
x2
z0 = 40 = 1, q0 = x20 = 1, d0 = y02−1 = 1 und
qn = x2n = yn−1 + 32 xn−1 = 2dn−1 + 1 + 3qn−1 ,
dn = yn2−1 = 23 yn−1 + 2xn−1 − 12 = 32 (2dn−1 + 1) + 4qn−1 −
= 3dn−1 + 4qn−1 + 1 für n ≥ 1
1
2
=
B. Ein kombinatorisches Problem. Eine Urne enhalte q ≥ 2 Kugeln;
davon seien r Kugeln rot und die übrigen schwarz.
Frage: Wie müssen die Zahlen q und r gewählt sein, damit gilt:
178
(1)
Die Wahrscheinlichkeit dafür, daß zwei Kugeln, die ohne Zurücklegen
gezogen werden, beide rot sind, ist 12 .
3.4 Satz. Aussage (1) gilt genau dann, wenn gilt:
(2)
q(q − 1) = 2r(r − 1)
q
Beweis. Es gibt
verschiedene Stichproben der Ordnung 2. Davon beste2
r
hen
aus zwei roten Kugeln. Also gilt (1) genau dann, wenn
2
q
r
=2
, d.h. wenn q(q − 1) = 2r(r − 1).
2
2
Zusammenhang mit der Pell’schen Gleichung.
Wir betrachten anstelle der gewöhnlichen Pell’schen Gleichung
Y 2 − 2X 2 = +1 die Gleichung
Y 2 − 2X 2 = −1 ; H̃2 : Y 2 − 2X 2 = −1
(3)
3.5 Satz. Seien q und r positive ganze Zahlen. Genau dann erfüllen q und r
die Bedingung (2), wenn (x, y) = (2q − 1, 2r − 1) die Gleichung (3) löst.
Beweis. Seien x, y ∈ N>0 mit y 2 − 2x2 = −1. Wegen y 2 = 2x2 − 1 ist y
ungerade. Es folgt
y 2 ≡ 1 mod 4 und daher 2x2 = y 2 + 1 ≡ 2 mod 4.
Also ist auch x ungerade. Schreibe daher x und y in der Form
y = 2q − 1 und x = 2r − 1 mit q, r ∈ N>0 .
Aus y 2 = 2x2 − 1 folgt (2q − 1)2 = 2(2r − 1)2 − 1, d.h.
q(q − 1) = 2r(r − 1).
Seien umgekehrt q, r ∈ N>0 mit q(q − 1) = 2r(r − 1).
Setze y = 2q − 1 und x = 2r − 1. Dann gilt
y 2 − 2x2 = 4q 2 − 4q + 1 − 8r2 + 8r − 2 = −1.
Damit ist gezeigt:
179
3.6 Korollar. Genau dann erfüllt das Zahlenpaar (q, r) die Bedingung (1),
wenn (x, y) = (2q − 1, 2r − 1) die Gleichung Y 2 − 2X 2 = −1 löst.
√
√
Sei nun wieder
K
=
Q[
2],
R
=
Z[
2] und N die Norm von K, d.h.
√
N (y + x d) = y 2 − 2x2 , wenn (x, y) ∈ Q2 .
Es gilt also:
√
(x, y) ∈ N2>0 löst (3) genau dann, wenn N (y + x d) = −1.
√
Insbesondere ist dann y + x d ∈ R× .
√
Nach 1.4 gilt R× = {±(1 + 2)n | n ∈ Z} und
√
N (1 + 2) = −1. Es folgt
√
√
√
(4) {y + x d | (x, y) ∈ N2>0 , N (y + x d) = −1} = {(1 + 2)2n+1 | n ∈ N}
3.7 Korollar. Die Menge aller (q, r) ∈ N2>0 mit q ≥ 2 und q(q−1) = 2r(r−1)
erhält man mit der Formel
m 1/2
1/2
3 4
q
, m ∈ N>0 .
+
=
1/2
1/2
2 3
r
Beweis. Nach 3.5 gilt
{(q, r) ∈ N2>0 | q(q − 1) = 2r(r − 1)} = {(
x+1 y−1
,
) | x, y ∈ H̃2 ∩ N2>0 }
2
2
√
Nach (4) ist H̃2 ∩ N2>0 = {(1 + 2)2m+1 | m ∈ N}.
√
√
Nun ist (1 + 2)2 = 3 + 2 2 und daher
√
√
√
(1 + 2)2m+1 = (3 + 2 2)m (1 + 2)
3.8 Lemma. Für u, v, x, y ∈ R gilt:
√
√
√
y
3 4
v
=
genau dann wenn (3 + 2 2)(v + u 2) = y + x 2
x
2 3
u
Beweis. Es ist
3 4
2 3
v
3v + 4u
=
und
u
2v + 3u
180
√
√
√
(3 + 2 2)(v + u √2) = (3v + 4u) + (2v + 3u) 2. Aus der linearen Unabhängigkeit von 1 und 2 über Q folgt die Behauptung.
Aus 3.8 ergibt sich rekursiv
√
y + x 2 = (1 +
√
2)
2m+1
m y
3 4
1
⇐⇒
=
x
2 3
1
Zurück zum Beweis von 3.7.
Wie gesehen durchläuft (q, r) die Paare
√
√
x+1 y−1
x+1
mit y + x 2 = (1 + 2)2m+1 , m ∈ N≥0 , q =
,
≥2
2
2
2
q
1
m=0:
=
was nicht zugelassen ist.
r
1
m ≥ 1 : Wie gesehen ist
m q
y
1/2
3 4
1
1/2
1
1
= 2
+
=2
+
=
r
x
1/2
2
3
1
1/2
m 1/2
1/2
3 4
+
; offenbar ist dann q ≥ 2
=
1/2
1/2
2 3
Für kleine m ergibt sich:
4
1/2
1/2
3 4
also q = 4, r = 3
=
+
3
1/2
1/2
2 3
3
4
= 6,
=3
2
2
7/2
1/2
41/2
1/2
21
4
+
=
+
=
; q = 21, r = 15
3
5/2
1/2
29/2
1/2
15
21
15
= 210,
= 105
2
2
3 4
41/2
1/2
120
+
=
; q = 120, r = 85
2 3
29/2
1/2
85
120
85
= 7140;
= 3570
2
2
Probe:
3
2
Probe:
Probe:
181
§4 Kettenbrüche
In Kapitel III, 6.6 haben wir gesehen, daß sich jede reelle Zahl x als Grenzwert
einer Folge (rn ) rationaler Zahlen schreiben läßt. Für Zahlen x > 1 gibt es eine
kanonische Wahl für diese Folge, die sogenannte Kettenbruchentwicklung
von x.
An ihr läßt sich ablesen, ob es sich bei x um eine quadratische Irrationalzahl
handelt oder nicht.
Definition. Seien x1 , . . . , xn positive reelle Zahlen, n ≥ 1.
Der zu (x1 , . . . , xn ) gehörige Kettenbruch < x1 , . . . , xn > ist rekursiv erklärt als
< x1 >:= x1
< x1 , x2 >:= x1 + x12
< x1 , x2 , x2 >:= x1 +
..
.
1
<x2 ,x3 >
< x1 , . . . , xn >:= x1 +
=< x1 , < x2 , x3 >>
1
<x2 ,...,xn >
=< x1 , < x2 , . . . , , xn >> (n ≥ 3)
Beispiel. < 1, 2, 3, 4 >=< 1, < 2, 3, 4 >>= 1 +
=1+
(n = 1)
(n = 2)
(n = 3)
1
<2,3,4>
=
1
1
1
43
1
=1+
=1+
=1+
1
1
4 =
< 2, < 3, 4 >>
30
2 + <3,4>
2 + 3+ 1
2 + 13
4
4.1 Bemerkung. Für n ≥ 3 gilt < x1 , . . . , xn >=< x1 , . . . , xn−2 , < xn−1 , xn >>.
Beweis. n = 3 :< x1 , x2 , x3 >=< x1 , < x2 , x3 >> gilt definitionsgemäß .
Schluß von n − 1 auf n. Sei n ≥ 4 und 4.1 bewiesen für n − 1:
< x 1 , . . . , xn >
Def.
< x1 , < x2 , . . . , xn >> = < x1 , < x2 , . . . , xn−2 , < xn−1 , xn >>>=
Def.
< x1 , x2 , . . . , xn−2 , < xn−1 , xn > .
=
=
I.V.
Offenbar gilt für a1 , . . . , an ∈ N>0 :< a1 >= a1 und für n ≥ 2 ist
1
∈ Q (Induktion nach n) und
< a1 , . . . , an >= a1 + <a2 ,...,a
n>
< a1 , . . . , an >> 1.
Es gilt auch umgekehrt
4.2 Satz. Jedes x ∈ Q, x ≥ 1 besitzt eine endliche Kettenbruchentwicklung
x =< a1 , . . . , an > mit ai ∈ N>0 . Eine reelle Zahl x ≥ 1 hat somit genau dann
182
eine endliche Kettenbruchentwicklung x =< a1 , . . . , an > mit a1 , . . . , an ∈
N>0 , wenn x rational ist.
Beweis. 1 =< 1 >, wir können also x > 1 annehmen.
Schreibe x in gekürzter Darstellung
x=
y0
mit ggT (y0 , y1 ) = 1 und y0 , y1 ∈ N>0 .
y1
Schließe induktiv nach y1 . Für y1 = 1 ist x ∈ N und x =< x >.
Sei y1 > 1 und 4.2 bewiesen für alle reellen Zahlen > 1 mit einem Nenner
< y1 . Dividiere y0 durch y1 mit Rest und erhalte
y0 = a1 y1 + y2 mit a1 > 0, 0 < y2 < y1 , also
y0
y2
y1
x =
= a1 +
und
>1
y1
y1
y2
Nach Induktionsvoraussetzung besitzt x2 = yy21 eine endliche Kettenbruchentwicklung x2 =< a2 , . . . , an >, i ∈ N>0 . Es folgt
x = a1 +
y2
1
= a1 +
=< a1 , x2 >=< a1 , < a2 , . . . , an >>=< a1 , . . . , an > .
y1
x2
Anmerkung zu 4.2. Der euklidische Algorithmus für das Paar (y0 , y1 ) liefert
die Zahlen a1 , . . . , an mit x =< a1 , . . . , an >:
y0 = a1 y1 + y2
y1 = a2 y2 + y3
..
.
, 0 < y 2 < y1
, 0 < y 3 < y2
yn−2 = an−1 yn−1 + yn , 0 < yn < yn−1
yn−1 = an yn
wobei ai ∈ N>0 , i = 1, . . . , n. Man schließt induktiv:
y0
=< a1 , . . . , an >
y1
Konstruktion der Kettenbruchentwicklung einer irrationalen Zahl.
Für x ∈ R, x ≥ 0 bezeichne bxc den ganzen Anteil von x.
Sei nun x ∈ R\Q, x > 1.
183
1. Schritt. Setze a1 := bxc und x1 := x.
Nach Voraussetzung ist 0 < x − a1 < 1, d.h.
x2 :=
1
x−a1
> 1 und x2 6∈ Q, und daher
x = a1 +
1
=< a1 , x2 > .
x2
Man kann wegen x2 > 1, x2 6∈ Q, das Verfahren mit x2 fortsetzen.
2. Schritt. Setze a2 := bx2 c und x3 :=
x2 = a2 +
1
.
x2 −a2
Dann ist
1
=< a2 , x3 > und
x3
x =< a1 , x2 >=< a1 , < a2 , x3 >>=< a1 , a2 , x3 >, x3 > 1, x3 6∈ Q.
Fahre so fort: Seien a1 , . . . , an−1 ∈ Q>0 und x1 , . . . , xn ∈ N\Q, xj > 1 schon
konstruiert.
Schritt von n − 1 auf n. Setze an := bxn c und xn+1 :=
1
.
xn −an
Dann ist xn+1 > 1, xn+1 6∈ Q und
(1)
xn = a n +
1
xn+1
=< an , xn+1 > .
Damit sind ai und xi für alle i ∈ N>0 rekursiv definiert und
xi > 1, x1 ∈ R\Q, ai ∈ N>0 . Ferner gilt
(2)
x =< a1 , . . . , an , xn+1 > für alle n ∈ N>0 .
Beweis. (2) gilt nach Schritt 1 für n = 1.
4.2
Schluß von n auf n + 1 :< a1 , . . . , an , an+1 , xn+2 > =
I.V.
< a1 , . . . , an , < an+1 , xn+2 >>< a1 , . . . an , xn+1 > = x
Definition. Die rationale Zahl rn :=< a1 , . . . , an >, n ≥ 1 heißt der n–te
Näherungsbruch und xn die n–te Restzahl von x.
Offenbar gilt: < ak , . . . , ak+n−1 > ist der n–te Näherungsbruch von xk und
xk+n−1 ist die n–te Restzahl von xk .
4.3 Satz. Die Folge (rn ) konvergiert gegen x.
184
4.4 Lemma. Sei (a1 , . . . , an ) ∈ Qn>0 beliebig vorgegeben.
Setze rn :=< a1 , . . . , an > und definiere rekursiv:
p0 = 1, p1 = a1 , pi = ai pi−1 + pi−2
für 2 ≤ i ≤ n. Dann gilt
q0 = 0, q1 = 1, qi = ai qi−1 + qi−2
(3)
rn =
pn
.
qn
Beweis von 4.4. (Induktion nach n.) n ≤ 2:
r1 =< a1 >=
a2 p1 + 1
1
p1 p2
;
=
=< a1 , a2 >= r2
= a1 +
q1 q2
a2 · 1 + 0
a2
Sei n ≥ 3 und 4.4 bewiesen für n − 1. Insbesondere gilt 4.4 für das (n − 1)–
tupel (a1 , . . . , an−2 , < an−1 , an >>. Es folgt
< an−1 , an > pn−2 + pn−3
=
< an−1 , an > qn−2 + qn−3
an−1 pn−2 + pn−3 + a1n pn−2
=
=
an−1 qn−2 + qn−3 + a1n qn−2
< a1 , . . . , an−2 , < an−1 , an >> =
(an−1 +
(an−1 +
1
)pn−2
an
1
)q
an n−2
+ pn−3
pn−1 +
1
p
an n−2
1
q
an n−2
qn−1 +
+ qn−3
an pn−1 + pn−2
pn
=
an qn−1 + qn−2
qn
=
Ferner gilt nach 4.1 < a1 , . . . , an >=< a1 , . . . , an−2 , < an−1 , an >>.
Beweis von 4.3. Seien pn und qn wie in 4.4.
Behauptung. Für alle n ∈ N>0 ist
(4)
x=
pn xn+1 + pn−1
qn xn+1 + qn−1
Beweis. (Induktion) n = 1:
a 1 x2 + 1
1
p1 x2 + p0
(2)
=
= a1 +
=< a1 , x2 > = x
q1 x2 + q0
x2
x2
Schluß von n − 1 auf n, n ≥ 2 : xn+1 =
1
xn −an
, also
an pn−1 + pn−2 + xn pn−1 − an pn−1
pn + (xn − an )pn−1
=
=
xn − an
xn − a n
pn−1 xn + pn−2
=
xn − a n
pn xn+1 + pn−1 =
185
Analog zeigt man: qn xn+1 + qn−1 =
qn−1 xn +qn−2
.
xn −an
Es folgt
pn xn+1 + pn−1
pn−1 xn + pn−2 I.V.
=
= x
qn xn+1 + qn−1
qn−1 xn + qn−2
Behauptung. Für alle n ≥ 1 ist
pn−1 qn − qn−1 pn = (−1)n−1
(5)
Beweis. (Induktion) n = 1 : p0 q1 − q0 p1 = 1 · 1 − 0 · a1 = 1 = (−1)0 .
Schluß von n − 1 auf n, n ≥ 2:
pn−1 qn − qn−1 pn = pn−1 (an qn−1 + qn−2 ) − qn−1 (an pn−1 + pn−2 ) =
I.V.
= pn−1 qn−2 − qn−1 pn−2 = (−1)(pn−2 qn−1 − qn−2 pn−1 ) = (−1)n−1
Daraus folgt schließlich
(4) (pn xn+1 + pn−1 )qn − (qn xn+1 + qn−1 )pn p
n
x − = =
qn qn (qn xn+1 + qn−1 )
pn−1 qn − qn−1 pn (5)
1
1
qn (qn xn+1 + qn−1 ) = (qn xn+1 + qn−1 )qn < q 2
n
Wegen qn −→ ∞ für n −→ ∞ ist
lim rn = lim pqnn = x.
n−→∞
1
2
qn
eine Nullfolge und
n→∞
Schreibe für diese Tatsache auch
x = lim < a1 , . . . , an > oder x =< a1 , a2 , a3 , . . . > .
n−→∞
186
§5 Periodische Kettenbrüche und
quadratische Irrationalzahlen
Wir werden sehen, daß die Kettenbruchentwicklung einer irrationalen Zahl
x > 1 genau dann periodisch ist, wenn x eine quadratische Irrationalzahl ist.
Sei also x ∈ R\Q, x > 1 und x = lim < a1 , . . . , an > die Kettenbruchentx→∞
wicklung von x.
Definition. x =< a1 , a2 , a3 , . . . > heißt periodischer Kettenbruch, wenn
es ein n0 ∈ N>0 und ein k ∈ N>0 , gibt, so daß
an = an+k für alle n ≥ n0 .
k heißt Periode von x. Schreibe in diesem Fall
x =< a1 , . . . , an0 −1 , an0 , . . . , an0 +k−1 >
Spezialfall. Sei x periodisch mit n0 = 1, also x =< a1 , . . . , ak > .
Dann nennt man x auch rein periodisch.
Nach Definition der Restzahlen gilt für alle l ∈ N>0 .
xl =< al , al+1 , . . . >, insbesondere für l = k + 1
xk+1 =< ak+1 , ak+2 , . . . >=< a1 , a2 , . . . >= x
Aus §4, (4) ergibt sich also im rein periodischen Fall
x=
pk x + pk−1
, d.h.
qk x + qk−1
qk x2 + (qk−1 − pk )x − pk−1 = 0
Fazit. Hat x eine rein periodische Kettenbruchentwicklung, so ist x eine
quadratische Irrationalzahl.
Sei
(6)
aX 2 − bX − c = 0
die normierte Gleichung von x, d.h.
(7)
{a, b, c} ⊆ Z, a > 0 und ggT (a, b, c) = 1
187
Definitionsgemäß ist D := b2 + 4ac die Diskriminante von x.
√
b
Die Lösungen von (6) sind dann 2a
± aD und D > 0.
√
Es ist also√x = u + v D mit u, v ∈ Q, und die zweite Lösung von (6) ist
x0 = u − v D, die zu x konjugierte quadratische Irrationalzahl.
Allgemeiner Fall. Sei nun x > 1 periodisch, aber nicht rein periodisch:
x =< a1 , . . . , an0 −1 , an0 , . . . , an0 +k−1 >, n0 ≥ 2
Dann ist xn0 =< an0 , . . . , an0 +k−1 > rein periodisch, also nach dem Spezialfall
eine quadratische Irrationalzahl.
Es gilt:
(∗)
x = x1 , x1 = a1 +
1
1
, . . . , xi−1 = ai−1 + , . . .
x2
xi
Behauptung. Die Zahl x ist ebenfalls eine quadratische Irrationalzahl und
sie hat die gleiche Diskriminante wie xn0 .
Wegen (∗) folgt dies sofort aus
5.1 Lemma. Sei y = α +
1
x
mit α ∈ Z und x ∈ R\{0}.
a) Genau dann ist x eine quadratische Irrationalzahl, wenn dies für y
zutrifft.
b) Ist x eine quadratische Irrationalzahl und x0 konjugiert zu x, so ist auch
α + x10 konjugiert zu α + x1 .
c) Ist x eine quadratische Irrationalzahl, so haben x und y die gleiche
Diskriminante.
Beweis. Sei (a, b, c) wie in (7). Es gilt
1
1
az 2 − bz − c = 0 ⇐⇒ c( )2 + b( ) − a = 0
z
z
Damit ist a) und b) im Fall α = 0 gezeigt. Ferner ist
1
D( ) = (−b)2 + 4ca = b2 + 4ca = b2 + 4ac = D(x).
x
Es ist also noch zu zeigen: Ist x eine quadratische Irrationalzahl, so gilt:
188
i) x + α ist eine quadratische Irrationalzahl.
ii) Ist x0 konjugiert zu x, so ist x0 + α konjugiert zu x + α.
iii) D(x) = D(x + α)
i) und ii) wurden bereits im §1 gezeigt.
Sei (6) die normierte Gleichung von x.
Dann ist x + α eine Nullstelle des Polynoms.
a(X − α)2 − b(X − α) − c = aX 2 − (b + 2aα)X − (c − bα − aα2 )
Es ist ggT (a, b + 2aα, c − bα − aα2 ) = ggT (a, b, c) = 1, also
D(x + α) = (b + 2aα)2 + 4a(c − bα − aα2 ) = b2 + 4ac = D(x).
Damit gilt auch iii).
Fazit. Hat x ∈ R, x > 1 eine periodische Kettenbruchentwicklung, so ist x
eine quadratische Irrationalzahl.
Davon gilt auch die Umkehrung, man hat also
5.2 Satz. (Euler, Lagrange) Sei x ∈ R\Q, x > 1. Genau dann hat x eine
periodische Kettenbruchentwicklung, wenn x eine quadratische Irrationalzahl
ist.
Beweis. Sei x > 1 eine quadratische Irrationalzahl. Wir werden zeigen, daß
x ein periodischer Kettenbruch ist. Dazu sind einige Vorbereitungen nötig.
Definition. x heißt reduziert, falls für die zu x konjugierte Zahl x0 gilt:
0 > x0 > −1.
5.3 Lemma. Sei D > 0 eine ganze Zahl, die kein Quadrat ist. Dann gibt es
nur endlich viele reduzierte quadratische Irrationalzahlen mit Diskriminante
D.
Beweis. Sei x > 1 eine quadratische Irrationalzahl mit Diskriminante D und
√
b
D
0
normierter Gleichung
(6).
Da
x
>
1
ist
und
0
>
x
>
−1,
muß
x
=
+
2a
2a
√
b
und x0 = 2a
− 2aD sein.
Es folgt 0 < −x0 =
√
−b+ D
2a
< 1, also
√
b
= x + x0 > 1 − 1 = 0, also b > 0 und b < D wegen x0 < 0.
a
189
Es folgt 0 < b <
√
D.
Damit kann b bei vorgegebenem D nur endlich viele verschiedene ganzzahlige
Werte annehmen. Wegen −x0 < 1 < x gilt
√
√
−b + D
b+ D
<a<
2
2
Also gibt es bei vorgegebenen b und D auch für a nur endlich viele Möglichkeiten.
2
Schließlich ist c = D−b
durch die Vorgabe von a, b und D schon festgelegt.
4a
Insgesamt haben wir gesehen:
Ist D vorgegeben, so gibt es für a, b und c in einer normierten Gleichung (6)
nur endlich viele Möglichkeiten, wenn ihre Lösung x > 1 eine reduzierte
quadratische Irrationalzahl werden soll.
Eigentlicher Beweis von 5.2. Sei x > 1 eine quadratische Irrationalzahl
und x =< a1 , a2 , . . . > ihre Kettenbruchentwicklung.
1
. Nach 3.5 ist mit x auch
Für ihre Restzahlen gilt: x = x1 und xn = an + xn+1
xn eine quadratische Irrationalzahl, und zwar mit der gleichen Diskriminante
wie x. Ferner besteht zwischen den Konjugierten x0n und x0n+1 ebenfalls die
Beziehung
x0n = an +
x0n+1 =
x0n
1
x0n+1
und damit auch
1
für alle n ∈ N>0
− an
Behauptung. Für alle n ≥ 2 gilt
−x0n =
(8)
qn−2 x0 − pn−2
qn−1 x0 − pn−1
Beweis. (Induktion.) x = x1 =⇒ x0 = x01 =⇒ x02 =
1
x0 −a1
=
q0 x0 −p0
p1 −q1 x0
Schluß von n auf n + 1, n ≥ 2 :
x0n+1 =
1
I.V.
=
x0n − an
1
qn−2 x0 −pn−2 −an pn−1 +an qn−1 x0
−qn−1 x0 +pn−1
190
=
=
−qn−1 x0 + pn−1
qn−1 x0 − pn−1
=
−
q n x0 − p n
qn x0 − pn
Aus (8) ergibt sich nun
−
1
x0n
=
(5)
=
qn−1 x0 − pn−1
qn−1 qn−2 x0 − pn−1 qn−2
1
=
·
=
0
0
qn−2 x − pn−2
qn−2 x − pn−2
qn−2
(qn−1 qn−2 x0 − qn−1 pn−2 ) − (−1)n−1
1
1
·
=
0
qn−2 x − pn−2
qn−2
qn−2
pn−2
n→∞ qn−2
Nach 4.3 ist lim
(−1)n−1
qn−1 −
n−2
qn−2 (x0 − pqn−2
)
= x und es ist x 6= x0 wegen D 6= 0.
Ferner ist qn−1 − qn−2 ≥ qn−3 ≥ 1 für n ≥ 4.
Für große n gilt also
1
1
− 0 −1=
xn
qn−2
(−1)n−2
qn−1 − qn−2 − 0 pn−2
(x − qn−2 )qn−2
denn dann ist qn−2 > 0, qn−1 − qn−2 ≥ 1 und
(−1)n−1
p
(x0 − qn−2 )qn−2
n−2
!
>0
ist eine Nullfolge.
Somit ist − x10 > 1 für große n, d.h. −1 < x0n < 0.
n
M. a. W.: Für große n ist xn eine reduzierte quadratische Irrationalzahl mit
Diskriminante D.
Da es davon nach 5.3 aber nur endlich viele gibt, gibt es positive ganze Zahlen
i0 und l mit xi0 = xi0 +l . Es folgt
< ai0 , ai0 +1 , . . . >= xi0 = xi0 +l =< ai0 +l , ai0 +l+1 , . . . >
Für alle i ≥ i0 ist also ai = ai+l , d.h.: Die Zahl
x =< a1 , . . . , ai0 , ai0 , . . . , ai0 +l−1 >
ist ein periodischer Kettenbruch.
191
!
Herunterladen